Vous êtes sur la page 1sur 100

CHAPTER 12

Linear Equations of Order n


A

LINEAR DIFFERENTIAL EQUATION of order n has the form


I)

PO

dy
dx

+ , I

n-i

d'2y+
+ .......+

dx
dx

>' + ,

dx

'i-i

where P 0 Q, P 1 P, .......

, Q are functions of x or constants.

If Q = 0, I) has the form

d'y + ,

2)

d
dx

P" d'
2y

n-i

dx

++

"- 2

dx

+ P. = 0

and is called homogeneous to indicate that all of the terms are Of the same (first) degree in y and its derivatives.
2

Examples. A)

B)

+ Zx d Y - 5

+ 2y

- 3

- xy
dx

= 0,

sin x, of order 3.

of order 2.

dx

Equation Bj is an example of a homogeneous linear equation.


SOLUTIONS. If y = y 1 (x) is a solution of 2), then y = C 1 y 1 (x) where C 5 is an arbitrary constant, is
also a solution. If y = y 1 (x), y = y2 (x), y = y3 (x), .....are solutions of 2), then y = C5 y1 (x) +
C2 Y2 (x) + C3 y3 (x) + ......is also a solution.

A set of solutions y = Y 1 (x), y = y 2 (x), . . . . , y = y, (x) of 2) is said to be linear/v independent


if the equality
where the

C's

C1Y5 + c 2 y 3 + c 3 y 3

........+

are constants, holds only when c 5 = c 2

= 0,

= C 3 = ... =

c = 0.

Example 1. The functions e" and e' are linearly independent. To show this, form c i + c 2
= 0,
where c 1 andc 2 are constants, and differentiate to obtain c i e X - c2e_X
0. When the two relations
are solved simultaneously for c 1 andc 2 ,we find c. = c 2 = 0.
Example 2. The functions

ex.

2eX,

and

e_x are linearly dependent, since

c1ex + 2c 2e X + c 3 e = 0 when c 1 = 2. c 2

1. c 3

0.

A necessary and sufficient condition that the set of n solutions be linearly independent is that:
78

79

LINEAR EQUATIONS OF ORDER n


y i

y2

... y

y.

0.

(n-i)(n-i)

y I
If

y = y1

(n-i)

y.

(n-i)

. . *Yn

(x) y Y (x), ........ y = y, (x) are n linearly independent solutions of 2>, then
C 1 y ( x) + C,y 1 (x) + .....+

3)
is the primitive of 2).

If y = R(x) is a particular solution, also called particular integral, of 1) then


4)

Cy 1 (x) + C,y,(x) + .....+ Cy, (X)

R(x)

is the primitive of I). Note that 4) contains all of 3). This part of 4) is called the conipleznentar.yfutiction.
Thus the primitive of I) consists of the sum of the complementary function and a particular integral.
Attention has been called to the fact that the primitive of a differential equation is not necessarily the
complete solution of the equation. However, when the equation is linear, the primitive is its complete
solution. Thus 3) and 4) may be called complete solutions of 2) and I) respectively.
LINEAR DIFFERENTIAL EQUATIONS with constant coefficients (equationsB) above) will be treated
in Chapters 13 -16. Those with variable coefficients (equation A) above) will be considered in Chapters
17-19.

SOLVED PROBLEMS

1. Show that the equation ._Z -

- o has two distinct solutions of the form y

dx2 dx

If y sax, for some value of a. is a solution then the given equation is satisfied when the replacements
Y

ax

dy
dx

e . -

2
ax d y 2ax
a e
ae . -
dz2

y - dy -
2y
We obtain d 2
-

Thus y e

-x

and

2x

are made in it.


ax 2
e (a - a -2)

0 which is satisfied when a = -1. 2.

are solutions.

2. Show that y Cte + C2e 2X is the primitive of the equation of Problem 1.


Substituting for y and its derivatives in the differential equation, it is readily checked that _C1e_ X + C,e2'
is a solution. To show that it is the primitive, we note first that the number (2) of arbitrary constants
and the order (2) of the equation agree and second that

80

LINEAR EQUATIONS OF ORDER n


e

Since

-x

2x

W
-e

-x

Ze

3e i' 0.

2x

3. Show that the differential equation x'


of the form Y - z'

6x

-x

and

+ 12y

dx

are linearly independent.

has three linearly independent solutions

After making the replacements

dx

rx "
dx2

in the left member of the given equation, we , have


2,3,-2. The corresponding solutions y
x

Since W

2*

3*2 -2x

6x

3r 1 - 4r + 12. = o which is satisfied when

z"(r 5 -

y =

are linearly independent

-2

20 A 0.

The primitive is

- sin x is a particular integral of

4. Verify that y
primitive.

-1 - r(r-1)(r-2)x5

= r(r-1)x' 2.

dx

2y

Cj x' + C2 z 5 + C3x'2,

cos x + 3 ath z and write the

Substituting fory and its derivatives in the differential equation, it is found that the equation is satisfied.
From Problem 2, the complementary function is y
Hence, the primitive is y C51_x + C2 e -

sin

z.

S. Verify that y s mx is a particular integral of x 5 - - 6*

+ 12y 12 mx - 4 and write the

primitive
Substituting for y and its derivatives in the given equation it is found that the equation is satisfied. From
Problem 3, the complementary function is y =
Hence, the priniltive is y Cx 2 +
6. Show that

dz

dx 5

-36

C1 x 3

+ 5

dx2

C1 x 2

+ C3 x

+ ,x

3 e C,x".

2 + in x.


dx - 2y

0 has only two linearly independent solutions of

the form y
Substituting for y and its derivatives in the given equation, we have
is satisfied when a
1, 1, 1.-2.
X

x
ex

eX

Since
e

are .

-2e-

2x

eX and y

eX

fi o, but.

ex.

x
e

ex
x

which

-2x

_29- 2x

x
ex

e(a- a5. 3a 2+ Sc - 2) 0

-2x
-2x

0, the linearly independent solutions


7. Verify that y

eX,

xe,

x2eX.

are four linearly independent solutions of the

and y

equation of Problem 6 and write the primitive.


are solutions. By direct substitution in the given equation it is

By Problem 6, y e x and
found that the others are solutions.
x

C
C

Since

xc

2 K

K
K

xc

+ e

x
xc

^2.

x
xc +

2K

e +2xc x

C
C

3e

2K

xc

^4xeK +

these solutions are linearly independent and the


K

-2K

8. Veril'v that y
primitive.

4e

cos 3x
e

-2x

h1OC 1!
X 10-21

-2K

I 1 3 6 -8 I

-8e

-2x

primitive is

C1e + C,xe +

-2x

-2e

2K
K
x e + 6xe 4 6.

-2x

x c

2 K

Cx e

C4.

-2X
2

s in x are solutions of

+ 13y 0

+ 4
dx

dx2

Suhstitu4ing 'or y and its dcrivaties. it is found that the equation is satisfied.
o, the solutions are linearl y independent.
Since W 3e
cos 3z + C2 sin )
Hence, the prlmlti\c IS y

SCPPLEMENTAR'I' PROBLEMS
9. Shos that each of the lllosvIn sets ot"I .-unctions are linearl y indcper.dent.
c

a sin ax, cos ax

hie

QX

sin bx,

1,

o,, e

Cos bx

x, x 2

2K

tX ,

&'!
e

In x. x

In x. x 2lflx

(a b jc)

Form the differential equation having the given prinlilive.


10.

yCe

11. y

Ce

2K
2K

+C2e

+ C,xc

12.

C1CX +

13.

Ccos

14. y

-3K
2K

Cite 2X +

3x *

A,is. y"+y'-6y0
+

22K

C,x e

- 3y'

e/12

C, sin

ax +

C1 sin

17.

y in Sin(x .-C1 )

19.

12y' -

+ C2 cos x

+ 9y x cos x

4y

Cxi' C 2 +

xC5+C2y+ylny

4 xyl - y 3x'

x /9

zy" - 7' + 4x5 y


)

C,

BY

2y

- 3xy'

2
IS. y =C 1 x+C,x lax +,xin x

18. y 2

eosx + sin x)/32

C1x 2 + C,x 2I x

16.

6y" +

+ (y ' 2 + I 0

yy " + (l2 = 2
yyr' +

and write the


CHAPTER 13

Homogeneous Linear Equations with Constant Coefficients


THE HOMOGENEOUS LINEAR EQUATION with constant coefficients has the form
I)

p0

d'1y

d'2y + .......+

+ P,y

=0

dx's

in which P0

O P, P,....... P,

are constants.

By a convenient change of notation, writing


I) becomes
(P D' +

Now

+ .........+ P

+ P 2 D

P, D '-'

is an operator which

dx

acts

= Dy,

= .-() = DDy = D 2 y, etc..


thdx

j D + P)y = 0.

on y and

ciX

3) P, D"+ P 1 D

-+ ?, I D +

P2D

is simply a much more complex operator. Ho'.ever, we shall find it very convenient to consider 3) at times
as a polynomial in the variable D and to denote it by 1(D) .Thus. I) may be written briefly as
4)

1(D)y = 0,

It can be shown in general and will be indicated by an example that when 3) is treated as a polynomial
and factored as
5) 1(D) = P0(D-m)(D-m2)(D-m,) ...... ...(D-m_1)(D-m,),

then
6)

F(D)y = Po(D-1)(D_)(D.m9) .........(D-m,1_)(D-m,)y = 0

remains valid, i.e., is equivalent to 1) when D is treated as an operator.


2
-
EXAMPLE. In the D notationd 3i d
d. 5 d.2

factored form,

- 4_ + 4y = 0

becomes (D - D -

4D+ 4)y

D -1) (D -2) (D + 2)y 0. Now

(D-1)(D_2)(D+2)y

(D-1)(D2)(

- (D - I)

2) y = (D-1)(D-2){

+ 2 y ) - 2(

dxd

4-

- d

4y) - 1(.- dxd 2 a2

- 4

a 2

2')

(D-1)(-i -

+2y))

4y)

4y)

+4y -

dx5

a2

dy
- + 4y

dx

In Problem I below, it will be indicated that the order of the factors here is immaterial.
82

0.

a and, in


HOMOGENOUS LINEAR EQUATIONS WITH CONSTANT COEFFICIENTS

TEIEEQUATION F(D) = (Dm)(Dm2)(Drn,)

83

(Dm,..1)(Dn) = 0

is sometimes called the characteristic equation of I) and the roots m 1 , m 2 . . , n1 are called the
characteristic roots, Note that it is never necessary to write the characteristic equation since its roots can
be read directly from 6).
TO OBTAIN THE PRIMITIVE of 1) we first write the equation in the form 6).

............

Suppose m 1 m2

a)

Y=
involving

+ C2e*2X + C 3 e I 3 X

Ce t

linearly independent solutions of I) with

Thus in the example above, where

Then

Mn'

d 3 Y-

+ .........

an X

arbitrary constants, is the primitive.

- 4

4y = 0 or

dx 5 dx

(D - 1) (D

2)

(D + 2) y = 0,

the characteristic roots are 1,2-2 and the primitive is

CeX + C 2e 2X + C,e_2X.

See also Problems


b)

Suppose

Y = C je$ t X

is the primitive.

in,.

.......... m,.. 1

m3

m5 = M2

Then

+ ........+ C, e'

+ C, e * '

+ C2 xe'

5-7

In general, to a root m occuring r times there corresponds


-'

tX

+ C2XeIX + C , x t e*X

+ ......+

CXr 1 e tx

in the primitive.
Thus to solve

2LY - 2

- 4 A

8y = 0, write the equation as

(D - 2D 2 - 4D + 8) y = ( D - 2)2 (D + 2) y = 0. The characteristic roots are 2,2,-2 and the primitive is


y = C 1 e ZX

C 2 xe 2X

+ C 3 e 2X

c) If the coefficients of 1) are real and if


ponding terms in the primitive are
+ Be

See also Problems 8-10.

a+bi

is a complex root of 6), so also is

= e

ahi.

The corres-

= e(Ccos bx + C 2 s j n bx)
= Pe3X sin(bx + Q) =
where A,B,C 1 ,C,.P,Q,R are arbitrary constants.
Thus the characteristic roots of
are
12

dx

PeX cos(bx + R),

5y = 0 or ( D2 4D + 5)y=0

2ii. Here a = 2, b = 1 and the primitive is y =

e2

(Ccos x

+ C2 sin

x).
See also Problems II -15.

HOMOGENEOUS LINEAR EQUATIONS Will-I CONSTANT COEFFICIENTS

84

SOLVED PROBLEMS
1. Show that (D-a)(D-b)(D-e)y
(D-a)(D-b)(D-c)y

(D - b) (D - c) (D - a)y.

(D-a)(D-b)(

dx

- cy)

- (b+c)

(D-a)(
dx2

dy - (a+b+c) LY + (ab # be + ac)

(D-b)(D-c)(

(D - b) (D - c) (D - a) y

dx

- ay)

- (a
+b+c)

dz 5

2. Verify that y

dx2


x
cx
Ceax + C2 e + C.

- abcy,

dx

(D-b)(

+ bcy)

dx

- (a+c)

+ acy)

+ (ab+cc+bc)Z - abcy.
dx

satisfies the differential equation (D -a) (D- b) (D- e)y ' 0.


ax

bxcx

We are to show that (D-a)(D-b)(D-c)(Ce + C2 e +Ce )

0.

(D_a)(D_b)(D_c)Ct e ax (Db) (D_c )(D_a ) CL e ax (D-b)(D-c)0

and similarly for the

0,

other two terms.


3. Verify that

y - Ce

ax

+ C2 xe

ax

+ Cox

2e*x

satisfies the differential equation

a) (D _a)S Ct e X - a)(D(D 2- a)C1 e

This follows
w since:

ax

b)

- CD - a)2 o

ax
2
(D-*) C2 e

0.

(-'i)0

2
2a
ax

C) (D - ii) Cx e
= 2(D-a) C1 xe

(D - a)3 y

a nd

2(D-)0 r 0,

4. Find the primitive of (D -a) 2 y = 0 (a) by assuming a solution of the form y =


the equivalent pair of equations (D-a)y
a)

2r

(D-.) y

(D-a)(D-a)x e

ax

u, CD -.)v
ax
(D-*)rx r- ax
e

Ifewrite (D-a)y
Solving(D-a)v

v then (D-a)2 y

r(r -1)x

(D-a)(D-a)y

0. we obtain = C2 e'. Since (D-a)y

r_2 ,
e

0 when r

dz

(D-.)t
Ce

- y

-ax

r -ax

- j e

ax
(C2 e )dx

C + C2x or

0.
nz

is linear of the first order,

ax ,, ax
Ce + 2xe

DISTINCT REAL ROOTS.


5. Solve _Z + LY - 6y
dx
dx2

0.

We write the equation as (D 2 + D - 6)y

(D 2) CD + 3)y

0.

The characteristic roots are 2,-3, and the primitive is y = Ce +

C2e

SX

0, 1.

y z xeax

its solution by the method of Chapter 6 is


ye

and (b) by solving

0.

Thus the equation has two linearly independent solutions y eax and
The primitive is y Cie ax + C2xex,
b)

x ex

HOMOGENEOUS LINEAR EQUATIONS WITH CONSTANT COEFFICIENTS


6. Solve
d,3

LX

dx
D(D - 4)(D 3)y 0.

We write the equation as (D 5 -D 2 - 12D)y 0 or

The characteristic roots are 0.4-3, and the primitive is y

+ C2 e + C2e3X.

+ 2

7. Solve

- 5

dx

We writethe equation as

or

2D 2 -50- 6)y

(D

(D -

2)(D 1)(D+ 3)y 0.

The characteristic roots are 2, -I. -3, and the primitive is y

2x

+ C2 e'

REPEATED ROOTS.
30 - i)y

- 3D

8. Solve

or (0-

j)3

y o.
x

C2 2ex.
The characteristic roots are 1,1,1,l. and the primitive is y - Cie" * C2 xe +
+

(0 -2)(D+ 2)(D . 3)2 y ' 0.


-2x
2x
Coe
The characteristic roots are 2, .2, -3, -3. The primitive is y - Cie

9. Solve

(D

502 -24.')-36)y 0

10. Solve (D -

-110 -4)y

a Q

Of

C o e

-5%

C4xe

(0 + 1)5 (D -4)y a 0.

or

Cox Cox 2 )
+
The characteristic roots are -I- I ,-l.4. The primitive is y e (C-,+

C4e

COMPLEX ROOTS.
11. Solve(D 2 -20 + 10)y
The characteristic roots are

3i and the primitive is

y a eX(Cico 3x Ciii 3x)

Or

12. Solve (D 3 + 4Thy a o or OW 2 + 4y 0.


The characteristic roots arc 0,2i, and the primitive is
13. Solve(0" D

201)

+ 3)y

Or

(D 2

The characteristic roots arc -1 j V, j

Cocoa 2x + C2 s1n 2x.

y a

+2D +3)(D 2 -D + 1)y a 0.

i IT. and the primitive is

e(CjcoS V11x + C2 5in /x) 4 e(C,COUY'X

ceXco(3x+C).

or

Ce1n(3x+C4 )

+ C4iuv'x).

(1)2_ 4)

(D2 + 9) y a 0.
or
14. Solve (D" 5D 2 - 36)y 0
and the primitive is
The characteristic roots are 2, 3i,
y
a

since cosh 2z
15. Solve

(1)2_

Ae 2 '

Be -2X

+ C3 c08 3x + C4 siri 3x

C 1 cosh 2x + C2 sinh 2x + C3 C08 3x + C1n 3x


2x + _2x)

20+ 5) 2 y
y

and

tnh 2x =

2X -

2%)

0. The characteristic roots are I 2i, 1 21. and the primitive is


eX(Cjcos lz + C2 9in 2x)

ze

( C3 coe 2x + C4 sln 2x)

e X { (Ct +C3 x)cos2x + (C,.C4 x)slfl 2x}

85

86

HOMOGENEOUS LINEAR EQUATIONS WITH CONSTANT COEFFICIENTS


SUPPLEMENTARY PROBLEMS

Solve
16.

(02 + 20 - 15)y 0

17.

(D + 02 - 2D)y - 0

IS. (02+60+9)7.0
19.

is
(D -

20.

(D'-4D 13)y -0
9

+ 12D2 - 8D)y - 0

Ans.y C1 e 5 ' +

y - Ct +
7

1C x + Coe-2x

1Sx +

y-. 1 +C,.

, xe -5x
2x

+i..5 xe

2x ,, 22x

+t,4Z1

7 - t (Cl
cos 3x + C, sin 3x)

21.

(D

22.

(D 5 - 02 + 9D - 9)y 0

y C1 e' + C, cos 3x + C, sin 3x

23.

(D + 4D2 )y 0

y . C1 +,z+C, coo 2x+C4sjn2x

24.

(& - 6D 5 + 13D - 120 + 4)y 0

7 (C 1 + C,x )C X

25.

(D 6 + 9D + 24D' + 16)y 0

y - C1 cos x + C, sin x + (C, # C,z)co. 2x


+ (C,+C,x)sin 2x

+ 25)7 0

y C1 cos Sx + C, sin 5x

+ (C,

+ C4z).t

CHAPTER 14

Linear Equations with Constant Coefficients

THE PRIMITIVE OF
+ .........+ P,.. . 5 D + P0 = Q(x),

F(D)y = ( P0 D +

I)

where P0 O.

P, P,, ......P, are constants and

u Q(x) 0, is the sum of the complem-

entary function (primitive of P(D)y = o obtained in the preceding chapter) and any particular integral of I). (See Chapter 12.)
At times a particular integral may be found by inspection. For example, y = i x is a particular
integral of 0 5 - 3D 2 + 2) y x, sinceDy =D t y 0 Such equations occur infrequently, however, and we
proceed to consider in this chapter two general procedures for obtaining a particular integral. Other procedures will be given in the next two chapters.
In each of the procedures below, use will be made of an operator
=

F (D)

defined by the relation

y. When the operator is applied to I) we obtain


L_.F(D)y

C y C

F(D)

F(D)

or
1

2)

. i--Q
Dnin

FIRST METHOD. This consists of solving a succession of linear differential equations of order one, as
follows:
TO OBTAIN

SOLVE

SET
du

= efQe'dx

dx

V=

dv
- iT4V
dx

Dm 1 cL

= U

=e*,%_tx

y = e

- my = w

Six

ue

dx

fwe - o il cix.

As is indicated in Problem 3 below, the following formula may be established:


A) y = etX J' e 1 '2'1

f'"'f........
See Problems 1-6.
87

88

LI\AR EQUATIONS WITH CONSTANT COEFFICIENTS

SECOND METHOD. This consists of expressing


D-m

D-,n2

-_L
F (D)

as the sum of n partial fractions:

Then

D.-m1

B) y =Niel"

Q ""' d,c + N2

*2X

dx

N, e"

Q e' K dx.

In evaluating both A) and B,, it is customary to discard the constantsof integration as they appear:
otherwise, one obtains the primitive rather than a particular integral of the differential equation. The
complementary function is then obtained by inspection and added to the particular solution to form the
primitive.

THE FOLLOWING FORMULAS will be found useful.


Ox

e bx = cosbx - i

cos bx + I sin bx

sin bx - e

Lbx

-e

tbx

cos bx =

21

bX

= cosh bx + sinh bx

sinhbx = ( e bx -

e-

bx

sin br

+ e_t

= cosh bx - sinh bx

coshbx

= i bx

e-bx

SOLVED PROBLEMS
I.

Solve (D 1 -3D+2)y - e or (D-1)(D-2)y x


x
The complementary function is y ..,i +
1
Let u a - ex .
D-2
U - - ex.

Then (D-2)u

or

du

u,
and
Now y
(D-1)y-uor

and a particular integral is

-2x
r -x
ex ue -2xrjix e
dx je
x -e x and

x -x X
.zr
je *'

-XC

X.
The primitive is y C1 eX * C2e CX - me

2.

-2x
2
or (D - 1) (D + 2)2y xc
Solve (D3 + 30 - 4)y - ze
The complementary function is y -

x +

2x

2-2x +
e

-2x
xe

and a particular integral is

1
1
-2X
1
Y ..xe
D-1 D+2 D+2

Let u a ._L.
D+2
Let v

D+2

du
-2x
Then +saxe
dx
u. Then

dx

and u - e

-2x

2x 2x
1 2 2x
fxe e dx -Xe
2

2 2x
2x 1 2 -2x 2r
+ 2v 1re
and v e
f_2 xe .e dx

1 3 .2x

xc


89

LINEAR EQUATIONS WITH CONSTANT COEFFICIENTS


dy 1
_yxe

Then

Now y - v.

andy

-2x -x
e

--e

xe

x + ,,-2x
+

xc

-2x

(x5 +x

+ -x+-).

2 -2x
the remaining terms of the particular
- (z
1 5 + x ) c
18

-2x

integral being absorbed by the complementary function.


3. Find it particular integral of (D -a) (D- b)y
A particular integral is given by
Q z u. Then

Let

Now y

4. Sole

or

e x je

- 3D 2)y

x jQe

eJQe_bX

e 6x fQe -bx dx and

- ay u

dx

D-a D-

bu Q and

Lu -

Then

u.

D-a

Q.
. _-!

y =

-bx
x r (-a)x r
Jye

d. d.

e je

D-1 D-.2
x r x t x
eJe Je

Second Method.

and a particular integral is y =

r5x . -x
c

1 x '4X
--e e
4

The primitive is y

5. Solve (D2 + 5D+4)y

Ci e

+ C2 e

1 2x

12

-,x 3 'ix

-- e -x je

(c -

-xc

Zxf 5x 2x
e e

5z

e
12

1 xrx
-eJe dx
3

1 sx
--.
e
12

dx

5X

3 -2x.

(D + 1)D+4)y

C2 e. and a particular integral is

Cse_X

Y =

(dx)

1 SX

+ -

The complementary funtion is y

First .fei1iod.

+ e

or

3 -x

dx + e

2x

1
)ex
D-1+D1 2

(D-1-2)
)(D
x

5e

5 e

xtxl x
=eje-e dx
3

dx)

(dx).

(D -1) (D - 2)y

The complcmcntar function is y ;CtCX +C2 e


Fr.i Method.

(D + 1)(D +4)

L -

(3-2x).

C- f (-

.1)x

f -

2x)e (dx)2

D+1 D+4
'ix

1 ix

-e )dx

dx

18

The primitie is

1 x5

dx

D-1

1 x

_x C
e J (e - -2 xe )dx

-x 1ex - -21 xex

c (

1 x

e )

11
- -

x.

90

LINEAR EQUATIONS WITH CONSTANT COEFFICIENTS


Second Method.

1
1
-
D+1 D+4

'

D+4

1 e-xf(32xeXcLx - 1
- e

Xf(32x)elXdx

1 e-x
(3eX_ZxeX+2e%)

I -%X(!
e (-*x
e

The primitive is

-X +

6. Solve (D 5 -5D 2 +8D-4)y e

C,e

4x
'x
- xc +.!e )
8
2

1
- 11
2
8

-x.

-4% 1 11

- _Zx

or (D-1)(D--2)2y

The complementary function is y

C1CX + C2 e 2

+ C,xe 2 , and a particular integral is

I
(D-1)(D

2x

C.

_2)2

1
1
2%
C
D-1 D-2 D-2

j e

(2-1)x r (2-2)x

exj e x

2% -2x
e

dx)
)

eXj e x

2 x
XC dx

fe

(a2 x, erx
x

2x 2

e(xe -2xe +2e) =

e (x-2x+2).

2x
., x
2x
2x
The primitive is y
+ Cxe
+ x e the remaining terms of the particular integral
e + C2 e
being absoi-bed in the complementary function.

7. Solve (D 2 +9)y x

CO3 X.

The complementary function is


1

C 1 cos 3x

and a particular integral is

+ C2 sin 3x,

Ix cos x

cos x

D2 + 9

(dx)2

(e (X + c_tX). so that:
It will be simpler here to use cos x
-21x
-5'x re bix r
-'4ix
2
e
+ e
) (dx)
.1
y
J x(e

-3

1 -3ix i- bix 1
-tix
1 -21x
1 . .-iz
(ixe
+ e
je
e
+ -4ixe

ix
I iix
I -six r 1
+ e
e
j ( ire

ix 1 1
2

( xc
8

ix

iiz

-ix

32

ie

ix

lx

IXC

+ f- sin x.

1 2tx
+ - e
) dx

1 4 i
1 2s
+ - Xe
Ic

16

-ix

1 e
x(

x(e + e ) - - t(e -e )
16
64
3X cos

2tx

-liX
761-)dx

ix

1 2ix
1 2tx
If
---Ic
16
32

-ix

+ e
2

1 e

ix

-e
2i

-ix

The primitive is y C 1 cos3x+C,s1ri3xi-x Cos x +


sin X.
a
32


LINEAR EQUATIONS WITH CONSTANT COEFFICIENTS
8. Solve (D 2

+ 4) y 2 cos Cos 3x

cos Zr + Cos 4x.

The complementary function is

(Cos 2x + Cos 4x)

-2tx I
J(cos Zr

2tx

Cos 4x)e

( C 22tr

-7ix( 2iX
,je

C 1 cos Zr + C2

C- 2ix

( e

-2ix I

(-e

2x C -2tx

.j e

-{-X(e

2ix

-2jt

le 2tx- e -2 ix
4

[(e

2ix C

J(1*e-

2'x

ix

+ e

+e

-2ix

2ix

1 Otx
I -2ix
2i
1 -Mix
+ x + -e
- -e
) - e
(x - -e

Mtx

)
+

4 -(e

2ix

+ e

-2r

dx }

) +I
(e

+e

MiX

+ e

-MIX

))

d-

-bit

1 2ix

1 -bix

- e

1 Mix-Mjx
) - -(e
+ e
))


-2ir
je 2ix +e

21

))dx

+ e

1
2 j(C4
-ix
I Mix
bit
-2x
r_{e
e
+1+e
+e
)idr -
1

and a particular integral is

-2
dx - e 2ix C
.j(cos 2x + cos 4x)e

-{e

iii Zr,

- 1 ( - 5-
)(cos 2x cos 4x)
4 D +2t-2

D2 +4
{ e

91

l e

Mix

12(

+e

-MjX

11
-x
sin Zr + - cos2x - 1 c054x.
4

16

The primitive is

12

= C1 cos 23rC2 sin 2x * .! x sin 2z - _! COS 4x.


4

12

-3X

9. Solve ( D 2 -9D+18 ) y

ee

The complementary function is

1
e
(D-6)(D-3)

-3X

Ci e 3x

,bx

t2e , and a particular integral is

bx r -3x

e Je

je

C -5x-3x

.e

-3x
bXe
r...3 (- ee
1 bx fe e (_e 5X)d.x
ee - )dx = -e
J-l3

I e -3x
e bX
9

= -e

The complete solution is y

ce

5x

(C2

l e-Sr
e
e6x.

Note.When the factors are reversed, a particular integral is

e5x J e3x fe

Using the substitution eX v, we obtain


1f 1f V

9 1J
or y
-13

v 2

v 1
1
V
(du) 2 1
'2)d v e
je(---V
9,,
9v2

-Sr

e , as before.

e -sx -bx
e
(dx)2.

92


LINEAR EQUATIONS WITH CONSTANT COEFFICIENTS
SUPPLEMENTARY PROBLEMS

10.

Evaluate, omitting the arbitrar y constant.


__!__.

a)

D1

b)

c)

e x

Ans.

__!_'

d) __!_2 * 1)
D-1

Ans. xe ' e)

D-1
1
(x
+ 1)
D+1

Ans. x

f)

D+2
1
-

D+2

Ans. _L (2 sin
3

ax

sjtj

Ans. x - 2x + 3

.-2x

sin

3x

Ans. - - e
3

-r

3x - 3 cos

cos

ax)

3x

Salve
11,

(D 2 - 4D + 3)y 1

12.

(D2-4D)y5

13.

(D 3 - 4D 2)y

14.

(D 5 - 4D 5 )y

15.

(D -4D)y x

2x
-2x
2
yC+C2 e +C,e
-x/8

16.

e2
(D 2 - 6D + 9)y

yC1 e 5 x +C5xe3X +e 2x

17.

(D2+D_2)y2(1+z_x2)

x
2
-2x
yC 1 e +C2 e
+x

18.

(D 2 .1)y

4xeX

x
-x
X 2
y Ci e + C2 e + e (x - x)

19.

(D2 -1)y

sin 2 x ( 1 .- Cos 2z)

20.

(D2-1)y

(1X)2

1
yC1e +C2e
-X ._+F Cos 2x
x
-x
-x
y z C1e + C2 e - 1 + e ln(1 + ex)

21.

(D 2 + 1)y

csc x

22.

( D2 -

Ans.

y Ct e X + C2 e + 1/3
yC L .C2 e _5x/4

7 Cj

2
5
yC 1 +C2 x^C,x +C4 e2x +C6 e -2x -5z/24

3D + 2)y sin

+ CseX -

5x2/8

X:

y C1

C2 x

coax

+ C, sin + sin in s inr - x coax

X
2x
2z
y C1 e + C,e - e

sin

-x

CHAPTER 15

Linear Equations with Constant Coefficients


VARIATION OF PARAMETERS, UNDETERMINED COEFFICIENTS
TWO OTHER METHODS for determining a particular integral of a linear differential equation with constant
coefficients

I) FD)y = (D'

PD" -1

+ P2 D" 2

+ ....... +

= Q

will be exhibited by means of examples.


VARIATION OF PARAMETERS. From the complementary function of I),
Y = C' Y2

+ C2 y 2

(x) + ........+ Cy,,(x),

we obtain a basic relation


2)

y = L(x) y(x) .4 L 2 (x) y 2 (x) + ........+ L. (x) y,,(x)

by replacing the C 's by unknown functions of x, the L ' s. The method consists of a procedure for determining the L's so that 2) satisfies I).
See Problems 1-4.

UNDETERMINED COEFFICIENTS. The basic relation here is


3)

y = A r5 (x) + B r2 (x) +

r3 (x) + ........+ 0

where the functions r 1 (x) ....... r(x) are the terms of Q and those arising from these terms by differentiation, and A ,B, C., G are constants.
For example, if the equation is F(D)y = x 5 , we take for 3) y = Ax + Bx 2
if the equation is f()y

el + e 5x we take for 3)

since no new terms are obtained by differentiating

eX

i'

Cx + D;

y = AeX + Be
and

e3X;

if the equation is f(D)y = sin ax, we take for 3) y = A sin ax + B cos ax;
if the equation is F(D)y = see x, the method fails since the number of new terms obtained by differentiatingQ = sec x is infinite.
Substitution 3) in 1), the coefficients A,B, C,'"' are fo"nd from the resulting identity.
See Problems 5-6.
The procedure must be modified in case:
a) A term of Q is also a term of the com'emcntary function. If a term of Q , say u is also
a term of the complementary function corresponding to an s-fold root a, then in 3) we introduce a term
ii plus terms arising from it by differentiation.
For example, in finding a particular integral of (D - 2)2 (D + 3)y = e 2' +

93

94

LINEAR EQUATIONS WITH CONSTANT COEFFICIENTS


the basic relation is y = Ax 2 e 2 ' + Bxe 2 ' + Ce 2 ' +
+ Lx +,, the first three terms arising from
the fact that the term e2X of is also a term of the complementary function corresponding to a double
rootm = 2;hence, use is made of x 2 e 2X and all terms arising by differentiation.
Sec Problems 7-8.
b) A term of Q is x 'u and u is a term of the complementary function. If u corresponds to an
s-fold root n, 3) must contain the term x'u plus terms arising from it by differentiation.

For example, in finding a particular integral of (D - 2) (D + 3)y = x 2 e 2 + x 2, the basic relation


is

+ Bx e e 2: +

+ DX 2 e 2X + Exe

4.

4' Ox 2 + lix + J,

the first six terms arising from the fact that e2 is a part of the complementary function corresponding
to the triple root m 2.
-
See Problem 9.
SOLVED PROBLEMS
VARIATION OF PARAMETERS.
I. Show that if y Cy1 + Cy, +. C3 y3 is the complementary function of
F(D)y (D + P1 D 2 + P2D + I', ) y . Q then
I)

L571 Loy* + L,y,.

where L, L,. L satisfy the conditions


Ly5 + Ly2 + Loy* 0
4)

Ly,' + Ly Ly
9.
7 ,,
I II
L jy + L 2 72

. 0
U

Q.

s a particular solution of the differential equation.


We obtain, in view of A), by successivel y differentiating
B)

y - L1 y 5 L,72 + Loy*
Dy
L5y + Lgy + Loy' + (Ly 5 + Ly2 + Lya)
L2y + Loy' + Loy'
2
D y L 1 y' + LyLy' + (Ly 5' + Ly + L,'y) L1 y L2y + L,y
D 3 y
L1y + L 2 y + L1 y + ( Ly1 + L,y2 + L,y,) - L 1 y'1 + L2 y + L.;.'+
Q.
I ,?

Then F(D)y

L(y,' +

+ P2 y

II

P,y 5 ) + L 1,

P1y + P2 y

P3yj

+ L 3 (y' + P1 7 + P2y + P3 y1 ) + Q
= L 1 F(D)y 1 + L 7 F(D)y, + L, F(D)y 3

+ Q

0 + 0 + 0 + Q

Q.

since y. 72. Ys are solutions of F(D)y 0.


In using this method:
a) Write the complementary function.
b) From the function I), which is to be a particular integral, by replacing the C's of the complementary
function with L's.
c Obtain equations B) by differentiating I) as many times as the degree of the differential equation.
After each differentiation, set the sum of all terms containing derivatives of the U s equal to zero,

except in the case of the last differentiation when the sum is set equal to Q. The equations obtained by
setting the sums equal to zero and are the equations A,'.
d) Solve these equations for L. L.I. ........
e) Obtain L 1 , L. ....... . by integration.

VARIATION OF PARAMETERS, UNDETERMINED COEFFICIENTS


2. Solve

(D2-2D)y

esinx,

formthe relation
obtain, by differcntiation

L1
L2e
2
2r
+ (L + L,te
2L2c

Dy

and set

2x

y C + C2

mplemcntary function is

,'

I)

$ 2x

+ L2 e

Since no' Dy = 2L 2 e 2 , D 2y = 4L 2 ,2x

0,

, 2x

and we set

2L2 e

slnx and L 2 - e(s1n x + cos

Thus, L, =

From I), L = _L e 2X

er

sin x and L 1

= -

2L 2 c

2x

= Q r e snx.

x).

cos z).

A particular integral of the given equation is


L1 + L 9 e 2 '

and the primitive is

C1

cos x) - e(sjn x + cos x)

e(s1fl x -

+ C2e 2X -

sin .s

sin

3. Solve (D 3 +D)y = csc x.


The complementary function is y = C + C2 COS
Cs in
y
L, + L 2 cos x + Lsin x
From the relation
X 4

Dy

we obtain
I)

and set

(-L2sinx

Lcosx + L31sinz)

L3 cosx) +

+ Lcos x + Lsin x

Then

-L2slnx + Lcosx.

Dy

D 2 y ( - L2cos x - LsIn i ) + (-.Lsin x + L2'cos x).

and we set

2)

-Lsinx + Lcosx

Then

- L2cos

3
Dy

and we set

sinx -

csc x and

Solving 2)and 3), L

x Lsin x,
L 3 cos

-Lcos x - LsIn x

3)

Adding I) and 3).

(L2

-1

Q
-

cac X.

ln(csc x + cot x).

L 2' = - cot x, so that

and

x) + (-L 2 cos x - Lasin x)

L3

-x

- in sin x.

and L,

Thus, a particular integral of the differential equation is


y

- in(cse x + Cot x) - cos x in sin x - x Sin x,

L1 + L 2 cos x + L 3 sln x

and the primitive is


Y = C1 + C2 cos x + C2 sjn - ln(csc x + cot x) -coax In sin x - z sin

4 Solve

(D 2 -60

5X/x 2

The complementary function is


From the relation

y=

1)

Dy
j (

C5e

+ C9xe 3x

L1e 5X
+ L2xe5'

we obtain

and set

x.

(3L1 + L 2 ),
e

3X

3x
Xe

3x

o.

3L2xc

+ ( e

+ L2xe3x

95

96

LINEAR EQUATIONS WITH CONSTANT COEFFICIENTS


Then D ty
and we set

I
I5x
gL1 + 6L,e 5x 9L,ze 5x (3L 5 + L,)e + 3L2xe

(3L 1 + L 2I )e 3 + 3 2 x

2)

e5xlx2.

Solving 1) and 2) L'1 - lix and L = liz'. so that L1 .


- In z and
Thus, a particular integral of the differential equation is
L (5x
+L2xeSr

and the primitive is y

c 1 e

- a Sr

lax - c 5x

- a in x

UNDETERMINED COEFFICIENTS.

X
Solve (D 2_ 2
stn z.
complementary function is y C 1 + C1ex .

Y
Then
and

DY
2
Dy

As a particular integral, we take

AeX sin BeX C08x.


(A_B)eXajnx + (A+B)eXcoax.
x
- 2Be slnx 2Aex coax,

(D2-20)y _ 2AeX 5Inx - 2BeX cos z

eXslnx Q.

Equating coefficients of like terms, -24 1 and -28 0. so that


A - and B 0.
Hence, a particular integral of the differential equation is
Y

and the primitive is

Aer sin x + BCX coax - 4e sin x,


C 1 + C2e -
sin x.

This was solved above as Problem 2.


Solve 0 2 -21)+3)y

x 3 + sin X.

The complementary function is

9'v(Cl

x + C2 sIn /2- x) As a particular integral, we take

Y Ax ' +Bz'+Cr+E+F sin z+G cog x

Then

and

Dy
2
D

3Ax'+2Bx+CC sin x.pCos


6Ax+_F sin x_G co g
3Ax3+3(B_24)x2+ (3C-48+

(D2-.2D+3)y

z,

4)x+(3E-2C+28) + 2(F#G)slnx + 2(G-F)cosx

= x 5 + sin X.

Equating coefficients of like terms, 34


and C-g; 31-2C+28..0 and

1 and A z 113; B - 2A o and B 2/3; 3C-45+ 6.4 0

E -8/27; 2(F+C)=1, G-F=o and FC.

Thus, a particular integral of the differential equation is


Y

1
2 2
2
8 +
x, + - x + -
x -
(sin + Cos x)
3
3
9
27
4

and the primitive is


Y = Cx(C1COS /z C2 sth v'x) +

(9z3 + 18x 2 + 6x -8) +

( 51fl COSx),

VARIATION OF PARAMETERS, UNDETERMINED COEFFICIENTS


7. Solve (fl3 2D 2

97

e + x2.

-D-2)y

CSeX + C 2 e
The complementary function is y
+ C3 e 2 . Since e occurs in Q and also in the complementary function corresponding to a root of multiplicity one, we take as a particular integral

I)

Then

Ax 2

Bx + C + Ex

+ Fex

2Ax + B + EXCX

Dy

(E+F)eX.

fly

2A + ExeX + (2E+F)eK.

Exex + (3E+F)eX.

and

24x2 - 2(B+A)x + (4A B-2C) + 6E C X

(D5 +2D 2 D-2)y

ex + x2.

Equating coefficients of like terms, - 2A


1, B + A
0. 4A -B -2C a, 6E
1; hence, A
f, C - . E J, and F is arbitrary. Now F should be arbitrary here, since C, e ll is a term of
the complementary function. Thus, in writing I), the inclusion of Fe' was unnecessary.

Hence, a particular integral is


and the primitive is y

8. Solve

(1)2

C2eX +

41)+4)y
y

xc

C2 e

+xe

x 2 x

xex,

* Ce- .2 +

x -

2x

2x
2x
The complementary function is y
Now
Now C is
i a part of Q and also occurs in the
Ce + C2 xe
r nmplementary function corresponding to a root of multiplicity two. As a particular integral, we take
S

Ax e

2x

+ Bx e

Note that terms involving xe


and
function with arbitrary coefficients. Then
S 2r
i.'tx C

Dy

5A + 2.B)x e
4

D 2 y

(D2-4D+4)y

1/6,

2 2x

2 2x
+ (3C + 2E)x e

3 2x

2Exe

2x

2x

LO.

Thus a particular integral is


and the primitive is y
9. Solve

. 5 2x
c.x

are not included, since they appear in the complementary

(48 2C)x e

Equating coefficients of like terms,


Bo, C

e
(.20A + 168 +IC)xSe2X + (128+12C+ 4E)x2e2X + (6C# BE)xe
5 2x
22x
2x
2x
5 2x
2x
JAx e
I2Bx e
6Cxe + MeX C + Xe

4Axe2' (20A+48)z

and

2x

2x

22
(D 4)y
x

sin

Ce

128 0,

1,

1
1

- x 5,2x + 20
6

2x

20A

+ C,xe

1,

2E

0;

+ 2Ee

hence

2x

A 1/20,

5 e 2x

_L 32x

20

6C

2x

+ 1
-

x3e2r.

2x.

The complementary function is y

C5 cos 2x + C2 sin

2x.

Since x 2 sin 2x occurs in Q and sin 2x is a part of the complementary function corresponding to a
root of multiplicity one, we take as a particular integral
y = Ax 3 cos 2x 8x 5 sin 2x

Note that H co
Dy

Cx 2 cos 2x + Es 2 sin 2x Fx C08 2z Cx sin Zx.


K sin 2x is not included, since these terms are in the complementary function. Then

2Bx5 cos2x - 2Ax 5 sin 2x (3A 2E)x 2 cos 2x (3B-2C)x 2


+ (2C+2G)x cos 2x * (2E-2F)x

sin

2x

F cos 2x

sin
C

2x

sin

21

98

LINEAR EQUATIONS WITH CONSTANT COEFFICIENTS


02y

- 4Ax5

cos 2x - 4M Sin 2x + (12B-4C)x 2 c os 2x + (-12A-.4E)x 2 sin 2x

+ ( 6.4+ 8E-4F)x cos 2x + (6B-8C-4C)x Sln2x + (2C+4G)coa Zx + (2E-4f)sin 2x,

and
4)y

12!3x cos 2x - 12412 sin 2x

(6A+8)x cos 2x (613-8C)x sin 2x

+ (2C+4G)cos2x + (2E-4F)sin 2x

x2 sin 2x.

Equating coefficients of like terms, -12A 1.

12B 0, 6A + 8E = 0, 6B - 8C 0,

2C+4C=0, 2E-4f=O; hence . A-1/12, BO. CO3 E

- ..! x cos 2x + ! x 2 sin 2x

A particular integral is y

and the primitive is y

12

16

C1coa 2x+ C5 sin 2x -

32

COB 2x +

12

1118, F

16

1/32, Go.

x cos
Sin 2x +

32

x cos 2x.

SUPPLEMENTARY PROBLEMS
Solve, using the method of variation of parameters.
10. (D2 + 1)y Cac x

Ails. y Ccos x + Cs1n x + Bin x in sin x - x cos x

II. (D' + 4)y = 4 see 2 2x


12.

Ans. y Ccos 2x + C2 sin 2x -1 + sin 2x 1n(ec 2x tan 2x)

(D2 - 4D + 3)y - (1+ e) 1 Ans. y Ce + C 2e 3x +


2

13.

CD - 1)y - e

14

- 1)y

-x

sin e

-x

cos e

-r

Any. y C L S

-x -2
(1+e )

-x -

Apis. y C t e X + Cse

2X + ( ex_

e x sin

1121 + e)

e-x

_ 1 +e_XIn(i+ex

Solve, using the method of unie1ermiid coefficients


15 (D2 + 2)y ex + 2

'-Ins. y CCOS /'x + Cain 4

16. (D2 - 1)7 - e x sin 2x

Anc. y

17. (D2+2D+2) y x 2+Btn.x

,In.c. y = e1 (Cc c-

18. (D2 - 9)3' - X+52X_ sin 21


19. (D 5 + 3D 2 + 2D)y = x 2

i. 4,I + 8

C2e' -

Ans.y -

15.
1

(D 2 + 1)y

- 2in x

4_i

(D5
-D2 -4D
+ 4)y
2 - 4%

?.x

2x 4 cos 2x)/8
- 1)2

sin x) +

4-

(sin z- 2 cos x)

- x/9 - e2X/5 +

sin 2.s

(Use Ax, Bx 2 +
ins y C1 +

14.

+ p5/3 +

+ C3 e

2X

1
1 2
11
+ -x5 + -x +
8
4
4

Ans. y -.C1 ccrox + C1 81nx + 2x coax + x


22

+ -5xe

21

sin x

+ e 21

Ans. y C1 e 1 + C2 e 21 +

c3e

-2x

12
2


+X 4-

1e
-X
6

2x

CHAPTER 16

Linear Equations with Constant Coefficients


SHORT METHODS
A PARTICULAR INTEGRAL of a linear differential equation f(D)y = Q with constant coefficients is given
1 Q-Forcertain formsof Q the labour involved in evaluating this symbol maybe considerably
byy
=( D)
shortened, as follows:
If isof the form e.

a)

= __ aX =

1(D)

1 e,

F(a)

1(a) A 0.

0. and Problems 4-5 when 1(a) = 0.

See Problems 2-3 when 1(a)

IfQisou the form sin(ax+b)or cos(ax+b),

b)
y

sin(ax+b)

cas (ax +b) =

cos(ax+b),

1(a2)

Ij

0,

0.

F(a2)

1(D 2 )

0, and Problem 12 when f(.a2) = 0.

See Problems 7-I1 when F(a2 )


c)

F(a2)

F(a2)

1(D 2 )

sin(ax+b),

If Q is of the form x,
1 x
Y
-

= (a0

+ aD +

a2 D2 + .....+ aD')x'.

a0

0,

1(D)

obtained by expanding -1

in ascending powers of D and suppressing all terms beyond Do , Since

1(D)
"a
Dx = O hen n>m.

d)If

See Problems 13-15.


eaxV

is of the form COXV(x),

= e

ax

V.

F(D + a)

See Problems 17-20.


e)

Y = J.---xV =
1(D)

If Q is of the form xV(x),

X I

V- F'(D) V.

1(D)

(F(D))'

See Problems 21-23.

SOLVED PROBLEMS
I. Establish the rule in a) above.
Since when
F(D)e
14

ax

20x

ax

ye . Dy'aeax Dy.ae .
- ax

P,.D e
r

v ox

.
i..Pa
e

F(a)

ax

e .

Tax

Tax

ae

............., De
ax
1
Hence, -
F(D)

P(a)

ax


COEFFICIENTS

2. Solve (D - 2D 2 -50 + 6)y

or (0- 1) CD -3) (D + 2)y


The complementary function is y Cex + 2 e3x+ C3e -2r
1
e
(0-3) CD 2)
1

A particular integral is y

C1eZ + C2 e 5x

Hence, the primitive is y

(4- 1)(4 -3)(4 + 2)

'x

1 'x

e
18

3'lG

-2x

C* C -

- e

4x.

18

x
2
3. Solve (D52
- 200_ 3D 6)y
(e + 3)

The complementary function is. from Problem 2, y Cl ew + .-.2 e 3X +


I

A particular integral is y

2x

(e +3)

(D- 1)(D-3)(D #2)

C
3(1)6

1(-1)4

The primitive is y

4. Solve (D -

202

C1CX

Ox

(D- 1)(D-3)(D+2)

3e

18

C2e5x

-2x

Ox '
C

CD- I)(D-3)(D# 2)

(D-1)(D-3)(D+2)

,,

Ox

Ox

Ox
,. e -Ox + e
3e
-
+ -3
18
2
2

-504 6)y e 5x
x
5x
Cj e + C2 eCo
e -2x

The complementary function is y


A particular integral is y

(D - 1) (D - 3) (D + 2)

e. Now 1(a)

1(3)

0. and the short method

5K
e
)

does not apply. However, we may write


1


x
e
(D1)(D3)(D+2)

1
1
5x

D-3(D-1)(D#2) e

1
1
x j 1
x
e x5x -5x
e
dx
e
= ze
i
10
10
-Ox
SX,
C e x
The primitive is y
C
2 e #
+ xc /10.
e
Ox
x
5. Solve (D i - 5D 2 + 8D-4)y y e
2e +3e -x

2x
Ct ex + C2 e 2x + C3 xe
, and a particular integral is

The complementarN function is y


1
Ox
-e
(0- 1)(D_2)2

I ' e 2x)

1
1

C Ox

CD-2) 2

Ox

fJ (dx)

2
0-1
+ 2e

(0-1)(0-2 )
+

2(

2e

+ ( D-

3
1)(D-2y

3
___ ______
4
D-i (D-2)2e
(D-1)(D-2)2
x
3
-x
C
(_2)(_3)2

1 -x

-e
6

2 OX

-Xe
2

21 x

-x

- 6e

5x


SHORT METHODS
t i e* + ,,

The primitie is y
6. Establish the rule in

hi

2*

Cox*
.x

1(D 2 )

1 2 Zr

II
1 -x
x - e

above for coI(ax 46).

Since, when y coax 6), D 27


Dy

2r

-a cos(ax +

(-a 2 ) 2 eoZ(Qx + 6),

6), Dy

(_a2)?Co$(ax+b), then

COs(az .6) -

E P,.D 2 ' coo (ax 6) P,.(-a 2


1
Hence, ii-- Cos (ax + 6)
F(-a
1(D)

F(-a2) con (ax + 6).

Cos (ax +6)

CoS(ax 4 6).

7. Solve (D2 + 4)y a In U.


C 1 coa 2* + C1 sln 2*, and a particular solution is

The complementary function is y


Y a

__.L_ sin ax

1
_(3)I

D1+4

The primitive is y Ccoa 2s + C,sin 2* -

sin
sin as
- as.
4
sin 3*.

8. Solve (0' lob' + 9)y eos(2z +3).


The complementary function is y Ccos x + c7 sin z # co coa 3* c4 .in 3*, and a particular
integral is
1
1
1
- cos(2x+3).
cos(2x #3)
Cos (2x .3)
7 I
15
(-3)(5)
(0'. 1)(b'+ 9)
The primitive is y e C 5 cos z +
9. Solve (D+3D4)y

C,sth x C,cos

3* C1 sin 3* -

is

Coa(2x + 3).

a sin 2*.

The complementary function is y C1.* +


7 1

sin Ix

D'+3D-4

and a particular integral is


1

sin

(D-1)(D+4)

2x.

The operator here is not of the form ..._.L.. and the short method does not apply. However, we may

F(D)
use either of the following procedures to shorten the work.
a) y -

(D+1)(D-4) sin 2*
(D2-1)(D2-16)

1
sin 2*
(D.-t)(D+4)

._!_(... 4 1 1n2x_8c08 2x_4 8 1 0 2x)


lo

b)

sin

2* -

D'+ 30-4
- _!_ (30+8)
100
The primitive is y

sin 2x

1 2*

(-4) +3D-4
-

100

-ax

+ C,,2 e " -

- (D'-3D-4) sin 2*
-L

100

_.!(481fl2xP3 coo
50

sin 2*

1 (4 sin 2* 3 cos 2x.

3D+8 sin 2*

9L)'-64

3D-8

(8 cos 2* +8 sin 2x)

2X).

50

(4 sin 2* +

3 coo 2*).

102

LINEAR EQUATIONS WITH CONSTANT COEFFICIENTS

10. Solve (D + D2 + D + 1)y

sin 2* + cos 3*.

The complementary function is y


(sin

2*

(D2+1)(D+1)
_!_!_slnZ* 3 D+1
.! (D-I)sin
15

Ccos X + C2 81n

cog 3z

80

and a particular integral is

cos 3*) = I
sin 2* +
(D2+1)(D+1)

COS

(D-1)cog 3x

15

3*

(D2+1)(D+1)


3D2-i

80+j

C,e,

(2 cos 2* - sin 2*) -

1 (3

sin 3x cos 3* ).

80

The primitive is
Y

C1c0sx

+ C9 81nx +

C,e

cos Zx - sin 2*) -

sin 3* + cos 3*).

11. Solve (D 2 -D+1)y sin 2*.


The complementary function is

Y - 1
0 2 -. D+i

e(C1 cos

+ C,sin,4z),

sin 2* -

(-4)-f).1

.! D -3)ith 2*

and a particular integral is

sin 2* -

sin 2*

0+3

-1(2 cos 2* - 3 sin 2*).

The primitive is y eC1 eos vi x

Cgsin +v' z +

2 cos 2* - 3 sin 2*).

12. Solve (D 2 + 4)y cos 2* + cos 4*.


The complementary function is y C1 cos 2* + C,sin 2*, and a particular integral is
(cos 2* + cos 4*) .__ L_ cos 2* +
0+4

0+4

cos 4*.
0+4

The method of this chapter cannot be used to evaluate __!_ cos 2* since, when 0' is replaced by

+4
-4. 0+ 4 0. However, the following procedure may be used.

Consider!.__ cos(2 +
02+4

-
1
co
+ h)z
co(2 + h)x
-(2+h)2 + 4
4h +h2
1
-
(cos 2x - Iix sin 2* - ( hx)'Cos 3* ..........
h(4 + h)

by Taylor's thercom. The first term, cos 2*, is part of the complementary function and need not be considered here. Hence, a particular integral is
1

CO5(2+h)x

h(4+h)

D 2 +4

(hz

$ifl 2* + 9(hx)coe 2* - ..........)

(xsinZx + hz2coe2x - ..........)


4 +h

Letting h-.o.wcobtain -.L_ cos 2*


D+4
is y

C5 cos

2* + C,sin 2*

Problem 8, Chapter 14.)

xs1n

2* -

sin 2x. Since -.!__ cos 4.z


D+4

Cos 4x.

- .1 cos 4z,

12

the primitive

(Compare this solution with that given in


SHORT METHODS

103

13. Solve (202 +2D+3)y - xt+2x-1.


e(C1 co8

The complementary function is y

1
(xt +2x-1) - +2D+3

is
- 2 ( + 2)
(x 2x 1)
39

v'

x + C2 stn

Ax),

1 - D - -!. D2 )( z 2 +
27
9

and a particular integral is

2
2
(2) - -it
x + - - 25
27
27
3
9

2i
1
by direct division.
+ ..........)
(... -
2D.20+3

Note:

e ( Cj cosh'x

The primitive is y

C,

sin 4vx)

ii
2
25
-r + -x - 3
27
9

It

14. Solve

-20+4)y x +3x2 Sx +2.


Cie- 2 e(CcoS x + C5 siri x )

The complementary function is y


1

1+
(4

(x 3x 5x#2)

D5 -20+4
i
13
-x + * +

The primitive is y

7
8

3a
5
-x
--x 2

,x
@X
x
Ce_ +

+ !D
16

and a particular integral is

!' - ! D'.)(x' 4 3x- Sr 2)


32
64

s
z + C1 aia x) + -i x + 1-z
4

3
2

4 x t

5
4

7
8

- -x - -

15. Solve (0' - 4D2 + 30)y


The complementary function is y

1 1 2
26
8
(z+.x+
)
D3
27
9

The primitive is y

C,.x C. 1 3x

and a particular integral is

1
1 1
4
1
2
)x
- - (- + - +
- '5(3'-4D+3 0 3 9

1
- 0(02 -40+3)

C1 +

1 3
26
4 a
4 -x + x, since
27
9
9

-x

ffzsdx.

4 t
26
c, ex + c, + x + x
4 x.

C 1+

27

16. Solve (D'+ 20 3 -34y xt+ 32x4 4 sin z.


The complementary function is y
1

C C,x + C, eX + C

and a particular integral is

(z2+32X+4 star)

D2(D'W-3)
1
Dt(02#2D_3)
1


1
x'} +
( D1+2D_3

lx +

+
4(44-3)

star

D'(D'+2D-3)

D'(D'+2D-3) '

(-1)(-142D.-3)

star

104

LINEAR EQUATIONS WITH CONSTANT COEFFICIENTS

D23

..

1 3

---(-i -

27

c,

c,x

+x

(3x

y aQxU

D'y

a U((D.c)U

ffY

I)

e(D+)'U.

F(D).0

Let V

Dy
xD(D

aeU

)LI

I P,.D (a C.) I

eSX (D ea) 9

and

.SiV

_L

a e(D+a)1U,

P,, (D

3X

F(D+a)U.

Then, from I),

.a)

, Si V

{F (D ) , Si -.__
V) .
P(D)
P(D+a)

V
P(D+a)

1
-

is

t x

a C1 .

z a

(D.3)'-4

a X
( 1
. - !D .
5

and a particular integral is

+ C,.-x,

ia

D'-S

The primitive is

2 sin x).

x a5

The complementary function

)U

U a Si

F(D)

( D 1 -4)y 0

2
3 ix
-
+ (coax +
20
5

F(D)eU aF(D+ a)U.

1
F(D

18. Solve

a e (1) + a)U,

etV

sothat u

F(D)e a' _____


V

(CoSx + 2 sin

+ 8z + 28)

e(D'+2aD+a 1

and
9 Si

P(D+s)(

108

17. Establish the rule in d) above by first showing that


Since when

->.

3 ix
- C
20

2-' +7z)

2
sinx
D-2

e 2x - 2.__!. s in
20

The primitive is

I I 9x2+12x414)

3 ix
e 20

2
2
_!D_ D)z
+
9
27

_! _1

31

25

xz

12
-

125

Si

ii
)z

D'+6D+5

z +
25

62
125

a SX(25162)
0
CI O' 1
..a
125

19. Solve (1) 1 4.

21)+4)y a x sin 2x.

The complementary function is


Y

x
x
a sin 2x a

e(Ccos

vi

1
sin
(D+i)2+2(D.1>.4

zx
C T7-3 sin 2x a

4D-3
16D - 9

The primitive

is

sin 2z a

sin viz),

ax _

and a particular integral is


1

sin U

D2.SD+7

cx

- e (4D-3)sin 2x a (8 cos 2x - 3am 2x).
73

y a e(C5 cos viz + C2 aln viz) -

(5 Cos 2z - 3 sin 2z),

SHORT METHODS

The complementary function is y C, , x


(2*15* + 3cX

Col

20 - 2

Xe

5*

cos 2*

10)x

3 r
2
5*
-1c
(r -x) - - e (Co.
8
2

2* un

The primitive is y =

+1

1
co. 2*
-4-20

D D+2

3 x D-2
- -e Cos 2z
2
D2-4

e Cos 2x

D2-4D+3

3*

D22D

Dt +20

+ 3

D'- 4D+3

2C

2, 5* - 4

and a particular integral is

+ C,.,

D2 -4D+3

105

2xe + 3e Co. 2*.

20. Solve (D 2 - 4D + 3)y

+
1
- e 5* .(2x-1)
2
D

Zr).
e

*(
3* 2
(z - x) - 3 e CO. 2* + sin 2x).

21. Establish the rule in e) above by first showing that F(D)xU rF(D)U F'(D)U.
Since when y - zU, Dy xW U. D2 y xD 2U 2W.
D'y

ZDrU rD'

U - D'U + (.. D')U. then


dD

ZP1f(xU)
r

I) F(D)rU

Let V F(D)U

11

x -!1(D)

F(D)

V. Then, substituting in 1),

1(D)
F' (D) __ L V, x
1(0)

1(D)x--V 1(D)__.L V
1(0)
1(D)

and -J_. xV

__L

so that

P,.(-ff)U zf(D)U + F'(D)U.


dD

YP,tfU +

P(D)

V - F'(D) V.

v - -J--- F'(D) _L v

1(D)

1(D)

1(D)

-F'(D)--. V,
Z-J--V
1(D)
1D>

22. Solve (D 2 +3D+2)y x-sln 2*.


The complementary function is y - C j e_* + C21 2 * , and a particular integral is
Y

x sin 2* x

sin 2*

(D2+3D+2)2

D2+3D+2

D2 +3D+2

2D.3

sin 2* -

20+3
1
sin 2z
- xsin2r -
3D-2
D +6D5+1302+12D+4
1
sin 2* -
30-2
30+2
sin

x
9D24

2D43

1 (W+3)(30-8) Ia 2*
4 9D264

+ -

24 sin 2* + 7 cos

-. x(3 cos 2* + sin 2*)

y Ce

2*

200

20

The primitive is

sin 2*. replacing D2 by -4,

(-4)'+6(-.4)D+13(-4)+12D+4

-Z

C2 e

-2*

30*-I

200

cos 2* -

5x

12- sin 2*.


100

106

LINEAR EQUATIONS WITH CONSTANT COEFFICIENTS

23. Solve (D' 1)y

sin 3x.

The complementary function is y


Y -

1 x2
02_1

Sifl

1 z sin 2.x -
3x x
D'-1

D'.- 1

z2

sin 3z - x

(D

0'-i

2.0
z sin 3x
(D'l)'

I
20_ sin 3x - 2D (x
sin 3
1)2
(0' D'-202s1

sin 3x - x

X '

Ci e" C,e, and a particular integral is

sin 3z - 2D

12

4D3-4D sin3x)

(D'-20'+l)'

isin3x}

(x

(0'- 1)2

8D' sin ax
(D- 1)

1'
9 sin 3x
- -1 x 2 sln3x - -3 cos
x
3x - - D(x 51fl3x) +
10
50
50
125
- .!
10

2 sin 3x -


x
Ce + C,e-

The primitive is y

24. Solve (D -

3!)2

x cos 3x +

25

250

sin ax.
2
25x-13
sin 3x - -3 x cos ax
250
25

-60 4 8)y =

The complementary function is

C1eX + se + Cae_2.

1
2

By a):

1
39
-x (----x--).
e
28
784

28 784

e-5; -

y x

D3-12D2.39D-28

1
-3x (----D)x
39
e
By e)

-60+8

1
(3332

y e -5x

and a particular integral is

ze -5x

3D' - 6.0-6
0 5 -3D'-6D+8)'

- ..! xe3' - 3D -60-6


28
(28)
The primitive is

= - -! xe' - 39 -3x
28
784

C1eX + Ce +

- !

(28 39).

25. Denote the real and imaginary parts of a complex number: by Rel:) and Im(z) respectively. An alternate
short method for Problems 9-I1 makes use of sin bx lm(e t ) and cos bx Re(etb9.
Consider, for example.
112iX
F(20

F(D):
+

F(3i)

(D + D' +.0. 1z
-

3+ 6i

L 1 (co2+ Ls1n2x)
15

. 3 tx

for which

111Z
8+ 241

L_ 1 (cos3x+ isin3x) -
80

is a particular integral. Then


Re [z Re
12 I- .i- ( 2sin2x cos 2x) - '(3 sin 3x + cos 3z)
15
80

SHORT METHODS
is particular integral of

107

F(D)y = cos 2x + cos 3x


15(2cos2x - s1n2x) +

is a particular integral of

F(D)y

1 (3 cos 3x - sin 3x)

sin 2x + sin3x
Re[z 1 ] + Im42I

F(D)y = cos 2x + sin 3x .

is a particular integral of

Im(z 1 1

and

+ Re 4 2 1

is the particular integral in Problem 10.


SUPPLEMENTARY PROBLEMS

Find a particular integral.

26.

(Y + I.) + I)y = e 3

27.

(D i - 1)y =e

28.

(D_2) ye

29.

(D 4 -

30.

(D5+1Y

(f)2
31.

6, x - 3c - 2x + 5

x
y xe /2

x +xe 2%

y=e

52x
+xe
/6

sin 2x

sin 2x

COS X

33.

(D 5 + D2 + D + 1)y

34.

(D2-1)y

35.

D"(D 2 - 1)y

36.

2
-2%
5
+ cos 3
(D 2 + 2) y x + x + e

37.

(D 2 - 2D - 1)y

38.


2%
(D-2) 2 ye Ix

40.

15

= COB

e +e

-x + sin x

cos 2x

+ 2xe ) - x

(Biflz COB

y = x -2

2
2

= z

-1)y xe

x - sin x)

"

(D 2 + 5)y

(COS

y -

+ 4y = sin 2x

32.

39.

Ans. y = e 3 '/13 + 2e - e 2 + 5

-(x + 30x")

(x' +r 2 3%

COSx

cos

2x

5%

(D 2 + 5D + 6)y = e

y -

-2%

2
(see x)(1 + 2 tan

-1)+

x)

y.e

In

e(4x -3)
-

2% tan

- ! cos 3x


CHAPTER 17

Linear Equations with Variable Coefficients


THE CAUCHY AND LEGENDRE LINEAR EQUATIONS
THE CAUCHY LINEAR EQUATION

I)

dy

..1d"-ty

dx"

dx""

pox"+ PX

in which p0

2)

p .......

dy
+ .....+ p_ 1 x -

p y

dx

Q(x),

p, are constants, and the Legendre linear equation

(ax+b)"_Z + p(ax+b)"t d"-"y + .....

p _ (ax + b ) X + py = Q(x).
dx
dx"
dxt
of which I) is the special case (a = 1, b = 0), may be reduced to linear equations with constant coefficients
by properly chosen transformations of the independent variable,

THE CAUCHY LINEAR EQUATION. Let x =

Dy =
D2y =

Dy

eZ;

and

cix dzdx xdz

1 d 2 y dy

_(.L -
x dz 3

dz

dz

dz

x 2D2y =

dxxdz

- .a (

x 3 dz 2 dz
I
..J'..
.i2

then if J9 is defined by j9 =

+ 1
x3 dz 3
dz2
.-I..
+ 2)
and
dz

x3Dy =

'_ 1)(D -2)y.

x'D'y =

(1)(-2) ..... (-r +1)y.

After making these replacements, I) becomes


{p(-1)(-2).... (s-n +1) + p1 (1)(-2)

.. (s-n +2)
+p

a linear equation with constant coefficients.

D y

'dz =
a
d,
dzdx
ax+bdz
a 2
d2y
(ax tb)2dz2 -dz

and

and

= Q(eZ)

See Problems 1-3

THE LEGENDRE LINEAR EQUATION. Let ax + b = eZ; then


Dy =

+ ........

1 + p,1 }y

(ax +b)Dy = a

(ax+b)202y =
108

dz

109

THE CAUCHY AND LEGENDRE LINEAR EQUATIONS

(ax +b)'Dy = a'(-1)

.....

(-r

41)y.

After making these replacements, 2) becomes

............................................
+ P, , S D +
a linear equation with constant coefficients.

Q( e- b)
Sec problems 4-5.

SOLVED PROBLEMS
1. Solve (x 3 D + 3x 2 D - 2xD + 2)y 0,

e ' reduces the equation to

The transformation x

- 3& + 2)y 0

+ 3t-1) - 2 + 2}y

y C. 1 # C2' C5e7'
Since z In x, the complete solution of the given equation is y C i s + Cox In

whose solution is

2. Solve

We 2xD - 2)y

x in x 3z,
reduces the equation tois

The transformation x

2JD

2 - 2)y
The complementary function is y CL fJ
1

+ C/z2.

it

s.'

C,.t :), and a particular integral is

..

3. )

, +

a 3

a + 3

(-1)(1)
C

( 2 2 -

( -

a sin in

+ 4)y cot In
e

{(- 1) -

s/f.

2:

dx (z - 1) + 3:.

a.

reduces the equation to

(It' - 2It

4) y

The complementary function is y


y

'

The transformation x -

IS

)z

C5 . + .' ( C,co. a + C,sin a) 4 e'(x -2) + 3z/


Cx + x(C1 cos lax + C,.in Ins) + jx1 (in x -2) + 3x In x.

Thus, the solution is

3. Solve

1.,-

CoSz +

+ 4)y

cot a

+e

sin Z.

e 2 (Ccos v1z + G,sin A


z
e

z), and a particular solution

sin

It2 2It.4
1
cosz +
3-2It

z
e

1
sin:
D23

1 3 Cos

Z-25in2) +
2e

sin:.

LINEAR EQUATIONS WITH VARIABLE COEFFICIENTS


Thus, the solution is
y
e ( cos V11 4- C2 s1nW5 z) + i(3 cos z - 2 sin z) +
13

x(C5 cos t/Tlnx + C 2 sln

4. Solve

(x

2) 2

'1nx) + .

1 - (x + 2)
+ y
dx 2 dx

3x

L (305 In -

2 sin In

13

slnz

4- 1

x sin Iiix.

4.

Put x . 2 x e 2 ; then the given equation becomes


{(-1) -

+ i)y

C i e-' + C 2 z

The complementary function is y

(j9 _ 1)2
The solution is

(3

z +

+ 2) [C 1

Y (
. x

5. Solve

3eZ

2)

. 2

'.-je .

12y

C2

3 2

ln(x

{9Q-1)+ 9-36)y

e - 2

Ce

3x2 + 4x

3 2z

Oz

e -.2.

= In(x

+ 2),

2
in (x +2)) - 2.

1.

192 12x + 3)
Zz

C2

e_22+ 1

1
2

1
22
ze
108

(D 2

C -

27
2

1) or

(e

- (

C1 (3x + 2)2 + C2 (3x 4 2)

4)y

2z

(e - 1).

Dx
e )

1)

+ u-8 ((3 + 2)2 In Ox + 2) + 11

SUPPLEMENTARY PROBLEMS

Solve.
6. (X202_.3x0+4)y

x ,

7. (x2D2-2xD+ )y

lnxlnx

S. (x 5 D 5 21 2D 2 )y

(D - 1)"
or, since

+ 2)

Ce + C2e

or

reduces the equation to

9(2-4)y

The complete solution is y

and a particular integral is

Jf (dz ) 2 -

((3x + 2)21)2 + 3(3x 2)1) - 36}y

The transformation 3x + 2 e

3c2 2.

X2

mx

A p is. y C1x2 + C2 x 2

8in(1r. x)

Ans.

Ans. y

Inx

+ X

+ ! x 2 In5x

l 2 x + in
a C1 x + C 2 x 2 + ( i f

Cl+ C2 x + Coln x

x in x
in x)

+ '(COS in x + sin

9. x 3 y' + xy' - y

3x'

Ans. y

C1 x + Cx mx +

C3 .%

ln 2 x -

x4/9

10. ((x + 1)21)2 4 (x + 1)1) - iJ y lfl(x + 1)2 + x y


Ans.

II. (2x + 1) 2 y" - 2(2x + 1)y' - 12y

6x

C1 (X.+ 1) + C2 (Z + I)
Ans. y

C1 (2x + 1)

in(x 4-1)2 + (x + j.
C2 ( 2X +

1)

3x/8

Jn(x + 1)+ 2
1/16

CHAPTER 18

Linear Equations with Variable Coefficients


EQUATIONS OF THE SECOND ORDER
A LINEAR DIFFERENTIAL EQUATION of the second order has the form

R(x)X + S(x)y = Q(x).


dx 2 dx

I)

lithe coefficients R and S are constants, the equation can be solved by the methods of the preceding
chapter otherwise, no general method is known. In this chapter certain procedures are given which at
times will yield a solution.

CHANGE OF DEPENDENT VARIABLE. Under the transformation

Y = uv.
dv
- - U- +
dx - dx
I)

with

d2y

-
thc2

dx

+ R(x)

dx 2

d2v
U- +
dx2

dv du
(L

d2u

v,

+ S(x) v = Q(x)

dx

2du1

+ R(x), S, (x) =
U dc

R(x) = -

a) 11 u is a particular integral of

3)

du

V-.

becomes

2)

ii = u(x) and v = v(x),

2
d

+ R(x)
dx
&

du
dc

+ R(x) + S(x)u), Q1 (x)

+ S(x) y = 0, then S = 0 and 2) becomes

d 2 v dv
- + R 1 (x) - = Q(x).
dx
dv

The further substitution 4c& = p t

dx

2
dv

reduces 3) to

- =

+ R(x)p =

a linear equation of the first order.


b) If u is chosen so that

R1 (x) =

See Problems 1-6.


+ R() = 0 or

U = e
it

dx

I.,

= - Rx) dx then

112

LINEAR EQUATIONS WITH VARIABLE COEFFICIENTS

Now du
S5 (x)

uR(x) and d'u


- = R(x) -
dx
dx'

S(x) + 1!1

dx Ud2

dx

sothat

S(x) + j R(x) du -

--

-4R2-.

and Q1 a Q/u.
S - - 2 -& = 4, a constant, 2) becomes

If S5(x)

with constant coefficients.


If S5 (x)

+ Av a Q/u,a linear equation

dxdx

d'v + y a Qx'/u, a Cauchy equation, and the substitution

a i4/x 2 2) becomes

dx'

x a 01 will reduce it to one with constant coefficients.


See Problems 7-10.

CHANGE OF INDEPENDENT VARIABLE. Let the transformation be a a 8(x). Then


dx

dsdx

dx'dx

dx'

d:dx'

and 1) becomes
el
d 1 t dx

+ (_! +

R ! ) + Sy

dx' dxdr

or

d'z+RdX


AN),
dx

dx'
Let a a 0(x) be chosen so that

dx

(dZ)S

__

dx

C C

SYa

Nast

the sign being that which makes

dc

real and a 2 being

any positive constant. (One may consistently take a s 1.)

d'a

Rdt

If now dx' +

- A, a constant, then 5) becomes- +

()2

dx

dx
with constant coefficients.

dx

ay a 2a linear equation
dx

See Problems 11-14.

OPERATIONAL FACTORING. It may be possible to separate the left member of

CP(x)D' + R(x)D + S(x)Jy = Q(x)


into two linear operators

6)

F (D)

and F, (D) so that

{F(D).r,(D))y = Fi(D)(F,(D)y)

Then, setting

F, (D)

= V. 6) becomes F t (D)v

{P(xD' + R(x)D + S(x).y = Q(x).

= Q(x) ,a linear equation of order one.

EQUATIONS OF THE SECONb ORDER

113

The factorization in this section differs from that of Chapter 1 3.With possible exceptions, the factors
here contain the independent variable x they are not commutative, an1 the factorization differs from that
when D is treated as a variable, For example,
(xD 2 - ( x 2 +2)D + x)y = {(xD-2)(Dx))y,

since

{(xD-2)(Dx))y = (xD-2)(

x)y = (xD-2)(y'xy)

= (x-. 2)(y'xy) = x(y"yxy') - 2(y'xy)

dx

= xy

2
'(x

+ ) y +xy =

( xD (x

2+2)D+x)y.

The factors are not commutative, since


((Dx)(xD-2))y = (D x) (xy' 2y) = xy" +y' 2 y ' x 2 y' + 2xy
= xy"(x 2 +1)y'+2xy = { xD2(x2+1)D+2x)y.
Finally, when D is treated as a variable rather than an operator,
((xD - 2) (D - x) } y = {xD 2 - ( x 2 + 2) D + 2x} y.

See Problems 15-17.

IN SUMMARY, the following procedure is suggested for solving


+ R(x) dy + S(x)y = Q(x)

dc

I) Find by inspection, or otherwise, a particular integral u = u(x) of the equation when Q(x) = 0.
The substitution y = uv will yield a linear equation in which the dependent variable v does not appear.
This equation is of the first order in dv/dx = p.

2) If a particular integral cannot be found, compute S -


the transformation y

= ye

R dx

dx

. If this is a constant K or K/x2

reduces the given ;'quation to a linear equation with constant coeffi-

cients or to a Cauchy equation.


3) If the above procedure does not apply, put

is real) and substitute in dx


(clz)2

=(choosing the sign so that the square root


cbt

NF51-

If this is a constant the transformation

J ./a -

d,c

dx

yields a linear equation with constant coefficients.


4) If the left member of the equation is operationally factorable, the problem is then reuccd to that of
solving two linear equations of order one.

Note. As a partial check on the work, it is desirable to know the type of equation which results when the
transformations in I )-3 are made,


114

LINEAR EQUATIONS WITH VARIABLE COEFFICIENTS


SOLVED PROBLEMS

1. For the equation

(J32

RD+S)y = 0, show that

a) y

h) y

eX is a particular integral if 1 + R
+ S = o.

is a particular integral if R + x.S

0.

c)

is a particular integral if I -R +S = 0,

d)

y =

is a particular integral if 42 +MR+ S 0.

a) If y

x is a particular integral of (D2 + RD+S)y

0 then, since Dy 1 and D2y=0, R+Sz=0.

d) If y = e is a particular integral of (D2 + RD#S)y = 0 then, since Dy


+ mIt # .S)y 0 and mt 2 + ait + S
2. Solve (D2 -

. h) and c) are special cases (a

ay and D2

1. at x -1) ofd).

+ ) y = 2x-1.

Here R+Sx

0 and y e x is a particular integral of (D2 - x

+ --) y = 0.

2
d2v
dv
du
The transformation y =xv, Dy = * v, D2 = x + 2

reduces the given equation to
dx

2v

d +2
x

dv - dv 3
d2i
dv
3
x v = x - 2x -I
3 dx xv+
d

Putting

d2 V

or

I dv

2-

dpi
P. d
1
_ - . this becomes - - - p = 2 - - for which

liz is an integra-

ting factor. Then


P

2
f=

2Inx+dv
p

= f(zx In i-1+Kx)dx

3. Solve

z2

x2 lriz+x+C 1 x 2

(x + t_i - x(2+4x2
+x )

Here, R + s.x

- x(2 + 4x +x2
x 2 (x + 1)

dy

+ x

y C,x 5 + G2 x + x5 In + x2.

+C2 . and

2
+ (2+4x +x )y = -x

2+4x+x2

2xlnx+1+Kx.

- 2.5.

0 and y

is a particular integral of the equation

x2(x + 1)

with its right member replaced by zero.


The transformation

y xv,

dy

dv

+u

d2y

A,

dv
=
z - + 2 -

reduces the given equation

dx2

d2t,
dv
dv
+ v) + (2+4x+x2 )xv
+ 2) - x(2+4x #.x2 )(x
dx
dx2

to

x2 (x +i)(x.

or

di, - -x+2dv =
dx 2 x.tdx

= p. this becomes dP -
Z
dx
x+i
factor. Then
Putting

x+2

x + 2 for which
xlj

-x

dx
-5(1+ xli
-L ) 4"

-x

= _!__ is an Integrating
xli


EQUATIONS OF THE SECOND ORDER

e
p
x+1

Here 1 + R #S 1 - - + x

2x

C 1x e

x1x
i
= 0 and y e s
x

2x+1

1 + Cj (x+1)e

C2x + x

( x 2 + x - 1)e

(x l)y

dx

dx2

dv

and

+ C 1x +

dy
- (Zr + 1)

CU

(4)2

4. Solve

(x + 2)
e

115

a particular integral of the equation with its

right member replaced by zero.


The transformation y =
equation

to

dv

Putting

1 du

d 2 v

dp

2
x

2)

d2y

+ v)

xd2v

dv

e ( + 2 + v)

dx2

- x p z

reduces the given

dx2

+ (

2x

orationy=e
The transfm

-') e x

K.

and

for which - is an integrating factor.


x

dv
dt

x
re 4 e + kx.

2x

Cx C + C9

xe

0.

6)y

4x '?

.2
rB s+

x-2

I -

e + - +

re + c 1x 2 C

(4x -

(x +

r C

-
V -
x
C

Solve (x -

dx

1
(x+ i--)e.

xe -e dx

xdv

e (

dx

p. this becomes

Then

5.

e x v,

when

and

a m 2,

2x

is a particular integral.

x-2
dy

2x dv

dx

dx

V. =e #Ze

tj .

2x
2x dv
d2 y - 2xdV
+ 4e u reduces the
- e +4e
dx2
dx2

gien equation to
d 2 u
(x -2)( +
dx2

Putting
P

dv

dv

p.

dv
dx

dv

4 - +

4v) - (4x - 'T)( * 2v)


dx

this becomes

dx

Solve _Z - 2tanx

dxt
By

dx

-16

o. Then

y z v

2x

ie

2x

(x

- 2)2C +

C2e

2x

2 secx,

+ 3y

inspection, it is seen that

The transformation

I
- p
x -2

1 dv
d2
- - = 0.
dx2 x-2dx

(4x -6)v 0 or

-1- = C, (x-2) 2 + C2 , and

K(x-2),

6.

sin

is a particular integral of

sin r reduces the given equation to

(D2 -2 tax D 3)y = 0.

116

LINEAR EQUATIONS WITH VARIABLE COEFFICIENTS


d 2 v
sin - + 2(cox -

s in zdv
dv
2
2 sec x or d
- + 2(cot xv
- tan x)
COSX dx

dv
The substitution -p reduces this to

2(cot x - tan z )p

4 ce' 2x.

4 c8c 2x for which an integrating

factor is 4 sin 2 2x. Then


fsin2x dx cos
- 2z +jK

p sinx
Li

csc 2x K cot 2x + C 2 . and

sin x

Y- 2 dy

7. Solve

1 )y

ZdX

dz2

Here R

The transformation

S --

d 2 v
dr2

8. Solve d2 Y- 2x

dx

(x 2

021

dx

e x

2)y

2. S -

2
eSr v

= C2 cosv'x +

Here S -

e5'

x.

v
d- reduces the given equation
dx

jx 2

3
-D+
3 )y
x
2
x
2

dR

and u

3.

C2sinV'x

dR

C2sin

2x - 1.

d2v
reduces the equation to - +
dx2
Crcos/x +

3v ex

C2

whose complete solution is

jvjx *

3
2

( CIL

C1cos x C 2 sjn x +

(D 7 -

2r)

5x2

9. Solve

sin

Cx sin x

R -2x, S x2 +

+C

a linear equation with constant coefficients, whose complete solution is

The tr
transformation

Thus,

u e

dx2

Cjx cos x

V = y/e

1 and

dx

d2v

Y = C 1 cos
x + C2 siri x

Here

(cos x - secx)

+ v

2c3c2x cot 2x +

+ C

dx2

Thus,

sec x

dy
dv
y uv xv. - x - + V.d2y
-

dx

to

ze

1 +

dv
-
dx

1.p

v' x) +

2x)

(Problem 2.)

--. and
2x 2 4x2

The transformation y = uv x 3/2 vduces


re
the equation to
2
3
2x -1
d v
= -
-
or
x 2
- 3 v -
3/2
dx 2 -
dx 2 4

XIX

= -

fRdx =

XI/2

a Cauchy equation.


117

EQUATIONS OF THE SECOND ORDER


Putting

r ,

(& -

we have

The complementary function is


.1

(2e

5z/2

-e

dy

.i - 4

30. Solve

C 2 : 5

Cx

.212

y / x

The complete solution is v


and

Ct e

Cie

Cx

+x

x 1 In

-z12

5/2

2e

i.', 2

- 1,'2

-e

z/2

and a particular integral s

5.212

+ C 2 x

z/2

5/2

xc

5/2

5:/2

J2

1/2

In

dx2

dA

Here S - SR -

dx

and

ue

; Ccos

4X)

4
dx

\f:;:-

When

C41n 4 x ) +

2(x+ i)
y

fl .rr

dx/dxt e R(d2/dx)

Note. The choice of a

dx2

y Ci e

2x//

yields

2z

e 2:

C,.

- 4 d
+ y
dz 2 /!dZ
X

Then

2
dy
Cot x - - 5112 X

2x

,x
+

dx
dx

2.x

51
C. z + C,. -

Li

3 is one of convenience only. Taking a 1, -

The transformation z v

r
2(x + . )

dx

+
x

# 4e
(gX)2

92 -4 + 3

Replacing a by e, we have

12. Solve

1_4dy

or

+ C.

whose complete solution is y ' c

/ t
z/i + Cg. -

x e x as new independent variable leads to

A - 4

Y C je

(dz/d.x)2

The introduction of
dy

30

2 v x whose complete solution is

C2 s1n /2 x +

/2 x

coe /2 x
y
Then (CI

d e

d'v
-

reduces the equation to

The transformation y y e

H. Solve el - (1 +

2frdx

y C1 e

,,

C i e

1 2z//

2.

whose solution is

and A

as before.

C06 I - Cos 3X.

dx2

Here S --sin

2x

and when

dx

../

sin x

d 2 x/dx 2 + R(dz/dx)
(dz/dx)

coax (_C0tXXSI12X)

22
aifl I


LINEAR EQUATIONS WITH VARIABLE COEFFICIENTS
Thus the introduction of z
- cos x as new independent variable leads to
2
d Ycos x
y
-
whose complete solution is y
C i e 2
C2e2 +
Upon replacing z by - cos x we hive y
2y
13. Solve d+

C2e

COSX

- coax,

di =

\ hen

-COSX

_!L,

xd.x

dx 2

C j e

d2z/dx2

X4

R(dz/d

= 0. Thus the introduction of

(dz/dx)2

as new independent Nariable leads to d - '


di
C1cos z
Y
K sin z + z 2
Upon replacing z by -l/x we hae y

c 2+

( 4x - 1)dY + 4.z 2 Y
xd.

When

C5cos(-IJx) * K Sln(-1/x) 1/z

+ R(dz/dx

2 + (4x - 11x)2x

(di/d2

,,

-z

-z + 3/4
-z -/2

e
(D + j)2

C2ze

Upon replacing z by x 2 we have


15. Solve (D

a.

3
- -D + x

1) y

-1,

-z
Cie +

-z

Ci e - " +

2. Thus the introduction

(2x2

dy
as new independent variable leads to d2 y
2
+
di
dz 2

of z

liz2.

= 3re

--2

v4

--

whose complete solution is

C1 COS(j/x) + C 2 Sn(1/x)
d 2y
14. Solve

i.

C2x

2 ex

whose complete solution is

3/2 -2

4- 2

2 + x

3e-z

5-x

(Problem 2.)

The equation is equivalent to D 2y DO


y)
2

D(D -x )y 2x - 1.
3
Putting (D - -)y v, we have Dv = 2x -1 and v
x 2 - x + K.
2
Now CD - 3
- )y x2 -x+K for which .! is an integrating factor. Then
2
2

K
- - + x
)d = n x + .! +
22
3
x

S(1

b.

+ c2

2
y = Cj x + C,x5 +x(1+xlflx)

and

The equation (xD 2 - 3D + )'y 2x2 - x is equivalent to (D - )(xD - 1)y


2

222

Putting xf1 - I)y = v, we have (D - 3 )v2x 2 - z for which 1


is an Integrating factor.
Then

.!.
Z5

0 - ! ) dx2

2 mx + -+k
2

and


119

EQUATIONS OF THE SECOND ORDER


(XD-1)y

ZX 5

mx

+ x 2 + Kx 5

or

(D-1/x)y

2x 2 tnx + x

Here 1/x is an integrating factor so that


y/x

J(2x mx

and

[xD 2 + (1 -x)D - 2(1

16. Solve

Now
and

xe

e- 2x

* x

(0 - 21y

e(1

(xD+ 1 +x)V

.2

mx +x Cx

+C2

mx).

e1 6x)

-Gx) Or (D + 1

vxeX

+ 1)v

e-x( -

x - 3x 2

5(1 -6x)dx

6).

Ke/x,

f((m - 3x)e

xe

-x

21y

(x

Using the integrating factor


(0-

3) e

- 11

3)D - 11v

L/ (z + 3)

(x

we have

f((x + 3)eX + k(x 3)_2x]dx

and

+ 3)2eX

v/ (x

+ 3)2ex.

or

(D - --)v
cx +

fe dx

(x + 3)e'.

we have

Using the integrating factor


ye

(x

21y

D -

(x + 3) e X K(x + 3).

2)y

2X

2x

Nx + 3)0

v, we have [x

Cj dx C2

xe

2z

The equaton may be ritten as


(D - 2 ) y

Ke_3X/x]dx

C1e j - dx + C2e

[(x 3)D - (2x + 7)0 +

so that

x 2 (1

+ C2 x +

[xD + (1 +x))

(1-3x)e

+ x + Kx + C 2 x

is an integrating factor so that

and

Putting

is an integrating factor so that

y e_ 2X

17. Solve

Cx

v, Ny e have

(D-2)y

Here

x mx -

+ x)] y - e(t -6x).

The equation is equivalent to


Putting (D-2)y

Kx)dx

r -

18. Show that the Riccati equation

dy

4X

xe

K(-

1-2x

e_2x)

C2

4eX + C(2x +7) + C2C2X.

y P(x)

y2Q(x)

R(x), Q(x) ^

o. is reduced to a linear

dx

1 du

equation of the second order by the substitution


Since
1 d 2u

1 du

1 d 2u

t dQ di

dxQu d. iQidx >

1 du 2

1 dQ du + (
1dU)2
P du

the substitution yields


1 dQ du
d2u
_R=O or __+(P__)-..-RQu-O.

dy
19. Use the procedure outlined in Problem 18 to solve dx

The substitution y

Qdxdx

dz2

QU

Ld

_
= -

u r -
2 du

152

+ - y + -x v -

reduces the equation to

2x

120

LINEAR EQUATIONS WITH VARIABLE COEFFICIENTS


( 2 - 3z 2 /2

d 2u

1
. u . O

du

In turn, the substitutidn


solution is

C1 e

Then y

dx'

L.

xd5

o whose

reduces this equation to 6-


dz2

+ C,.

du

'


- oeC

2k - (tin x + 3 Cosx)y

do

*x'

- A.

..x'

+ k.-

K'

%,hcrc A

y' Cos t s - 2.

sec t x d&
I du
The substitution y -.
i----
dtu

1 .
) i .

+ C,.

CLe
20. Solve

1T
du
- I---- z
d2u

or

5,,2d2z2

reduces the equation to

(t&nx - 3 coo x)

du

+ 2ucostx 0.

dxt

In turn, the substitution


dx 1

3dz

Then y

2u

di
'

dx

cos x,

0 whose sot
solution is u C 1 .

or

+ C,.

sin z,

reduces this equation to

Li

un x

(CL I' + 3C, )


+2M.
coo ucx I
inx
+ A.
c1.' C o e "I

I du
Qu dc

? sinx

age 'x

SUPPLEMENTARY PROBLEMS

Solve

21. sy" - (x +2)y' + 2y 0


22. (1+x t )y"-

x 4 ,(SS
lx +2)
C 1 x + C 2 ( x 2 -1) +

Ans. y U

lxy'

2y 2

4)y - Ixy'

+ 2y a

y-

23.

(x 5 +

24.

(x +1)y"-(Zx+3)y'

25.

y" - 2 tan g y' - lOy - o

26.

x t y' - x(2x+ S)y' + ( x 1 + 3x +

27.

4x

+(

Sx
._
y

x#2)y

Cx +2x 41)e

3)y (

x )r 9

22. X 5 y" + (x - 4.x t )y' + (1- lx + 4x S )y (x - x + l)e

30.

X Y"

Zx 5 y' 4 y

Xe

Cx'e' + r,xe x + e x (x
(x 4.

Xy" - y' + (zy 0

+ ,,x
t,e (x +1)S +

y (C,e Sx+ C,.)sec z

2y" + 4.53,? + (Z 2 + 1)y 0 0

29.

C1 ( x 5 - 4) + C,x

+ C, in x)

Ix ,.
y . (t.COs In x + C,ein In

+.

y a C 1 3In z C, CO, X
(1 +x)/x

y.- C 1 coa(l/x)+ C2 s1fl(1/2)+ (1#z)/x

EQUATIONS OF THE SECOND ORDER

4x 1 y'

A115.

i/x

31.

xy'

32.

(x Siflx cosx)y" - x

33.

xy - 3y

34.

Solve Problem 21 by factoring.

3$.
36.
37.

38.

COZ y' + y CO3 z

31y x (3* +

- ( 3x


2
x y - 4xy' (6 9z 2)y0
y".2y'+4y=4

C * co$(1/32 5 )+ C2 ein(1/3x 5 )+ 2/z

y Cj x

+ C2 co x -

C2.z -

y = C(3.z +4)

sin x

2 - x in x

+5x +

x (C,C083x+ C28inx)

Zxy + 2y (L-x2)/x

121

y = Cx +

3y/x

y ( C 1 cos 2* C2 sln
y

C, (,X

2 -

2* + 1)/z

1) + C2 x x in x

CHAPTER 19

Linear Equations with Variable Coefficients


MISCELLANEOUS TYPES

IN THIS CHAPTER various types of differential equations of order higher than the first and with variable
coefficients will be considered. There is no general procedure comparable to that for linear equations
However, for the types treated here, the procedure consists in obtaining from the given equation another
of lower order. For example, if the given equation is of order three and if. by some means, an equation
of order two, which is solvable by one of the methods of the previous chapters can he obtained from it.
the given equation can be solved.
DEPENDENT VARIABLE ABSENT. If the equation is free of y, that is, is of the form
f(dY

I)

the substitution

EXAMPLE.

d2 p

X2

cL*2

2P

ely

dx -

3p 2

fl,

x)

d 2 y
dp
P. i - .......will reduce the order by one.
dx

The euuation

dr

d'11y,
dx

dx 5
0,

+ x 3

.i dy -

(,Z)2

dx

dx2dx

0, of order three, is reduced to

dy rp , d2y - dp

of order two, by the substitution

d3y d2p

If the given equation is of the form

f(dY d

flu

d'y

'
dx's dr11

2)
dky

y
the substitution - = q,
dx k
dxk =

- .
cix'

dq

x)

= 0,

will reduce the order by k.


See Problems 1-5

INDEPENDENT VARIABLE ABSENT. If the equation is free of x, that is, is of the form
f( -X. d''1y. )
dx
c' dx'1

3)

the substitution

dp

dp c'
dY = P. d 2y
dx
dx2 dy dx

d 5 y

ddp)dy

= 0,

dy

d ^(dP)2)dY =
dy dx
d'2
122

dp 4 (dP)2 etc.
,2

dy


123

MISCELLANEOUS TYPES
will reduce the order of the differential equation by one.
EXAMPLE. The substitution
yy - y"(y' ) 2

= P.

dx

p d
dx2

1, of order three, to yp 2

4 p()2

_2. p2
dy dr 5

_.E +
dy

dy

dy2

() 2
- p 5 dy E
dy

reduces the equation

1, of order two.
See Problems &-lO.

LINEAR EQUATIONS WITH KNOWN PARTICULAR INTEGRAL. If a particular integral y =u(x) of


the equation
1 + ......+ P

(PO D' + PD

4)

D + P)y = 0

is known, then the substitution y uv will transform


(PO D'1 + P D' + ......+

5)

P,.. 1 D + P, )y = Q(x)

into an equation of the same oider but with the dependent variable absent. In turn, the order of this
equation may be reduced by the procedure of the first Section of this chapter. Equation 4) is called the
reduced equation of 5).

EXAMPLE. Since y = x is a solution of ( D 2 dy -

dv

- x -

.2
dv
U u
d 2 y
+ V. - x - + 2

dx

Here, the dependent variable v

+ 1)y = 0, the substitution y

reduces ( D 2 -xU + 1)y

d.

dx2

dx2

2x

to

d7v

dv

e2

x dx

2-x 2

dx2

is

ux,

missing and the procedure of the first section above applies.


See Problems 11-14

EXACT EQU ATIONS. The differential equation

Y. x) = Q(x)

dx

dx thc''

is calIco in exact equation if it can he obtained 1w differentiatin g once an equation

(dY

d2y

= Q (x) + C

dx

dx'' 1 dx'2

of one lower order. For example. the equation


3 y2y "

+ 14 yy ' y + 4(y') + 12y'y" = 2.x

is an exact equation since it ma\ he obtained h N ditTcreniiating once the equation


3 y2y= +

4y(y')2

= x + C.

The linear equation 4 is exact pr\ ji


Pn -

+ P

+ ......+ ( - 1) P.

EXAMI'LE. Consider the equation


5, P z 15x

and

The equation is exact since


der'atic O
17

(x - 2x)y"

= 0,

idenrica)t

= (&% 2(z52z)y'
.xd

15, P

8x2-5

p - p

p' - P0 = 5-15+16-6

(5x 2 -

3)y' .

5xy = C.

P'

5)y

ISxy' + 5y = o

i, and P0
0.

X5

in which

2-and

Pd"

P3

6.

The given equation is the exact


124


LINEAR EQUATIONS WITH VARIABLE COEFFICIENTS
If equation 6) is not linear no simple test for exactness can he stated. In this case, we show
that 6) is exact by producing the equation of one lower order from which it may be obtained by a diffe rent ia t ion
116) is not exact, It may be possible to find an integrating laclor. Again, no general rule can be stated
for determining an integrating factor.
See Problems 15-21.

SOLVED PROBLEMS
DEPE.VDEN T J'A RIA BLE .4 BSE.V T.
1. Soke 2

dy - (d).)2

dx

The substitution
Integrating. j in
and y

dz

4
p reduces the equation to 2

,.

2x

2 1e
2(1 +________
_
1 -C1e 2x

Zr - 2 1n(1C1e2" ) + C2.
0.

Then In q- in x 2 . in

K,

dr2

Kx2

and y

- 2q

0.

C5 z + C 2 x + C.

.
dx 4 di5

The substitution _Z
dx3
Then q

di.
p2-4

2x
2(1 + C i e )
p

2x
1Cie

d2y
The substitution
q
reduces
the equation
to

4 or

. 2x

.!
p + 2

In

p + 2

dy
2. Solve x - - 2
dx 5 dx2

3. Solve

dx

q reduces the equat i on to q

d5y/r
t(2x+C 1 )

- L ( 2X C 1 ) 112 +
105

1 and q 2 = Zr

d2y = dy
/a
1(Zr

+C 1 ) + k. -
dx 2 dx

K2 x 2 + k 3 x

4. Solve (dY)2 dx 3 dx3


dr2

dq

or

105

C1.

-1
15

liZ

(2x.C1) C 2 x 2 Cx C.

0.

d2y
The substitution - q red tices the equation to (dq)2

dq - q = 0 or q = x d-q +
dx
dx

dx

a CIa ra Lit equation.


Then

- d2y
dx

q - - Xx + K 2,

+ 2 C2 x 2 C 2 x

dy

2 + C 2

2
Kx K

+ C3 = C 1 x 3 + 18C. 2 +

Cr 2

4C 2 x

C2 x C3.

C2 ,

and

MISCELLANEOUS TYPES

125

d_
(I+ 2x).1 4z
- (1 -2)Z U. dxtdx

S. Solve

The transformation p

(1 +2x)p U e

reduces the equation to

4xp'.- (l-2x)p

4x
P,
p +-

or

i-2x

Since I -R +8 0, we use the substitution


p I . ( v'- v),

to obtain (1+)v 20 1 or (1 +

)2

pae

V.

The substitution 1+2x

Then

KL

v)
v- 2(1 + 2x)v'

reduces the equation to [4

K2.2 -

-I) - 4)v

Legendre linear equation.

or

- 2) v

X, (I + 2z) 2 - (1 *

C.($.x + 1 + 13). -x
y 4. -x B(x 2 + 3x). + C

or

-x
-x
y Xi
+ K,(t+ x)h.X_j (14 2x) c-',

dx

.- - x
y t i
+

and

(I + 2x),

C, + (2x +

INDEPENDENT VARIABLE ABSENT.


6. Solve

y"

U (y')5

y'.

The substitution y ' p, y' u p


reduces the equation to p
dy
de
Then .- . dy, are tan p'y+K t . and
p2

yyN

or

+ i.

dx

eot(y./(1)dy dx, in in(y . X) u x+K,, sin(y+Ki)eC,.X,

Now

7. Solve

a p +p

Iand y are sin . ,.

2(y') 2 - 2y'.

The substitution y 'U p, y" p

reduces the equation to p(y

dy

- 2p + 2)

0.

Here p 0 and y C is a solution, or


-- 2

lri(p - i)

in A


p Al y' + 1, or

I I
y

d
22

dx.

l.A y

Ay U tin(Ax+B).

Then ! ire tan Ay x + K, ire tin Ay Ax B. and


8. Solve yy" - ( y' I y2 In

The substitution y'

dy1
Then

a p, y"
a

in2 y + c,
y2

dy

In

reduces the equation to


2y2pdp -

or

2p2ydy

In Z.

y
dy
y/ln1y.0

dt,

and

In( lny + /in2 y+c ) ix + ink.

126

LINEAR EQUATIONS WITH VARIABLE COEFFICIENTS


/in2 y * c =

In

Now

AeIX.

This may be written as in

y t

/1n 2 yC

C 1 e'

2 t2x
K e -2ke

K -my, and C

C2eX

or, finall y , in Y

jC

C2 e

xmy.

-x

C 2 are arbitrary constants.

9. Solve yy " + W)
The substitution y

p. y"

p dy reduces the equation to py -


dy

ing factor. The solution of py 2 dp p 2y dy

/4
Now f2p

d 2y

10. Solve -

Putting y' a p

dx

+ K.

c2

a.

2zVTi
original variables, arc tan
- 1

for which y is an integrat-

/2 slnh 1

L at

C
C1

Xv . Then

sinh(x.C),

x 0.

. we have ZP dp 2e 2 dy whose solution is p 2 a e 2) K.

dz

becomes

sinh( /x k) a sinh(x X 1 ), and y 2

given that y a y' 0 when


dy

y dy is 2p 2 y 2 y + C2.

hose solution is

Using the initial conditions. 0


2y
e

since C 1 and

tan( x)

dx

1 which, by the substitution

dx. The solution of this equation is arc tan v'''T


- I

tan x

dy

i K and K = -1. Now p

x+C

or. in the

C. Here, the initial conditions require C 0 so that

and, finally. e 2 sec2x.

It should be noted that the form of the solution of the given equation depends on the sign
of the first constant of integration. If in
p 2
K, K is positi\e and . A 2 ,
solve
.
a dz. and obtain

di

2z
A(1

A 2

24x

' - A

+ Be *241 )

1-Be

obtain

1 , / T'
in

/'

i e 2y

T'.

Since A is arbitrar - y , we may write


'

4A2Be2A*

or

(1 - Be24*)2

e Y

- A
*24*
Be
A

C. Then

+A2

and

24* 2
Be
) and
24* 2
(i- Be

2.4 Cc
- 2C

24*

LINEAR EQUATIONS WITH KNOWN PARTICULAR INTEGRAL.


II. Solve

x (sin x)y" - (3*2 sin x x cos x)y" + (6* sin x 2*2 COsx)y' - (6 sin + 2* coSz)y 0.

By inspection it is seen that y x is a particular integral.


B y means of the substitution y
is reduced to

sin z

dv

xv. y

xv' 4 t', y" a x-."+ 2.i', y" xu #

- cos xdvdv
- o.
In turn, the substitution - q

dx 5 dx2

dx2

30, the equation

MISCELLANEOUS TYPES
reduces this equation to sin x
Then

dq

- q

d2
q - - C

in s in x + In C.

in q

Thus, the solution is

y a

cos x 0

t,,1 x 1

Cj x sin x

or

127

cotxx

sinx, and

C1 sinx Cx + C,.

+ Cox.

IV
12. Solve (x' - 3x + 6x - 6)Y - z 5 y" + x 2 y" - 6xy' + 6y

By inspection it is seen that

0.

is a particular integral.

y ax

ylYIV
- xv"+ 2v', y e" a xv + 3v",
-xv
u,
IV
2
reduces the equation to (x - 3x 3 + 6x - Gov + (-x + 0 5 12x 2 + 24x - 24)t/" - 0.
The substitution y -xv, y l -

Putting_! &

x(x-3x+6x-6
Integrating,

this equation becomes

q,

dx

(- x 4 #0 3 -12x 2 +24x-24)q -0

x -4 In x In(x3 - x 2

in q

or

+(-1+-
Z

+ 6x - 6) 4. in 4 or

)dx 0.

x - 3x + 6x -6

dx5
2

Then

At

so that

Xd.x

ii
2
D ( -) - .
S
x3

x2

1C;

A(x_3x46*_S 1)
x
X4

__.1(x-3x+6X6)
D+l

1
Now D(- )

8x -6

X -

- A j'

cLir2

and D'(!) x

+ .! -

5j( 1

x 1
a As

3+.1._6
5

! + 3D() + 3D() +

0 (!)

D+1

a (D+2D+1)(!) - z-2x+2
'3

2- 2x+2

d2v

Thus, - a A -
do

dv
-

ex

(x-1)e

+Bx+C,

X5

a
N

Cj

eS
S

+C,x'+ Cox +C4,

and

In this example, it is fairly easy to see that y x. y axe.

y -

C1 . S + C1 x 3 + C,x 2

y ax',

and y

ae

C,x.

are particular integrals.

Thus the complete solution could have been written down immediately.

13. Solve (2 sin x-x sin x-z Cos x)y'+ (2x coo x- sin x- coo x)y+x(Siflx- cog Z)7'
+ (coax - sin xy - 2 sin x -z 00$ S -z sin x.
By inspection it is seen that a x,

a to

and y a

sin

x are particular integrals of the reduced equation.

We shall obtain a particular integral of the given equation using the method of variation of parameters.


128

LINEAR EQUATIONS WITH VARIABLE COEFFICIENTS


We take

Then
and we set

L1x

+ Ls sin x

y' L, + L 2 e 2 L,cosx + (Lx + L,'e 2


A)

Lx +

Lol a in x)

+ Lsin x 0.

'y" L 5 e 2 Lsinx + (L's + L. r

Now

L'1cosx)

and we set
B)

L.r + L', cos x 0.


a

Then
and we set

- L,cos x + (L,x - Liin x)

, x
L,e - Lsin x 2 sin x -x coax - x sins.

C)

Solving A). B), C) simultaneously, we obtain


L' - ith x +

and L cos x + sin x,

COI.0

cos x - sin x> and L, Ix.-x(-sins + coo x) - .sin x - ecos x,


-1 x

and L. a

-x

Thus, the complete solution is


y

Cx C5.

14. Solve (1

-(x

C1 uInx + x.inx

3x ) ) ya

Lcoix - xsInz -

coax,

(x+ 4 )y' -+ + !)y . 32(

1).

xf

By inspection it is seen that y x is a particular integral of the reduced equation. The substitution
y zv reduces the given equation to
- (x-2)i' - (x+2)v'

and, in turn, the substitution v'.


(x *

A)

is

x)is" -

3x (x 1)

reduces this to
(T - 2)ii' -

(x 4

2)u

3S(Z I

0.

Since the sum of the coefficients of the reduced equation of 4) is identically zero,
integral and we use the substitution
U
U

p
UI
to reduce A) to

.XpI 4

(x + 5)1.N + (22+

Using the substitution

'

1
4 (1 4 2 - - )
dx
x s+1

Then
U


ex

2ew' +

3x.2(x +1) 2

(x + 2x + 2)z a

+ 1)2


xSc X
3.-x (2 + 1) for which
is an integrating factor.
x+1

Ufx'cix a 5 + K,
x+1dx

z-

i-x
v . -x.
-3xc-x 3. .x
+CL._+C,,
I

and

2)u.'

ex is a particular

z, this becomes
(x 4 x)a' 4

or

2x I

s .
i

S
y xv - -- 3 x
3
2

K z+1 x
Li

dv
dx

- 35 +

x(x + i)e

Ci s ax

is

i
-z - 3x .-

+ xe

Cox.

3+

+ C,

MISCELLANEOUS TYPES

129

EXACT EQUA T1O\'S.

P. (z) y

15. Show that

4.

(x) y' + P, (x)

is exact if and only if

y " + P, (x) y' + P4 (x) y - o

plY

P. - ,' $

Let the given differential equation be obtained by differentiating

R0 (x) y" + R 5 (x) Y. + R 1 (x) y' + R,(x) y = C1.


Since this differentiation yields R,y IV
we have

P0

R0 .

P1 =

+ ( R'+

R 1 )y'

P,

+ 9 1 .

+ (R

+ R,)y"

P,

R + A 2 .

(R + R 3 )y'

R + A,. and P 4 .

P-P,+P-P+" . R -(R'+R) + (Rr+RI) - ( R'+ R I )

Now

Conversely, suppose P4 - P,' + p,"


.. 1P0 y
dz

# (

P5 -

01 )y" + ( P,

- P1'

p" +

+ P01

)y'


o.

Ry . 0.

s0.

Since

(P, - P + P,' -

P" +
+ P, Y , + P. Y , + P. Y , - C- P,' + P11I, -

- PS Y ly+ P, Y'" + P, Y" P, Y' + P4 y, the given differential equation is exact.

16. Solve

xy

(rl x + 3)y" + (4x 2)y' +

o.

The equation is exact since P -P + P'- P"

2-4+2-0 0.

xy + (22 + x + 3 ) y " + (x + 2)' + 2y.

Consider the left member

To obtain the first term we must differentiate ry". Now

( xy ")

xy'+

y" and when this is removed,

we have (22 + x + 2)y" + (4z 2)y' + 2y. To obtain the first term of the reculting relation, we must
differentiate (22 + x + 3)y'. When
(Zr + 1)y' + 2y

dx

dx

W 42 + 2)y' (j2 + x + 2)y" + ( 2

1)y' is removed, we have

(2x + l)y. Thus the given equation is the exact derivative of


A) ry" +

( 22 + x

+ 2)y' + (Zr + 1)y

C1.

( Zr + 1) - (2x + fl + 0 o, we now treat the left member of A) precisely


Since P, - P5' +
as we did the corresponding member of the original equation.

Werernovc(xy') xy"+y' and have ( z 2


dir

+r*1)y' + (2x+1)y

dz

Hence A) is the exact derivative of


B) ry' + ( x2

a linear equation for which re

x(X4)

+x+1)y -

(: 5 x + C1,

is an integrating factor.

Thus, the complete solution of the given equation is


xyc

i x(z

Csfze2tX42x + c2 fe42)dX +

The following scieme will he found convenient.

C,.


130

LINEAR EQUATIONS WITH VARIABLE COEFFICIENTS


+ (x 2 +

x + 3)7" +

xy +

z+

2)y'

2y

+ 2)7' +

2y

2)7'

7"
(z 2 . x +

(4,z

(z 2 +

2)y" +

x + 2)y" + (2x + 1)7'


(2x + 1)y' + 2y
(Zr 4 1)7' 2,

(2x 1)y
zy"

A)

+x+

2)y'

(2x + 1)y =

zy,

+ x + 1)y
B)

17. Solve

( x 2 +x+1)y' +

(2x+1)y

( x 2 ix+1)y'

(2x+ 1)7

Cx+ C2.

+ (z+x+1)y

2y ey-4
dx

We write

2y

6)

2y 2.1 2yi
dx 2 d,

6i

c&2

4dydy

2()2
dx

&dx

Thus, the given equation is exact, being obtained by differentiating

A second integration yields

2y

dy

dx

(ii

3xy 2 )y' + 9(y 1 + 2xyy')y"

We write
3xy 2 )y

(1
(1

y (y ' )2

+ K.

whose solution is

x 4 C1 z 2 + C2 x + C3.

18y(y') 2

6x (Y,)5

2 y,, +
3xy2)y 9y
3zy 2 )y' 3y 2 y"
6y 2y"

6yy'

dx

z In x + k 1 x + K2

y2 x in
18. Solve

fI

2()2

2y .i

6y2y"

i8xyy'y"

6.

18y(y')2

18y(y') 2
12y(y')
+ 6y(y')
5(I)2
12xyy'y" +

llxyy'y"

(1.3zy 2 )y" * 6y 2 y' 6xy(y') 2


(1. 3xy 2 )y"
3yy + 6xy(y')2
3y2y'
2
3y y

6x(y')
Sx(y')

The given equation is exact, being obtained by differentiating


(1 . 3xy 2 )y'

6xyy5'"

6x K

+ Sx(y')3


MISCELLANEOUS TYPES

131

and this equation is obtained by differentiating (1 + 3xy 2 )y # + 3


zy 5 + y

In turn, this equation is exact and we have

19. Solve x 5 y' + 5x 2y + (_x)y - (2+x 2 )y

+ Cx 2 + C,x + Co.

40x - 4x.

It is readily verified that this linear equation is not exact. To test whether or not it has an integrating
factor of the form x', we multiply by x to get
2

5x' 2y" + (2x" - x'5)y' - (

and write the condition


4.
- (Zr'
- 2(. + 1)x' (a + 3)x' 2

(40x5 - 4x)x'

z')y

5(a + 2) (a + j)x' - (a 3) (a + 2)(a + 1)x'

0, for all values of x.

)x 4
(a + 2)x' 2 + (a 2)(a -a. 2 )x'
2

Then a -2, and x- is an integrating factor. Using it, we have


+ 5)'" + ( -x)y' - ( I + 1)y 40x - 4.x5
zy"

ry"' +

4y" + (- - x)y'


4y'+ (2 - r)y
X

and

( 2 + 1)y
2

4y" + (- - x)y' - ( + 1)y


x

.2DX2-

xy + 4y' + 2 -

The transformation y - reduces this equation to

and the complete solution is

C1e X +

v = x2y

= CL e
20. Solve 2yy + 2(y+ 3y')y" + 2(y' )

v" -

K.

(D2 -

1)v

Cae_X - (1+D 2 D+

+ C2 e

-x

Zx -

+ Kr.

Db+ . .)(x 5 x5 + Kr)

+ CO X + x

=2.

2yy' + 2yy" + 6y'y" + 2(y'

We write

)2

+ 2y'y"

2yy'

2yy " + 4y'y"

2(y')2

2yy ' + 2(y')2 2yy" + 4y'y" + 2(y')2

and thus obtain by intcgration


2yy" + 2(y')2 + 2yy' = Zr +
2yy ri + 2(y')

2yy

2yy'
y 2

.z2L

and obtain 2yy'


y

18

2 e X

2
cX -

+ 2

z2 +

+ K

2 .

X
X
2xe + 2e + K(xe -

By inspection, eX is an integrating factor; then


X> +

K2 e

or

+ C2x


132

LINEAR EQUATIONS WITH VARIABLE COEFFICIENTS

21. Solve r cosy y"' - 3x siny y'y"_ cosy y " Since

x cosy ' - '


x2

dxx

Cosy (y') + sin y (y') 2

x cosy

y' -

sin y -0.

' . the last two terms of the given equation suggest 1

as a possible integrating factor. Using it and integrating,


coayf - sin y(y')2

+ sin y =
X

The substitution sin y

or

eosyy" - siny

+ May C1x,

reduces this equation to Z 'r + z . C,x whose complete solution is


sin y

Cx

+ C,co

C,sln

x.

SUPPLEMENTARY PROBLEMS

Solve.
0

22. y " + (y ' )2 +


23.

(1+x 2 )y" + 2xy'

24.

Zy' -7

25.
26.

"+

y"

Ans.

2x

y in cos(x-C1 )
y C1 + C1

2/x - in x

+ Co

are tan x + 1/x

y C1 x 2 + C1 + ( x + 1) In

= x 2

y C 5 e

yy" + (y') 3 . 0

+ C,x + Cs + x'(x'-.4x+12)/12

27. yy"

( y ') 2 2

x = C1 + C,y # y in y

2 + c, x + c,
y=

28. yy"

(y')(1 - y' cosy + yy' sin y)

x - C, + C,in y + slay

y C1x +C,eX+C,eSX_2
29. (2x-3)y"' (Cx-7)y" + 4xy'-4y - S
Hint: y - x is a particular integral of the reduced equation.
30.

(2x 3 -1)y" - 6x 2y" + Sxy' 0


,2

32.

z..

(x + 2y)y" + 2(y') + 2y' 2

33.

(1 +2y +3y')y'+6y' Cy" + ( y ')' + 3yy") x


3x [y2y + 6yy 'y r' + 2(y')3 ] -

35.

yy'" +

Hint: e
36.

-x .

2(y')2 +

y+y+y

3y [yy " + 2(y')2)

= e

Hint: Use y + 2y - v.

C1 x 5 +

y5

2x

s an integrating factor. Solve also using

2(y+1)y" + 2(y')

Cx 2 + C,x + C, + xr'/24

- 2/x

Ans.

is an integrating factor.

3y'y" - 2yy" -

x 2 + C 1 z + C,

y(x y)

34.

Hint:

+ C,

x
-r
Iny"C1e +C,e

." .yy (y) y my

Hint: Use in y

C L (x' + 4x) + C2x 2

C,.x + C, + x in x

.4ns. y 2
2

y =

..

+ C,eX + C,xex + e 2x

V.

2 + 2y = 0 Ans. y + 2y -C 1 cosx +C, sin x

CHAPTER 20

Applications of Linear Equations


GEOMETRICAL APPLICATIONS. In rectangular coordinates the radius of curvature R of a curve y a 1(x)
at a geieral point on it is given by

[i +

) 2]

R
dx2

7>0. y"O

7>0, y">0

X.
P (X
y<O. ycO

ycO, yM>O

Let the normal at the point be drawn toward the x- axis. It is clear from the figures that the normal

and radius of curvature at any point have the same direction when y aiicl d ty/dx ! have opposite signs
and have opposite directions when y and d'y/dx 2 have the same signs.
PHYSICAL APPLICATIONS. OSCILLATORY MOTION. Consider a ball bobbing up and down at the
end of a rubber string.
If the other end of the string is held fixed and no external force is applied to the ball to keep it moving
once it has been started, and if the mass of the string and the resistance offered by the air are such that

they may be neglected, the ball will move with simple harmonic motion
x

A cos wt + B sin t

where z is the displacement of the ball at time t from its position of rest or equilibrium.
133

134

APPLICATIONS OF LINEAR EQUATIONS


For simple harmonic motion:
a)

The amplitude or.maximum displacement from equilibrium position is

= 0,

tan

it

B/A, and x =

/42 +

/A+ B

since when thc/dt

B2.

b) The period or number of units (sec) of time for a complete oscillation is 2it/w sec, since when t is
changed by 2n/w sec the values of x and cbc/dt are unchanged, while for any change oft less than this
amount one (or both) of x and c/dt is changed.
c)

The frequency or number of oscillations (cycles) per sec is /2it cycles/sec.

d) The differential equation of simple harmonic motion is


In the above illustration

md2x
- -"W2
- (A cos wt
dt1

- - kx where k is a positive quantity.

of
dt

B sin w t) = - kx

where n, is the mass of the ball and k = mw2.


If the above assumptions are modified so that the resistance of the air cannot be neglected, the ball
will move with free damped motion

cost

B 8inw).

The motion is oscillatory as before but never repeats itself. Since the damping factor e--It decreases ast
increases, the amplitude of each oscillation is less than that of the preceding one. The frequency is
See Problem 8a.
W2a cycles/sec.
If the resistance offered to the motion is sufficiently great, other cases will arise
See Problem 8b.
If in addition to a resistance, there is an external force acting on the ball or the complete system is given
a motion, the motion of the ball is said to be forced. lithe forcing function is harmonic with period 27/X,
the motion of the ball is the result of two motions - a free damping motion which dies out as time increases
(called the transient phenomenon) and a simple harmonic motion with period that of the, forcing function
See Problem 9.
(called the steady-slate phenomenon.)

HORIZONTAL BEAMS. The problem is that of determining the deflection (bending) of a beam under given
loadings. Only beams which are, uniform in material and shape will be considered. It is convenient to
think of the beam as consisting of fibres running lengthwise. In the bent beam shown, the fibres of the
upper half are compressed and those of the lower half stretched, the two halves being separated by a
neutral surface whose fibres are neither compressed nor stretched. The fibre which originally coincided
with the horizontal axis of the beam now lies in the neutral surface along a curve (the elastic curve or curve
of deflection). We seek the equation of this curve.

-----

:z
P(x,y)

..J--.Is
^ -^ Eut10 Cur's

APPLICATIONS OF LINEAR EQUATIONS

135

Consider a cross section of the beam at a distance x from one end. Let AD be its intersection with the
neutral surface and p its intersection with the elastic curve. It is shown in Mechanics that the moment N
with respect toAB of all external forces acting on either of the two segments into which the beam is separated
by the cross section (a) is independent of the segment considered and (b) is given by
M.

EI/R

A)

the modulus of elasticity of the beam and 1' = the moment inertia of the cross section with
Here, E
respect to .48 are constants associated with the beam, and R is the radius of curvature of ihc elastic curve at?.

For convenience, think of the beam as replaced by its elastic curve and the cross section by the point P.
Take the origin at the left end of the beam with the x axis horizontal and let P have coordinates (x,y).
Since the slope dy/thc of the elastic curve at all of its points is numerically small.
11 + ()
R

Cbt

ci

5/2
=

1-

2
dy

approximately,

cbt2

dx2

and A) reduces to

B)

= N.
dx2

The bending moment N at the cross section (point P of the elastic curve) is the algebraic sum of the
moments of the external forces acting on the segment of the beam (segment of the elastic curve) about
the line AB in the cross section (about the point P of the elastic curve). We shall assume here that upward
forces give positive moments and downward forces give negative moments.
EXAMPLE. Consider a 10 metre beam resting on two vertical supports, as in the Figure below. Suppose
the beam carries a uniform load df 3000 Nm of length and a load of 30000 N at its middle.
Y

10 -x
(10-z)

o
J
30000

4
3000x

30000

30000

The external forces acting on OP are (a) an upward thrust at 0. r m from P. equal to one-half the total
load, i.e.,(30000 + 10 3000)300ooN and (b) a downward force of 3000z N thought of as concentrated
at the middle of OP and thus jz m from P. The bending moment at P is

Al

30000x - 3000x(4z) . 30000x - 1500x2.

To show that the bending moment at P is independent of the segment used, consider the forces acting on
PR : (a) an upward thrust of 30000N at Rio - x m from P. (h) the load of 30000 N acting downward at
the middle of the beam. 5 xnifrom P. and (c) 3000 (10 - x) N downward thought of as concentrated at the

136

APPLICATIONS OF LINEAR EQUATIONS


middle of

PR, 4(10-x)m from P. Then


M u 30000(10-x) - 30000(5-x) - 3000(10-x)(I0-x)
30000x - 3500x 2 ,

as before.

A beam is said to befixedat one end if it is held horizontal there by the masonry. In the example above
the beam is not horizontal at 0 and is said to be freely supported there.

L
SIMPLE ELECTRIC CIRCUITS. The suns of the voltage drops across
the elements of a closed circuit is equal to the total electromotive force
E in the circuit. The voltage drop across a resistance P ohms is RI,
across a coil of inductance L henries is t di/dt, and across a conS
denser of. capacitance (capacity) C farads is q/C. Here, the current I
amperes and the charge q coulombs arc related by I dq/dt. We
will considcrR L,and C as constants.
The differential equation of an electric circuit containing an inductance
of capacitance C, and an electromotive force(t)is therefore

co)

RI + &

L, a resistance R, a condenser

(t)

de

or, since

E FAAA T

ldq/dt, dl/dtu'dq/dtt,

C)

.2 + R 2 + 2 - E(t)
dt

dt'

from which q -

q(t)

By differentiating

may beround.

C') and using 9


dt

D)

I, we have

+ R di

+A

dt'

from which

I = 1(t) maybe round.

SOLVED PROBLEMS

GEOMETRIC APPLICA TIONS.


1 ! Determine the curve whose radius of curvature at any point
in the same direction, (b) in the opposite direction.
a) Here

(i

34

+u

-y(1 '

The equation is exact and an integration yields

or

P(x,y) is equal to the normal at P and (a)


yy' 4

yy r + z - C 1 0

+ 1 0.

or

ydy + (x- C1 )dx 0.

I.

APPLICATIONS OF LINEAR EQUATIONS


(x _C 1) !

Integrating again y2
X.

k or Y2 +

(x -

C 1 )2

137

C,, a family of circles with centres on the

axis.
5A

_________________
b) Here
(i +
y
(l

1/2

or

yy " -

- 1 o.

The substitution y# p, y" p E of Chapter 19 reduces the equation to


dy

Then

ln(1Ip 2 )

Integrating,

In

ix

dy

1+p2 Y

+ in

cosh- 1 C 1 y -

p4

or

yp LP p 2 -1 0
dy

1+p 2 Cy',
+

C,,

C1y

dy

or

vy2_i

cosh(C1x + C2),

or

y
The curves are catcriaries and the equation may be written in the form
Y

_(8tx)/4 ).
J A(e*)'4 +
where

and

Cl

PHYSICAL APPLICATIONS

MOTION OF A PENDULUM.
P

2. A pendulum, of lengths and mass s, suspended at ? (see


figure) moves in a vertical plane through P, Disregarding all
forces except that of gravity, find its motion.
Under the assumptions, the centre of gravity C of the bob
moves on a circle with centre p and radius 1, Let 6,positive when
measured counterclockwise, be the angle which the string
makes with the vertical at time t. The only force is gravity,
positive when measured downward, and its component along
the tangent to the path of the bob is sg 8th B. If $ denotes the
1* 6 and the acceleration along
length of arc CO C, then $
the arc is
2
2

dt'
Thus

.i

dt2

2
or
I
d2 2 dt2
_

sin

1
-g sin 0.


dO
d0d8
___________
l() = 2g cos 8 4 C1 or
Multiplying by 2 and integrating,I(Wt)
dt

/2gcos0+C1

dt

-.

PIT

This integral cannot be expressed in terms of elementary functions.

When 8 is small, sin 8 9, approximately. When this replacement is made in the original differential
t.
equation, we have d 0 + 0 0 whose solution is 8 C 1 cosJI t + C,sin

Al

dt2
This is an example of simple harmonic motion. The amplitude is

+ C

and the period is


APPLICATIONS OF LINEAR EQUATIONS

138

MOTION ALONG A STRAIGHT LINE.


3. A mass a is projected vertically upward irom 0 with initial velocity v0.
Find the maximum height reached, assuming that the resistance of
the air is proportional to the velocity.
Take upward direction from 0 as positive, and let x denote the
distance of the mass from Oat time t. The mass is acted upon by two
forces, the gravitational force of magnitude mg and the resistance of
magnitude Xv K

dt

Hence, using

each directed down.


mass \ acceleration = net force,
d 2x

d 2x
- = - - K -

Integrating, 1)

or -+
C + C2 e

kt - g

v - =.-kC2 e

2)

dz

u = v0 .

where X

Ak.

t, and then differentiating once with respect to

-kt

dt

When t 0, x - 0 and

-g,

Then C 1 + C2

Making these replacements in 1), we have

0, v0 = - AC 2 - .

+ kv0)(1 -

it 2

The maximum height is reached when


Then the maximum height is

t,

v 0.

From

2),

g
+ kvo

-( g + kv 0 ) ( 1 -

c_ It )

and C
-

-g

(1 fl6 kVO)

1.

t.

C2 g+kvo
k2

A A

- C2

and t
A

!(v0 In
A
A

in g + AVO
g
g

4. A mass m, free to move along the x- axis, is attracted toward the origin with a force proportional to its
distance from the drigin. Find the motion (a) if it starts from rest at x xo and (h) if it Starts at x x0
with initial velocity v0 moving away from the origin.
Let

denote the distance from the origin to the mass at time t.


2

Then

- Xx or
dt 2 dt7

Integrating, 1)

x = Cl. sin At

+ k2

= 0, where K

C 2 cos At,

and differentiating once with respect to

2) v = - AC 2 sink t + AC cos At.

a)

When t 0,

x zo

and v 0. Then C 1
X z

b)

When t 0,

x xo

and

v = v0 .
X

0 from

x0 . from I). and

x0 Cos At.

Then C 2
110
-

2). C 2

Sin At

x. C = v0

/k, and

+ x0 Cos At.

In a) the motion is simple harmonic motion of amplitude x 0 and period


In h) the motion is simple harmonic of amplitude

22
y vo+ Ax0

2't/k.

and period 2n/k.

t,


APPLICATIONS OF LINEAR EQUATIONS

139

.'iIOTION OF COMPLEX SYSTEi!.

5. A chain hangs over a smooth peg, 8 metres being oil side and 12 metres on the other.
Find the time required for it to slide off (a) neglecting friction and (b) if the friction is
equal to the weight of 1 metre of the chain.
a) Denote the total mass of the chain by a and the length (metres) of the chain which
has moved over the peg at time t by x. At time t there are (8 - z) metres of chain on one
side and (12 + x) metres on the other. The excess (4 + 2x)nietres on one side produces an
unbalanced force of (4 + 2x)
a

(4 2x)

dt2

newtons. Thus,

or 10

20

= gx

4-

dt2

2g.

Solution 1

x
- -
2
10

Integrating

we have x

Differentiating once with respect to t,

'71-0 t

I (Ce

G2e

- C2e

/7Th t

- 2.

i7Th t>

.vc

When ro. x=oand v=0. Then C, C2 =1 and x


Hence

t =

cosh

in x + 2 +

(x 42) =


When x

FL8O

rn has moved over the peg.

+e

-i

-2

2 cosh)

T t -2.

/ 2 + 4x
2

ln(5 + 2V ) see.

dx
Solution 2. Multiplying th e equation byand integrating,
eg
we have
dt
to

dx + 2g

dtdt2

5(dZ)2

and

gx 2 + 2gx + Ct.

When t = 0. x z 0 and dx/d = 0. Then Cs, = 0 and


5(dX)2

gx2 + 2gz

dt

or

dx
10
/g /x2+tx

dt

The positive square root is used here since x increases with t.


Integrating, t

When i

P!
g

x = 0. Then C2

b Here . ddt2

in(x + 2+?

= -

4x ) + C2.

x+2+V 10
as before.
in 2. and
=
- in
V g 2

+ 2x)
-
20
20

or 20

(2x + 3)g.

dt2

Multiplying b y and integrating, we have


dt

( )
dt

2
gx

s3gx+Ct.

When t=o. z=O and v=O.Then C1 =o anddx - = I


1(x243x) or dt
dt
10
liicn t

L ln(x +

+ ITT +3x )

C2.

rw

dx


140

APPLICATIONS OF LINEAR EQUATIONS


When

t o x

= 0. Then C2 -

ri

When x8, t

9
V-

in

.J ! in

19 + 4V

and t
in
F70

1.4 sec.

6. A bead slides without friction along a straight rod


of negligible mass as the rod rotates with constant
angular velocity about its midpoint 0. Determine the
motion (a) if the bead is initially at rest at 0 and (h)
if the bead is initially at 0 moving with velocity g/2w.
Let the bead he x units from 0 at time t. It is being
acted upon by two forces, (I) gravity and (ii) the centN
ifugal force &J 2 x acting along the rod and directed
away from 0. Since the rod has rotated through an
angle Ut, the component of the gravitational force
along the rod has magnitude sg sin wt: its direction is toward 0.
Hence
d 2x

At -

r1 x -

sin wt

or

d2x

- - U2 x

- g sin ijt.

cit 2 dt2

Integrating,

I)

x -

2)
a)

When t o, x

wC1ewt - iC2e-

arid v

= 0

Then C + C2 = 0 from I
g

wt

4U

b)

C1 - C2

0,

arid

sin wt. Differentiating once with respect to t.

2U

Cos wt.

0,

- e ) +

When t =0, x'O


Then C1 C2

C t e t + C 3 e - t +

+ .1..

0 from 2). C1 -

2 s1n (> t

2l.i2

sinhUt

and

C2

in wt.

v=g/2w.

C1 C2 0,

C 'C2 0,

and

sin wt.

SPRINGS.
7. A spring, for whichk = 7ooN hangs in a vertical position with its upper
end fixed. A mass of 7 kg is attached to the lower end. After coming to rest,
the mass is pulled down 0-05 in and released. Discuss the resulting motion
of the mass, neglecting air resistance.
Take the origin at the centre of gravity of the mass after coming to rest,
and let x, positive when measured downward, be the change in position of
the mass at time t. When the mass is at rest the spring force is equal to but
opposite in direction to the gravitational force. The net force at time t is the
spring force -Ax corresponding to the change x in the position of the mass.
Then

cit2

= -700x Or d- + IOOx

dt2

141

APPLICATIONS OF LINEAR EQUATIONS


Integrating,

C1 sth 101

C2

cos 101.
10 C 1 cos lo - C 2 sin lOt 1.

Differentiating once with respect to t


When

0,05 and u =0. Then C 2

0,

0.05, C 1 0 and x

This represents a simple harmonic motion. The period is -_


10
cycles/sec, and the amplitude is 005 m.

0,628sec the frequency is -2_


21t

(a) o! 4

8. Solve Problem 7 if the medium offers a resistance (N) equal to

0.05 cos lOt

and

(h 980 v

1.59

, where v is expressed

in ms-1
d2x
'1-
dt2

( i) Here

and x

1 dx
- 700x - - -
dt

* - - 0 4 100)x
28
-,3179 I

1 dx
- -
28 dt

d2

or

-
dt2

100 x

Using the D notation,

[0 -(-0,01'19 + 101)) [1) - (-0,01'79 - loiflx

0,

(Cl cos lOt+ C 2 sin lOt).

Differentiating once with respect to t,


dx

Q,0179 ((10C 2_ o.o179Ccos lot -

When
Thus,

o,

vrO

and x = 0,05. C 1 0,05,

-0.01791

X r

(10

C1

0,0179C 2 )sin lOt).

0 10 C 2

and

0.0179 C 1

C2

0, 000895.

(0,O5COS 101 + 0,000895 sin lot ).

This represents a damped oscillatory motion. Note that the frequency 10/ 21t 7 1,59 cycles/sec remains
constant throughout the motion, while the amplitude of each oscillation is smaller than the preceding one
due to the damping factor
when

-0,01191

0.

At t = o the magnitude of the damping factor is I. It will be 2/3


= 22,1 sec. It will be 1/3 when e

2/3 or after

b)Here

e-

- 700x - 980
dr'2

-oi

Integrating,

dx

C2e-

x = C, e -

or (02

140 0

4.

-0 01791

100)x

= 1/3 or after t 61.4sec

i.3

Differentiating once with respect to t,


- 0,7C j

When

t0,

'71
-19 3t
- 139,3C2e

C --o

x 0.05 and v= 0. Then C 1 + C 2 = 0.05, -0,7C 1 - 139,3C 2 = 0, C 1 0,0503,

C 2 = - 0.0003

and
x

0,0503

C0. 7

0.0003 e - 13O,3

The motion is not vibratory. After the initial displacement, the mass moves slowly toward the position
of equilibrium as t increases.

142

APPLICATIONS OF LINEAR EQUATIONS

9. Solve Problem 8a if, in addition, the support of the spring is given a


motion y 2 0,3 cos 4tmetres.
Take the origin as in Problem 8 and let x represent the change in
position of the mass after t sec. From the figure. it is seen that the stretch
in the spring is (x - y) and the spring force is - 700(1 -y) -700 (x - 0.3
Cos 4t)N. Hence,
d 2x
7 -
d12

ld
-700(x -0.3 cos 41) - -

4 dr

or 0 2 - T8 D 4 100 )x = 30 cos 4t.


integratin

-QC1't

(C

30

cosloe + C 2 sin lOt)

D D/28.100
e

cos 41

-o.ciqt

(C1cos lOt + Csin 101) + 0.0006 sin 41 + 0.36cos 4t.


Differentiating once with respect to t.
V r

0.O179t

((icc2 -

0,0179 C)cos lot - (10C + O,O19C )sjn lOt)

+ 0.0024 COS 4t - 1,44 sin 41.


When j o,
z

t,

o and x

0,30.05 0,35. Then C

-0.01, C. -0.00026 and

cos lot - O-OCO215, sin 101)

0 0006sin 4t 0. 36 Cos It.

The motion consists of damped harmonic motion which gradually dies awa y (transient phenomenon)
and a harmonic motion which remains(steadystate - phenomcnon)Aftcr a time the onl y effective motion
is that of the steady-state. These steady-state oscillations will have a period and it frequenc y equal to
those of the forcing function y 0.3 cos 41. namely a period of 2T1/4
1.57 sec and it
of
= 0.63 71 cycles/sec.
The amplitude is

1w. O0OG

(0.36)2

0.36 m.

10. A mass of 10 kg is suspended from a spring which is thereby strelchcd 81 mm. The upper end of the spring
is then given a motion y r sin 21 COS 21 m. Find the equation of the motion.neglectingair resistance.
Take the origin at the centre of gravity of the mass when at rest. Let x represent the change in position
of the mass at time (.The change in the length of the spring is (x -y), the spring constant is 10(9,8)/0,081
1210 Nrn-and the net spring force is -1210(x -y). Then
2

d2

10 f = - 1210(x - sin 21 - cos 21)

di2

Integrating, x

or

os lit + C2 sin lit +

( sin 21

Differentiating once with respect to t,


- 11C sin lit
When 9 =0. x x land
Then
Hence.

l C
x

l2i

dt2

11C

121 (sin 2t +

COS

COS 21),

cos 111 4

(- sIn 21 + COS 21).

0.

C1 -0.034 and

ii c2

- 0.034 cos 11 1 -0, 188 sin lit

* 242

2t).

0,

C2 -0.188.

1.034 (sin 21 cos 21)


APPLICATIONS OF LINEAR EQUATIONS

143

II. A mass of 30 kg is attached to a spring for which k


750N m and brought to rest. Find the position
of the mass at time t if a force equal to 20 sin 2r is applied to it.
Take the origin at the centre of gravity of the mass when at rest. The equation of motion is then
d 2 x d2x
Or
30 - * 750x
sin 2i
dt 2 dt2

Integrating,

Ci

.-

Differentiating once with respect to


Using the initial conditions

x 0,

25x

sin 21

t, . - -. 5C 1 sin 5(

when

t o.

- sin 21.

5C2 cos St .

C1

cos 2t.

and

0. C
-

0,013 sin 51 0.032 sin 21.

The displacement here is the algebraic sum of two harmonic displacements of different periods.
12. A mass o19 kg is attached to a spring for which k r 729 N
and brought to rest. Find the motion of the
mass if the support of the spring is given a motion y r 0,3 sin gtm
Take the origin at the centre of gravity of the mass when at rest and let
of the mass at time i.
The stretch in the spring
9

S(x

-y)and the spring force is -

represent the change in position

729 (x - y).

Thus,

d2

d2
f - '729(z - 0.3 sin 9l) Or
-
d1 2 dt2

Integrating,

and

9C sin 9t

Using the initial conditions


X

x 0,

24'3sin9t.

C2 sin 9t - 1.35i cos 9t,

C1 cos 91

Six

9C2 cos 9i - 1.35 cos 9t


0

when : 0,

C 1 r o,

C 2

12.15t sin 9t
0,15

and

0.15 sin 91 - 1,35 cos 9t.

The first term represents a simple harmonic motion while the second represents a vibratory motion with
increasing amplitude (because of the factor m). As t increases, the amplitude of the oscillation increases
until there is a mechanical breakdown.
13. A cylindrical buoy 05 tn in diameter stands in water (density 1000 kg m3)
with its axis vertical. When depressed slightly and released, it is Found
that the period of vibration is 2 seconds. Find the mass of the cylinder.
Take the origin at the intersection of the axis of the cylinder and the
surface of the water when the buoy is in equilibrium, and take the downward
direction as positive.
Let x (m) denote the change in the position of the buoy at time i, By
Archimedes' Principle, a body partly or totally submerged in a fluid is
buoyed up by a force equal to the weight of the fluid it displaces. Thus the
corresponding change in the buoying force is 1000(0,25 r. g z and
II d 2x
dt2

-
- 62,i y

or

d2z

dt2

where W (kg) is the mass of the buoy and

9 9,8

62,5t x
W

=0

144

APPLICATIONS OF LINEAR EQUATIONS

Integrating,

Since the period is

______

C1 sin

2-
/62,5

* C2cos

62,5

2,

/_62,5 t
--*7

195 kg

HANGING CABLE.
14, Determine the shape of a uniform cable which hangs under its own
weight,. v N A- 1 Of length.

IN

Choose the coordinate axes as in the figure, the origin being at the
lowest point of the cable. Consider the part between 0 and a variable
point P(x.y). This part is in equilibrium under the action of (I) a
horizontal force of magnitude H at 0. (2) the tension T along the
tangent at P. and (3) the weight W of OP.
Since OP is in equilibrium, all force acting horizontally toward
the right and all force acting horizontally toward the left must be
equal in magnitude, and, also, all force acting vertically upward and
all force acting vertically downward.

Hence,

con 8 H.

T sin8

W,

and


Hdx

To solve the above equation, write

!/jT'

Integrating

dy

x
H

dp

o, p o and
and

x x, p P.

ci)'

= - 8Inhz.

dx

sinh z dx between the limits x

w
H
; (C08h

x -

where sis the length of OP. Thus,

and obtain

/i7

Integrating between the limits x

vs.

or

dx H

sirt1pP

,Hdx

dx

N'

ci)' N'
cLx
H

tan 8

Now llis constant, being due to the part OQof the cable, while

dx 2

1),

o,

o and x x, y y.

a catenary.

If the origin had been taken at a distance H/v under the lowest point of the cable (thus making H/v
the y- intercept of the curve) the equation of the curve would have been
H

y cosh-.x.

APPLICATIONS OF LINEAR EQUATIONS

145

HORIZONTAL BEAMS.
15. A horizontal beam of length 21 metres is freely supported at both ends. Find the equation of its elastic
curve and its maximum deflection when the load is w N.iof length.

y.
iii

iii

WX

Take the origin at the left end of the beam with the x- axis horizontal as in the figure. Let P, any point
on the elastic curve, have coordinates (x,y).
Consider the segment OP of the beam. There is an upward thrust w N at o, m from P. and the Load wx N
at the midpoint of OP. ix m from P.Then, since El d 2y/d.x 2
Al,
I)

= vii - wx(x) = wix - wx

El

Solution 1. Integrating 1) once.

At the middle of the beam

2)

El

= I and
-

dx

Integrating 2), Ely

1
-wix 2
2

dx

dy/d.z

1
-w
6

o.

+ C.

Then C1

U.
= -

1I

and

= 0.

Then

2
5
-viz
- -vi - -vi,

- viz - -1 vi - - 1 x + C 2 . At 0, x y
6
3
24

3)

and

(41x - x - 815x),

24E1

Solution 2. Integrating I) twice,

Ely

viz 3 - .!

vx + Cx

C2.

At 0. x m y m o,while at R, x 21, y = o.Using these boundary conditions in turn, we find C2 = o and


C1 -

w1 5 ,

as before.

The deflection of the beam at any distance x from 0 is given by -y. The maximum deflection occurs at
the middle of the beam (x 1) and is, from 3),
W4
.
5'Z
24 El (41 -1 -81) = 24 El

16. Solve Problem 15 if there is in addition a load of w N at the middle of the beam.
yT

ix-

0Hfix

w
0<1<1

w1+W

P
vx
1W

x
(X. Y)


146

APPLICATIONS OF LINEAR EQUATIONS


Choose the coordinate system as in Problem 15, Since the forces acting on a segment OP of the beam
differ according as P lies to the left or right of the midpoint, two cases must be considered.
When 0< x < 1, the forces acting on OPare an upward thrust of cZ + i *) N at 0. x rn from P, and
the load wx acting downward at the midpoint of. Jxm from P.The bending moment is then
1)

wx(z)

At =

NIX +

JWX

When I <x < 21 there is an additional force - the load W N at the midpoint of the beam, (x - I) rn from
P. The bending moment is then
2)

(wI + 4W)x -

At

lLz) -

- W(x - 1).

W(x -1) = wlx + j Wx -

Both I) and 2) yield the bending moment At = wZ+ J NI when x v I. The two cases may be treated at
the same time by noting that for 1)
-
wix + Wx
wx2
- 4W(I-x) + WI
and for 2) wLx+ j Wx j wx 2 - W(x-1)
-

3) El

wx2

Then

wix - wx 2 + kW(Ix) + jW1.

0(1 -x) + i W1

with the understanding that the upper sign holds for 0< x < I and the lower for 1< x < 21.
-1 W(L x) 3
12

- wZx - _!. wx'


6
24

Integrating 3) twice, Ely

+ ! WIx2

CIL

C2.

Using the boundary conditions xy.0at 0 and x 21, y=0 at R.


C,.-1 Wl,
12
Ely

21C1+C2_

NJ*

+!wl'+!W1 3 _W1 5 , and C=_i 5 _!Wi 2 .

1
1
3 - -
- - w x
wx
6
24

1
- 1
- - wl 5 x +
3
12

1
1
1
- wix5 - - w'x - - wi 3 x
6
24
3
y =

and

2p (4Ix5 -
24 El

12

I-x)3 + 1
4

12 El

W11
1 2
- WI x + 1
2
12

, 2
1 2x + .! WIS,
+ - 1 Wix - - W1
2
12
4
I ,_ X13 - 61
2x + Is).
(31x2 -

1 W 1
12

- 81 5 x)

*.2

The maximum deflection, occurring at the .iddle of the beam, is

5.1,

yIax

Wi5

-7 +

17. A horizontal beam of length 1 metres is fixed at one


end but otherwise unsupported. Find the equation
of its elastic curve and the maximum deflection
when the uniform load is wM -' of length.

Take the origin at the fixed end and let P have


coordinates (x,y). Consider the segment PR. The
only force is the weight w(I-x) N at the midpoint
Of PR (I-x)m from P. Then
E1E_2L

-w(I-x).(I-.x)

=.-kw(I-x)2.

Li '2

At 0 x0

= o; C,, . - wz
dx
6

Integrating again,

Integratingonce, El

and El -
dx

1
w 6

x -

Ely = - -I w(ix) ' - 1 wI 5 x + C2.


24
6

1 5
R

!w(I_x) + Ci.

Then


147

APPLICATIONS OF LINEAR EQUATIONS


At

0: zy . O;

then C2

24

w1 9,

Ely


x+1
24

l - x) -

and

Id
3
- X").
(41x611.1
24 El

The maximum deflection, occurring at

R(x 1).

is - y xax

8 El

. Note that this is not a relative

minimum as in Problem 16 but an absolute minimum occuring at an end o( the interval

0 x 1.

18. A horizontal beam of lenght 31 metres is fixed at one end but othervdlse unsupported. There is a uniform
load 01 N n of length and two loads of W N each at points I and 21 ru from the fixed end. Find the
equation of the elastic curve and the maximum deflection.

L(3L-z)w

Take the origin at the fixed end and let P have coordinates (x.y). There are three cases to be considered
according as P is on the interval (0 <x 1), (1< x < 21), or(21 < x < 31). In each case, use will be made of the
right hand segment of the beam in tomputing the three bending moments.
When o < x < 1. (P P 1 in the figure), there are three forces acting on PR: the weight (31 - x)w N taken
at the midpoint of PR, j (31 -x) m from P1 ;the load w N.(1 -x) m from P1 ; and the load W N, (21 -x)
in from P1 . The bending moment about P1 is

U1

( 31-x) 2 - W(I - x) - W(21-x).

El

Integrating,

dy/dx 0;

dy
El
dx

and
Ely

= -

v(31

-x) 2

! w(31-x) 5 +

El

At 0: x = 0 and

then C1 -

1
- 5
-i"(31 x) +
6

W(1

W(L - x) - W(21-x).

-x)2

W(21 -x) 2 + C1.

WI2,
2

1 W(I-x) 2 + 1 W(21-x)
2
2

- -
2


- 1
(31-x) - ! W(I-x) 3 - W(21-x) - -w i x
2
6
24
6
2'?

Al 0: xy=o; then C2 = 8
91)

-(31-x).4(31-x) - W(1-x) - W(21-x)

and

WI5

and

12

W
2

+ C2.

148

APPLICATIONS OF LINEAR EQUATIONS


.4)

Ely - -

w01 -x) ' -

W(i -x) 5 - .!
6

W(2i -z)

wlx--Wjx

27

wl
8

3
2

-WI

When I <x < 21, (P -P2 in the figure), the bending moment about
P7 is
M2 x -

and

Jw(3l-x) 2 -

El -Z . d2
B')

Ely

w(31 -x) 2 -

(3Z -x)" -

W(21 -x).

! W(21-. ) '

Integrating twice, we obtain

C2 x

When x 1, 8') and .4) must agree in deflection and slope


8)

Ely

When

21< x < 31. ( P

24

w(31-x)' -

W(2i-) 5 -

ix -. 2

W1 2,

27
+ w l + 3 W1 1.

. P in the figure), the bending moment about P, is


M.
- w(1_X)1.

El

- w(31- )2

Then

4i2
Ely

so that C3 C and C4 i C. Thus,

and

C)

C4.

1
- .! w(31_x)C + C,z + C, 9
2
1W11.
-.11._
1
wj 4
24
24
2

since, when x 21, there must be agreement with B)


in deflection and slope.
4), B), C) may be written in"the form
2

24 El

W
(121 x - 541 x 1
- x ) + x2
(2x - 9

. 2 C
(12Zx 5 - 5412
,j-1x )+ (x

6 El

- 7 (12lx 3 - 541 2x 2 - x) + __!_


24 EE1

The maximum deflection, occurring at R

(3 1'

- 6 1X 2

- 31 2x

- 51 2 x).

I.

I s ),

21

21,

31.

is - y ,,4 (Blwl' + 48 W
j
Note that the elastic curve consists of arcs of three distinct curves, the slopes of each pair of arcs at a
junction point being equal.
(x -31),

19. A horizontal beam of length 1 mis fixed at both ends. Find the equation ofthe elastic curve and the maximum deflection if it carries a uniform load of Nn- 1 of length.

!wI

WX

Take the origin at the left end of the beam and let P have Coordinates (z. y).
The external forces acting on the segment' arc: a couple of unknown moment K exerted by the wall
to keep the beam horizontal at 0 ; an upward thrust of fwlN at 0. x
m from P; and the loadx N acting
downward at the midpoint of OP. ix m from P. Thus,

APPLICATIONS OF LINEAR EQUATIONS


El .1

K + w1x -

wx2

Integrating once and using x x 0, dy/dz 0 at 0.


At
*

,cz +

R:

1W I 5

1. dy/'dx 0

wL

El

1
1
2
- -VX3.
Xx + - wix

th

since the beam is fixed there. Then


and

K = -

0.

Integrating and using x .y o at


1
Ely - -

149

El

_I 1012X + ! W IX2

ipx3.

0,

22
1
1
3
x + - wix

74wx

and

wx

Lx
-

The maximum deflection, occurring at the middle of the beam (x= 41), is
20. Solve Problem 19 when in addition there is a weight

'Ysax

3:4;l

N at the middle of the beam.

r"1
4(w +W)

4(w +W)

Using the coordinate system of Problem 19, there are two cases to be considered: from x 0 to x 41
and from x . 41 to l.
When 0 <x <41, the external forces on the segment to the left of P (x,y) arc: a couple of unknown
moment K at 0: an upward thrust ffr( w i+ w) N at 0, x m from PI ; and the load wx N, 4x from P.Thus,
El fl

K + (wL+Wx

La

Integrating once and using x 0, dy/d.x - 0. at


A')

LI

wZx

12

!WX.

0,

- Xx + . Ix - ! wx

Integrating and using x . y 0 at


A)

+ !

wx2

0,

1
2
1
1
5
5
EIy-Xx .wlx --wx
+ Wz.
12
24
12

When 41 <x < 1, there is in addition the weight


y K +
LI L

W Nat

1 wlx - l2

the middle of the beam, (x - 41) ni from P,. Thus,

+ !Wx - W(x - 41).

Integrating twice,

1
+ Lx
12

Ely !
2

B')

When
and

I,

.! w1x 5 - ! rx

Ely - . Kr 2

To determine K, use x
+ 1

1 v1 3

18

24

48

12

24

'

12

x - -

and

16

24

+ .! N'x 5 . W(x -

i l, dy/dz a 0 in A'). Then

12

24

'(2L x 12x -

24E1

x)+

2 2
1
a - -1 -viz
+ yr 3
24

1
24

1 5 1
Nix
12
18

-' -(4x

48E1

31x 2 ),

1
WX
12

0 z

- ! Wix' and

W(r

18

22

- ix -x> + -6
(1
i2x+9Zx2-4x),
48E1

The maximum deflection, occurring at the middle of the beam, is y 13


X
21.

and

x +

12

a ( 21x'
24 El

yr

Substituting in A) and B),

wi.

12

1 ,2 2
1
I
Ely - w x + viz

Ely

+ Cj x + c.

the values of anddy/dz for B') must agree respectively with those for A). Thus, C1 C, 0

B)

1
2

+ - W.1
W(x
12
6


1yr

*Zz<'
a

( vi + 2W1').
38411

A horizontal beam of length 1 m is fixed at one end and freely supported at the other end. (a) Find the
equation of the elastic curve if the beam carries a uniform load vN.' of length and a weight IV N at the
middle, (b) Locate the point of maximum deflection when 1 3 and N' a

Take the origin at the fixed end and let P have coordinates ( z , y ).There are two cases to be considered.
When 0cx< P. the external forces acting on the segment P1 R are: an unknown upward thrust N
atR, ( i - x )m from Pj;thc load .(Z_x)Nat the midpoint of p1 p , ( L-z)m from P 1 :andwN,
(if ..x)m
from P1 . Thus,
El

S(Z -

x) v(I-x)'(lz)

Integrating once and using s


El

a o

.!S(1_x)'
2

dy/dr

W(1-x) S(l -

x) 4(I-z) 2

0 at 0.

v(i_r)

W(1-x)2

i Sill 1 5
vl

WI'.


APPLICATIONS OF LINEAR EQUATIONS
Integrating again and using x y a at 0,
A)

Ely

24

Wi-x 5 + ( !S1 2 -
2
6

W12)x -

151

!w,

24

48

When I <x < 1, the forces acting on PB are the unknown upward thrust Sat R. (1 -m from P, and
the load w(L-x) N ,
L_x m from P,. Thus,
1
2
5(1 -x) - -w(1-x)
2

Eldy

B')

Ely

S(1_x) -

w(L-x)"

and

C1 x + C,.

as given by A) and B') must agree. Hence, C and C, in B') have


dx
the values of the corresponding constants of integration found in determining A), and B') becomes
Z, the values of Ely and El

When x

B) El

S(Z-z)'

1 ,2
1
L-x) + (- - wL 3 -
'4

24 (

1
WZ)z -


Sl + 1 wL' + -Lwz'.
48

To determine S, use x 1, y oat Bin B);thenS . 2 wl +


W. Making this replacement in A) and B).
a
is
y

22

48 El

(5 Ix5 - 31 x - 2x '4 ) +

y - (5Ix5 -

22

x - 2x)

96E1

_!_-(2
96E1

- 01x 2 ),

1 z I i I.

- 121 2x + 151x' - 5x),

and
L

x 1 1.

It is dear that the maximum deflection occurs to the right of the midpoint of the beam. When 1 3.
the equation immediately above becomes
5 98* - 2540x + 210).
(-2x -101 +
y

W . lOw,

46

Since

dx

El

0 at the point of maximum deflection, we solve


8x

+ 30*2

- 396x 540

for the real root x 1 . 61,approximately. Thus, the maximum deflection occurs at the point approximately
1'S? m from the fixed end.

ELECTRIC CIRCUITS.
22. An electric circuit consists of an inductance of 0.1 henry a resistance of 20 ohms arid a condenser of
capacitance 25 microfarads (I microfarad - io farad). Find the charge q and the current i at time t,
given the initial conditions (a) q 0.05 coulomb, i
0 when t 0,
dq/dt
(h) g 0.05 coulomb, i -0.2 ampere when t a,
C.x10l
Since L 0.1, R- 20, C 25 . 10", E(t) 0,
.c
L d2q + B
dt2

reduces to

dt

+ I
C

E(t)

2
T
t2

e 200+ 400,000q 0.
dt

FA2D 01 M
R

Integrating, q e

- loot

(A cos 100 Vt + B sin 100 v"3_9 t

Differentiating once with respect to t,


- dq
-
dt

a)

lODe

-loot

(('B-A) cos 1OO,'t - ( vL4+ B) .lnlOOv'5h].

Using the initial conditions

q 0. 05, i 0

when 0.

A o.os

and B

0.008.

e'(o.os cos 824.5* + 0.008 sin 624.51)


loot
and i - 0. 32 esin 624.5*.

Hence, q

Using the initial conditions q 0.05, i -0.2 when t = 0.

b)

A a 0.05 and B 0.0071.

q = . _ lO0t(O,05 coo 824.5* + 0.0077 sin 624.51)

Hence,
and

too

i 0

t (.. 0.2 cc. 614.5 - 32.0 sin 624.50

Note that q and i are transients, each becoming negligible very quickly.

23. A circuit consists of an inductance of 0.05 henry, a resistance of 20


ohms, a condenser of capacitance 100 microfarads, and an emf of
too volts. Find i and q. given the initial conditions q 0, i - 0
when e .0.
7
dt

dt'

_1 + 400

or

a 100

iooio

+ 300,000q 2000.

dt

Integrating,

Here o.os .i +

00t(4

910
E

.1

Co. 400* + B sin 400*) + 0.01.

C- box iof

Differentiating once with respect to e,


a

a 200, 200t ((_4+28) con 400*+(..B...2A) sin 400t).

Using the initial conditions: 4 -0.01, -4 + 2B 0, and


Then q
and

a (-

a 546-

200t

0.01

B a - 0,005.

Co. 400* - 0.005 sin 400*) + 0.01

sin 4001.

Here i becomes negligible very soon while q, for all purposes, becomes

q 0.01,

ElO0 Cc. 2001

24. Solve Problem 23 assuming that there is a variable emf of


t(t) 100 cos 200*.

The differential equation is now


400

+ 200,000q 2000 co. 200*.

dt'
q

2001(4

Co. 400* B sin 400*) + 0.01 cos 200*


+ 0.005 sin 200*

Then

and

_200*(( 2004 + 400Th Cci 400* + (-200B - 4004) sin 4001]


-2 sin 200*+ cos 20Ot.

C. 100 * 166f

APPLICATIONS OF LINEAR EQUATIONS

153

Using the initial conditions: A - 0.01, -200A + 400B + 1 0 and B -o.00'ts. Then
q

fZOOt(_ 0.01 cos 400t - 0.0075 sin 4000 + 0.01 cos 200t + 0.005 sin Me

e- 200 t (- cos 400t + 5.5 sin 400t) - 2 sin 200t + cos 200t.

and

Here the transient parts of q and i very quickly become negligible. For this reason, when the transients
may be neglected, one needs find only Uie steady-state solutions
q 0.01 cos 200(

and i

0.005 sin 200t

cos 200t - 2

sin 200t.

The frequency 200/2Tt cycles,'sec of the steady-state solutions is equal to the frequency of the applied
emi. (See also Problem 25.)
25. For a circuit consisting of an inductance L. a resistance R, a capacitance C, and an emf E(t) E 0 sin wt, derive the formula for the
steady state current

E0 .R sin
iit

I --(

- - Cos wt) - sin(i.it -0),

where
from

L.__L,
Ci,j

sin

and cos 8
z

By differentiating
and using

z /x 2 + u 2 , and 0 is determined

EE0 ath wt

L-2q

+ A Lq +
C
dt 2 dt

E0 sin

dq
= - . we obtain
dt
d 2 t dt
i
L - + A - + - = (LD 2 + RD + 11C)t
dt
C
dt2

1)

wE0 cos wt.

The required steady-state solution is the particular integral of I):


L

wE0 wE0
COS Wt =
Cos wt

1
LD2 +RD + 11C
A
D-(Lw --)w
Cw
wE0 (RD +Xw)
COS Wt
2 2
R 202 - X w

E0 B
- .- ( sin wt -

E0
A2

Cos wt) =

(R sin wt

-x

cos wt)

2 sin(wt
z

X is called the rL'QCWfl(e of the Circuit; when X 0 the amplitude of t is greatest (the circuit is in resonance). Z. called the impedance of the circuit, is also the ratio of the amplitudes of the emf and the current.
8 is called the phase angle.

At times t
we -8

11/2w. 371/2c.i, ..... .

the emf attains maximum amplitude, while at times given by

lt/2, 37t/2, ------, that is, when

/2+_0; 37t/2 + 0

amplitude. Thus the voltage leads the current by a time

8/w or

the current attains maximum

the current and voltage are out of phase by

the phase angle 0.


Note that 0 0 when K 0 that is, 0 0 if there is resonance.

154

APPLICATIONS OF LINEAR EQUATIONS

26. The circuit consisting of an inductance L. a condenser of capacitance C, and an emf k is known as an harmonic oscillator.
Find q and i when E E0 cos it and the initial conditions are
q . go, i.L 0 when e.o.

E=10 cm tit

L
Since B

0. the differential equation is


2
+

dt 2 CL

Therare two cases to be considered:


_L

(a)

cos t
t + B in _!__ t + 2
L D 2 + 1./CL

q A cos

a)

ICT
i

E 0 C cog wt

t + B sin

A cos

and

(b) w.

and

1-ij?CL

IEI

.-!-(_ A sin -!.- t + B


'VT
ICT

COS --L

1-J2CL

vICT
-

Using the initial conditions: A

E0C
2

E0Cw

t) -

. Then

and B

1-w CL
1

E 0 C

to sin

t +

- 1 2CL
i to

and

-L - ---( q
/ET
4Z

- E0C )sln -_-L -


sin cat.
1w2 CL
1 2 CL
)/E

cost.
+
dt 2 L

Here

b)

smut

Then

q = A cost + B sin wt + --2Lw

and

i = wA sin wt + B cos wt) +(smut


2L
A

Using the initial conditions:


Then
and

E0C Cos wt
2
1 CL

co g wt).

q0 and B

E0
q coswt + s1nit + - t sin wt
2L

q
=

COB

t -

sin wt

+ 0 ( I

2L w

sin wt + t

COB wt).

Note that in (b) the frequency of the cmf is the natural frequency of the oscillator, that is, the frequency
when there is no einf. The circuit is in resonance since the reactance X Lw - -L o when w

The presence of the term


circuit will be destroyed.

cog wt. whose amplitude increases with t, indicates that eventually such a

APPLICATIONS OF LINEAR EQUATIONS

155

SUPPLEMENTARY PROBLEMS
27. Determine the curve for which the radius of curvature is proportional to the slope of the tangent.
Ans. y

(J(x + c

- (x + C)
x +C1

k in

28. A 15 cm pendulum is released with a velocity of 1/2 rad/sec, toward the vertical, from a position 115 rad
from the vertical. Find the equation of motion. (Assume that g = 96 ms 2)

15 cos 8t -

Arts.

16

sin 8t

29. A particular of mass A is repelled from 0 with a force equal to k >0 times the distance from 0. If the
particle starts from rest at a distance a from 0, find its position t sec later.

Arts.

x x

ja(e

/t

+ e

-f7it

30. If, in Problem 29, k . and a 4 m determine a) the distance from 0 and the velocity when
b) when it will be 6 m from 0 and its velocity then.
Ans.

15

a) x

14.5 ITiS'

b) t

0.96 see,

2 sec.

4.47rfls'

31. A chain hangs over a smooth peg, 2 m on one side and 3 m on the other, If the force of friction is equal
to the weight of 025 m of chain, find the time required for it to slide off.
Arts.

/0 4g

(19 4') sec

32. When the inner of two concentric spheres of radii r 1 and r, r 1 < r 2 , carries an electric charge, the
differential equation for the potential V at any point between the two spheres at a distance r from their
common centre is
2
d-V + 2--
d
0.
rdr
dr2
Solve for V given V = v when r r 1 and V = V, when r = r2.
Arts.

V2r2(r-r1) - V1r1(r-r2)
r(r 2 -r1)

33. A spring is such that it would be stretched 6cm by a mass of 4 kg. A mass of tO kg is attached and brought
to rest. Find the equation of the motion if the mass is then
a)

pulled down 8 cm and released.

b)

pulled down 4 c and given an upward velocity of 05 ms'

c)

pulled down 6 cm and given a downward velocity of I ms

d)

pushed up 6 cm and released.

e)

pushed up 8 cm and given an upward velocity of I -25 ins -'

Arts.

a) x

0,08 cos 7,

d) x

21

b) x

-0,06 Cos 7t,

O,O4 Cos 7 - 0,071sin 7r,

e) x

c) x 0,06 cos 71 0,143 sin7t,

-0,08 cos7t -0,178 sin It.

156

APPLICATIONS OF LINEAR EQUATIONS

34. A spring is such that it would be stretched 72 mm by a mass of IS kg. A mass of 30 kg is attached and
brought to rest. The resistance of the medium is numerically equal to 120 dx/dt N. Find the equation of
the motion of the weight if
a)
b)

it is started downward with velocity 3ms


it is pulled down 140 min and given an upward velocity of 3 me

Ans. a)

C-2tsin 89

b)

e2t(O.14eo5 8t -0.34 sin Bt).

35, A spring is such that it would be stretched 77 mm by a mass of 15 kg. A mass of 15 kg is attached and
brought to rest. The mass is then pulled down 4 cm and released. Determine the equation of motion if
O) an impressed force 7 sin 6 t acts on the spring.

Ans.

i '

an impressed force 8 sin 8t acts on the spring.

Ans.

h)

0,04cos 8t -

sin 8t +

0,04 (1 - 12,50 cos 8(

sin 6t
1 sin 8t

36. Abeam of length lm is fixed atone end and otherwise unsupported. Find theequation of the elastic curve and
the maximum deflection if there is a uniform load of w N
of length and a load W N at the free end.
Ans. y

V
(4
24E1

Lx -

2 2
61 x -

x ) (x3 - 3112),
6L1
6

Y$Gx

_L_ 316e' + 8W13)

24E1

37. A beam of length 21 m is freely supported at both ends and carries a uniform load of w N rn of length.
Taking the origin at the midpoint (low point) of the beam, find the equation of the elastic curve and
the maximum deflection. Compare with Problem 15.
Hint: EIy
wI(l-x) - w(I-x) 2
w(12- x 2 ) and y
y' 0 when x
0.
Ans.

y (61 22
x -x
24E1

'S

y'

),

5w1

It

24E1

38. A beam of length 3 m is freely supported at both ends. There is a uniform load of v N of length and
loads of W N at a distance t m from each end. Taking the origin as in Problem 37, find the maximum
deflection.
Hint

hi

w 912

2
___ - x

Ans. Yriox -

-- - x,

31
<x<-..; and hi

w 912

(---- - x )

+ Wl

O<x<j

(405si1 + 366 Wi 5 )

39. A circuit consists of an inductance of 0.05 henry, a resistance of 5 ohms, and a condenser of capacitance
4(10)'S farad. If q r = 0 when t o.findq and z in terms oft when a) there a constant emf 110 volts,
b)
there is an alternating emf 200 cos 100 t . Find the steady-state solutions in b).

Ans. a) 9

e -sot -

11
iiVT
cos 50 v'iit -
sin so1.'Th t> + 11 -,
250
4750
250

16
b) q e -SOt (- -__


5o'igt

170

40. Solve Problem 39 after replacingthe

Ans. a)

1640Vi
sin 50vit) +
323

= 0.98e

,st

0.005e

4.43e

cos

17

loOt + sin lOot).

lOOt -4 sin LOOt)

ohm resistance with a 50 ohm resistance.

t
- 0.047e -55 + 0.002k - 9'S? t + 0.044.

b) q = - 0.018e-53t

19

l2VT
sin 5o/j t)
1615
+ 170

. e -50t (- 40
cos 5OVTt
17

44Th
V
-sot
- e sin 50vi9t

-967t

= 2.46(e

- 'p3

e-

94?

+ 0.034 sin loot + 0.014 cos toot,

+ 3.45 cos loot

- 1.38 sin lt0t

CHAPTER 21

Systems of Simultaneous Linear Equations


IN PREVIOUS CHAPTERS, differential equations involving only two variables have been treated In the
next several chapters, equations involving more than two variables will be considered. If but one of the
variables is independent, the equations are ordinary differential equations; if more of the 'ariables are
independent, the equations are called partial differential equations. In this chapter we shall be concerned
with systems of ordinary linear differential equations with constant coefficients such as

1
A)

2dz
dr + dt

4.; - y

I 2(D-2)x W- 1y

or A)

dx

(D+3)x +

3x y 0

z 0.

where D

dt

and
dx dy
- + -..
dt
dt
dz
dt
LY

dt

di

it

1
+ ( D . 1)y 1
or Bj(D + 2)x - (D - 1)z
1
Dx

2.; z 1

y2a0

in which the number of simultaneous equations is equal

1)y + ( D .# 2)z

10

the number of dependent variables.

THE BASIC PROCEDURE for solving a system of n ordinary differential equations in ii dependent variables
consists in obtaining, by differentiating the given equations, a set from which all but one of the dependent
v ariables, say r . can be eliminated. The equation resulting from the elimination is then solved for this
variable x, Each of the dependent variables is obtained in a similar manner.

EXAMPLE. Consider system A)

I)

dt

cIt

. c,

2) dx + 3z + y
de

D.

So/ui ion t,
First, we note that the general solution
3)

y ( t) of this system will also satisfy

d X
+ 3
dt 2 dt

dt

obtained by differentiating 2), Moreover, multiplying I) by -I, 2) by --I. 3) by I, and adding, we obtain

4 x

cIt2

which is also satisfied by x x ( t) y y ( I). This latter differential equation, being free of y and its
derivatives, may be solved readily; thus,
157

SYSTEMS OF SIMULTANEOUS LINEAR EQUATIONS

158

C1, cos

C2

sin C - -

COS C +

C2

810 C -

'et.

D' + 1

To find y in a similar manner, we differentiate 1) to obtain


2_.4i_4_Z set
dt
de l de l dt

5)

and between this and equations 1), 2), 3) eliminate x and its derivatives. However, it is simpler here to
proceed as follows. From 2), we have
_41-3z -(-C 1 slot + C, coat - e) - 3(C 1 Cost + C,

sint - jet)

dt

( C l - 3C2 )sin

C - (

3C 1

C1 cost + C, sin -

Thus,
general solution.

2cC.

C2)Co8 C

t - ( 3C1

(Cl - 3C,)sin

+C,)cos

C +

2e

is the

When the equations are written in the 1) notation, there is a striking similarity between the procedures
used here and the method of solving a system of n equations in n unknowns. This is due to the fact, noted
in previous chapters, that the operator D may at times be treated as a variable (letter).
Solution 2. Consider the system A')

I)

2(D - 2)x + ( D -

2)

(D+3)x +

1)y

0.

Proceeding as in the case of two equations in two unknowns x and y, we multiply 2) by D - I -Actually.
we operate on 2) with D - 1

dt

- 1), to get
CD 1)(D + 3)x

(0- 1)y

and subtract I) from it to obtain


or

-c

[(D -1)(D+3) - 2(D-2))x

(D' t)x

-e5.

Now this is 4) above as might have been anticipated, since operating on 2) with D -1 is equivalent to differentiating 2) and adding -1 times 2) as in the previous solution The general solution is obtained as in
Solution 1.
Solution 3.
2(D-2)

We may also effect a solution using determinants. From system .4') we obtain
D-1
x

D+3

or

D-1

Cl

(D' + 1)x a

2(D-2)

and

-e t and

1 ly
0-i

0+3
(D + 1)y

2(0-2)

et

0+3

4 e.

The first of these equations is 4) above, and the second would have been obtained by the procedure rejected in Solution 1. We shall now show why it was rejected. When the two equations are solved, we
have
g + C 4 slOt +
C, co
7) y
and
x ' C 1 cost + C, slot -
6)
We know from Solution I that .6) and 7) contain extraneous solutions. To eliminate them (that is, to
reduce the number of arbitrary constants), we substitute in 2) and see that
(C, 3C 1 + Cs) cog C + ( 3C, -C 1 + C4)e111 t 0
for every value oft. Thus,
C, - ( 3C 1 + C,) and C. C 1 - 3C,.
When these values are substituted in 6) and 7), we obtain the general solution found above.

SYSTEMS OF SIMULI ArN tUUZI

LI1r'

THE NUMBER OF INDEPENDENT ARBITRARY CONSTANTS


the system

- -

appearing in the general solution of

f(D)x + g(D)y = h1(t)


f 3 (D)x + g(D)y =

is equal to the degree in D of the determinant A

h(t)
f(D)

91 (D)

F 2 (D)

92 (D)

=
^

provided L does not vanish identically. If A 0, the system is dependent such systems will not be
considered here.
D-1
2(D-2)
=
A
= - (D + 1).
A')
For the system
1
D+3
the number of arbitrary constants appearing in the general solution.
The degree (2) i n agrees with
The theorem may be extended readily to the case of ,i equations in n dependent variables.
SOLVED PROBLEMS
1)

I. Solve the system:

2t

+ Dy

D -

+ 1

2Dy t.

2) (2D 1)x

obtain Dy ' 2t

+ 1 -

(D-1)x

The complete solution is


D-1

Note that

and

x - -

Substituting x a - t -2/3 in I), we

-at - 2.

Subtracting twice I) from 2), we have 3x

4.

C.

is of degree I in B and there is but one arbitrary constant.

2D+1 20

2. Solve the system:

I) (0

+ 2)x + 3y

2) ax

(D+2)y .

(02 + 4D -5)x
Operating on I) with D 2, multiplying 2) by -3, and adding:

Then

C 1 e

3. Solve the system:

From I),

C2e" -

I) (D-3)x
2) 2(0

+ 20 2)y

+ j)x +

(D - 1)y

(D.2)x

Ci e t

-6e

C2e

t +

e2t,

2 in
Cost.

Operating on I) with D-1 and on 2) with 2(0 2) we have


3)

4)

(D-1)(2

(D-1)(D-3)x + 2(D-1)(D+2)y

4(D+2)(D1)x

2(D+2)(D-1)Y

sin

2(D+2)CoSt

t)

2 cost - 2 sin
4

cost - 2 sin t.

since the operators have


Subtracting 3) from 4) and noting that (D - 1)(D + 2) (0 2)(D -1),
constant coefficients,

(4(D'. 3D + 2) - (D t 4D 3 ) J x and

.x -

t/5

ce

Co.

3D'+1ID+5
$ Ce
From 2),

( 3D'+ lID + 5)x

t C

2 cost

..t/3

+ 1- Cos t
8D+1

+ (8 sin t + cos t)/65.

(D-1)y cost , 2(D 1)x

cos t 8C1.
. 8Ce
-t

Then

- t

and

- i C,.

+ (47 cost - 14 sin t)/65.

4
5(8C t . -t C,.
-- C, . -

y -

- (18 cost + 14 sin t)/65

t13

+C11i -'

47

sin t - 33 cost ..t +


S
130

- 14 sin t e_ t )dt

3t
C1 c+C,c- t/ + 61
- sin

- 33 cci
130

+Ce.

Since the degree of A is 2, the general solution has but two arbitrary constants. Hence, when these
expressions are substituted for s and y in 1) it is found that C,
o. Then
+

C1."

8 sin t + cos t
65

is the general solution.

4. Solve the system

4
.. t
y - C1 .

I)

ID' - 2x - 3y

2)

(b2 +2)y+ z

+ 61 sin t - 33 con t
130

-0,

Find the particular solution satisfying the conditions x y 1, t -Dy -o when

I 0.

i
Operating on 1) with D' to obtain D's - 2D 2 x 3Dty - 4e ll
and making replacements D x 2x. 3y + ell
from I) and D 1y - x - 2y from 2). we have (D' - 1)x -do It

Cut + C,e_t +

Then x

Y - 1 r(D' 2)

C l cos t + C4 sin t
U -

and, using I),

!(Cif + Co e_t ) - (C. coo

C sin t> - 1H

Note Oct, x could also be obtained by the use of determinants. Thus,

I
When t 0,

-34 " 3
x -
D'+2
0
D+2

z C, C, C, +

Y - (C 1 + C,) - C. -

13

Then C 1 3/4, C, 7/4. C. - -1/I0, C4


-
-

(D - 1)x ofIt etc.

or
and
- 1

C 1 - C, + C4
and

0.

Lb' - (C1C,) - C+

0.

1/5, and the required particular solution is

(3+ 7_t) - 1 09 c
I(31 7t)
(19

- 2 sin t) + ? .It,
co. t - 2 sin t)-

is

e.

SYSTEMS OF SIMULTANEOUS LINEAR EQUATIONS


5. Sohe the system:

I) (D + 1)x + (0 - 1)y

Operating on I) with

D 2 +D

2y

1 and on 2) with

t2 21 - 3e 1

Operating on I) with D2 -D + 1 and oil


2x C 2 - 2t + e

161

2) (D 2 + D 1)x + (D2 - D 1)y

D + 1.

and

and subtracting, wc have


. t 2 + t -

c.

with 0 - 1, and subtracting, we have


and

i ti - t +

D-1

D+1

Note that

e,

ID+ L + 1 -D+1

2 is of degree o in D; hence, there are no arbitrary constants in

the solution.

6. Solve the system:

I)

2x -

2)

0,

0.

2
y a x

Operating on I) with D and substituting 0 2y _x from 2), we obtain


Dz - a 2 (-a 2 x) = Dx
and x

(C1 cos at/v'

+ ^4 x

C sin at/T2) + e

- (D + a )x

(C3 cos at//i *

0.
C4

Then 0

(1

I)

sin a//).

Substituting for z in I) and soTing.


1 7
- 0 z

52

(C, cos ae/v' - C 1 sin at//) + e-2


e

7. Solve the sstem:

I) D2 + 4)z - My

0,

(C,

sin t/i/ - C4 cos at//).

2) 3 D + (D' + 4)y

0.

Operating on I) with D' + 4 and on 2) with 3D. and adding, we have


((D

+ 4)2

90 2 )x

(D' + 16) (02 + 1)x

Operating on I) with

(L4' 16) (D

-3D

and

= o

and

C1 cos 41 + C, sin 4t + C3 cost + C4 sine.

and on 2) with D2+ 4, and adding, we have


y

K1

cosit + K, sin 4t + K, cost + K 4 sint.

To eliminate the extraneous solutions, substitute for x and y in I). We have


-12C 1 Cos 4t - 12C 2 sjn4t + 3C 3 Cos C + 3C4 sin
for all values oft; thus.

K1

x C,, K,

The complete solution is:

- C 1 , K,

- C4 . K

C,.

C1 cos it + C, sin 4t + C, cos I + C 4 sin


y

8. Solve the system:

12K 1 sin4t - 12K 2 Cos 4t 3K, sine - 31(4 cost = 0

C, cos 4t - C 1 sin it - C 4

I) Dx
(0 + 1)y1
2) (0 + 2)x - CD - 1)z

cos C

C 3 Sifl

C,
C.

1
3) (0 + 1)y + CD + 2)z
0.
Subtracting 3) from I), we have
I which is free of y.
4) N - ( D 2)z
Operating on 2) with 0 and on 4) with 0 2, and subtracting, we have (5D + 4)i -2; then
- C 1
'. Substituting for z in 3). (D + 1)y
-(0 2)z
1 - C5 e: then


SYSTEMS OF SIMULTANEOUS LINEAR EQUATIONS

162

e t

Substituting for y in 1),

D
Since

D+

e(et -

Cj e" )d
Dx

D+i

1 - (D + fly

Cr' ;

+ C,et,

1 -

+ C,)

then x

- Cj c"3

+ C,.

-(D -1)

D + i

D+2

-(5D2 + 9I 4) is of degree 2 in

0,

there are but two arbitrary

constants in the general solution. Substituting for x and z in 2), we obtain


( C1e1 x

+ 2C,) - (-
- C"'-"

0,

- Ce" )

1 - 6C 5 e'

1 and, hence,

C,

= . Thus,

- .! +

+ C,e, z

is the general solution.

1) (D + 1) 2 x + 2.Dy 3Dx

9. Solve the system:

2)

Dx

3)

- Dy - Di 0.

z =0

Find the particular solution for which x z = 1. y=0 when i


First, operate on 2) with 0 to obtain

D 2x + Dx 0.

4)

Next, add twice 3) to I) and subtract 4) to get (2D + 3)x = 1; then

xe

'2 5e''

+ C1e_ 31/2 +

x At

Cie-5t/2.

and

3 Ca -5t/2

- Dx

From 2),
From 3), Dy

I e1t12+

dt

C1

+ .13 C 5 e": then

'2

y =1 t .Ce_3th'2 + C,.
(D+1) 2 2D 3D
D
0
1
Since

2D' + 3D is of degree 2 in

D.

there are 2 arbitrary constants and the

=
general solution is

!+

When =0: x = + C 1

Cje_302, y

1 and

t -

C 1

C1 e

+ C 2 ,

z =

2C"-It/2.

y = (- )() + C2 o

and

C,

Thus, the required particular solution is

2 -5t/2

13 -3t/2
1 t--e

13

+1

-_3t/2

Note that a particular solution satisfying a given set of initial conditions cannot always be found.
For example, there is no solution satisfying the conditions x = 1. y = z 0 when t 0 since x
y = 0 contradicts x 1/3 + 2z/3. Similarly, y O. x 1, dxldt = 1 when t = 0
contradicts dx/dt -z

SYSTEMS OF SIMULTANEOUS LINEAR EQUATIONS

163

SUPPLEMENTARY PROBLEMS
Solve the following simultaneous equations.
10.

f - (D+1)y
x + (L)-1)y

11.

Ans. x (C 1 -C2 )coa t + (+ C 9 )s1ri t +


e

2t

y C 1 co8 t + C9 ain t + 2e

(D+2)z + (D+1)y

5x + (D+3)y
12.

x c 3C2

sin t - 3C9+C2 cost -

t 2 y Ccoa t + C 2 sin t +52t2 - 3t - 4

(D+1) + (2D+7)y

e t +2

2x+(D+3)y.

13.

2t /5 +

CLeIt (COS (t+Cs) - sin(t+C2)) - S


, , + 13

26

y C1 e -

(D-1)x + (D+3)y e
St
(D+2)x + (D+1)y
e + t

z 2C1 e

eln(t+C9 +

4-

17

2e

,- 3 13
17

S 2t
3
C + t
7
1
7

1
__
49

1 St
-t
1
26
y 3C e---e e
+ 1
4-t
17
2
7
49
14.

(D2 + 18)x - 6Dy 0


I
6E + (D + 18)y 0

x C9 cos 2t - C 98in 2t + C,cos 8t

C 1 8in 8t
y C2 coe 2t + Cl ain 2t + C4 cos at - C3 sin 8t

2
(1) + 4)x 4 y 8th 2z

15.

(D 2 + fl y - 2x -Cos 2
16.

(D2+D+1)z + (D 2 +I) ye
(D2 +D)x + D 2-t
ye

17.

(D-1)x + (D+2)y 1 +
(D+2)y +(D+1)z 2 +
(D-l)x + (D+1)z

3+

x - C1 coa ( 4/z + C9 ) + C3 c08 ( t/z

+ C4 ) + cos 2z

y -2C9 cos(V + C,) - C,co8(Vz + C4 )


X

- e -

y=2e

- C,

+e- t

X ' - 1 + te/2 + C,e


y = /6 +

+ -

cos 2,

CHAPTER 22

Total Differential Equations


THE DIFFERENTIAL EQUATIONS
A)

(3x2 y

2 - exz)dx + (2x
3 y + sin z)dy + (y cos z - eX)dz = 0.
(3xz + 2y)cbc + xdy + x 2 dz = 0,

B)
C)

ydx+dy+dz =0,

being of the general form

P(x,y,z,...,t)cbc + Q(x,y,z,...,t)dy + .....+ S(x,y,z.....t)dt

0,

are called total differential equations.


It may be verified readily that A) is the exact differential of
f(x.y,z) = x ) y 2 - e x z + y sin z = C,
C being an arbitrary constant. Such an equation is called exact.
Equation B) is not exact, but the use of x as an integrating factor yields
(3x 2 z + 2xy) dx + x 2 dy + x 5 dz = 0
which is the exact differential of x 5 z + x 2 y = C. Equations A) and B) are called integrable.
Equation C) is not integrable, that is, no primitive

I)

f(x,y,z) = C

can be found for it. It will he shown later (Problem 32) that for such equations a solution I)
can be obtained consistent with any prescribed relation g(x,y, z) = 0 of the variables.

THE CONDITION OF INTEGRABILITY of the total differential equation


P(x.y.z)dx + Q(x,y,z)c + R(x,y,z)dz = 0

2)

is
3)

P(Q -)
Bz By.
EXAMPLE 1.

) + R(
- 2)
Bx- Bz
By Bx

= 0. identically.

See Problem 1.

For equation B).

P=3xzi-2y. BP -2 BP
---=3x; Q=x,

2=0; R=x 2 ,

T. -

Bx

2x BR
=0, and
By

3) becomes (3xz +2y)(Q-O)+x(2x3x) +x 2 (2-1) = 0-x 2 +x 2 = 0. The equation is


integrable.
EXAMPLE 2.

P=y, LP =i,
Y(0 -0) + 1(0 -

For equation C,

=0; Q=1.
+ 1(1 - 0) / 0,

= 0; R=1,

= 0, and 3)becorncs

The eqhiiiion is not integrable.


164

TOTAL DIFFERENTIAL EQUATIONS

165

THE CONDITIONS FOR EXACTNESS 012) are


4)

y BX
EXAMPLE 3.

az

BX

For equation A,.


x
22
P=3x y -ex,

2
=6xy,

.P =--e,
r

= 6x 2 y,

T.
2 = cos x;

oR
x
x
R ycosz-e , - - e ,

- = COG z,

Q = 2xy+sin z,

ZX
ZX

and the conditions 4) are satisfied. The equation is exact.


EXAMPLE 4.

From Example I it is readily seen that 4) is not satisfied hence, equation B) is not

exact.
TO SOLVE AN INTEGRABLE TOTAL DIFFERENTIAL EQUATION in three variables:
See Problem 3.

a)

112) is exact, the solution is evident alter, at most, a regrouping of terms.

b)

11' 2) is not exact, it may be possible to find an integrating factor. See Problems 4-6.

c)

If 2) is homogeneous, one variable, say z, can be separated from the others by the transformation
X

d)

uz,

y =vx.

See Problems 7-10.

If no integrating factor can be found, consider one of the variables, say x, as a constant. Integrate

the resulting equation, denoting the arbitrary constant of integration by 4(z). Take the total
differential of the integral just obtained and compare the coefficients of its differentials with those
of the given differential equation, thus determining (x) . This procedure is illustrated in Problem
13. See also Problems 14-16.
PAIRS OF TOTAL DIFFERENTIAL EQUATIONS IN THREE VARIABLES. The solution of the simultaneous total differential equations

dx + Q1 dy + R, dz

5)

P1

6)

P2dx +

= 0

Qdy + R, di = 0

consists of a pair of relations


7)

f(x,yz) =

C3.

8)

(x,y,z) =

C1.

To solve a given pair of equations:


e) If 5) and 6) are both integrable, each may be solved by one or more of the procedures a) - d). Then,
7), say is the complete solution (primitive) of 5), and 8) is the complete solution of 6).
See Problem 18.

J) If 5) is integrable but 6) is not, then 7), say, is the complete solution of 5). To obtain 8), we use 5),
6), 7) to eliminate one variable and its differential, and integrate the resulting equation.
See Problem 19.

TOTAL DIFFERENTIAL EQUATIONS


g) If neither equation is integrabie, we may use the method of chapter 21, treating two of the variables,
say x and y, as functions of the third variable z.
At times it may besimpler to proceed as follows: Eliminate in turn dyanddz (or any other pair) between
5) and 6) to obtain
P Q

"2

R5


Q. dx

Q.

02 R2

R1 P
IR,

d, = 0,

Q1 R5

dx

dy=0

P,

Q R9

and express them in the symmetric form


9
__
Y

where

Q R

R1 P 1

Q1 R,

R1

z
P1 Q1

Xk

P,

P1

0.

(Note that this is the procedure for obtaining the symmetric form of the equations of straight line when
the two-plane form is given.)
Of the three equations
9')

Ydx = Xdy,

Xdz = Zd,c,

Zdy = Yd:

given by 9), any one may be obtained from the other two. Hence, in olctaining 9), we merely replace the
original pair of differential equations by an equivalent pair, that is, any two of 9').
If two of 9')are integrable, we proceed as me). See Problem 20.
If but one of 9') is integrable, we proceed as in!). See Problem 21.
If no one of 9')is integrable, we increase the number of possible equations. By a well known principle,
dx = ci)' = dx = 1 1 cix + m1 dy + rdz = 12thc + m2 c' + n,dz
X
}'
Z
11X+m1Y+n12
12X+m1Y+n,Z

where the I, m, n arbitrary functions of the variables such that


IX 4 mY + ,iZ 0.

By a proper choice of the multipliers, it may be possible to obtain an integrable equation, say
ci)' = idx+ m dy + n dx
I

JX+.,Y+j

or

a dx + b dy + c dx
aX#bY+cZ

= P& + q dy + r dx
pX+qY+j-Z

If so, we proceed as inf). See Problem 22.


In actual practice, it may be simpler at times to find by means of multipliers a second integrable equation
rather than to proceed as inf). See Problems 23-24.
lfiX+mY+nZ = 0, then also Ithc+mdy+ndz = 0.
If now 1 dx + m di). + n dz = 0 is integrable, we integrate and have one of the required relations.

See Problems 25-29.

TOTAL DIFFERENTIAL EQUATIONS

167

SOLVED PROBLEMS
I. Obtain the condition of integrability of Pdx + Qdy + R dz

0.

Suppose that the gi\cn equation is obtained by differentiating

f(x.y,z)

I)

and, perhaps, removing a common factor.x.y,x). Since from I) Lf dx +

it follows that

p,

Q, and

'ay

dy +

di

Bz

Now assuming the existence and continuity conditions,

A)

By

B) If
z

14

.iL

C)

'

yx

+Q

BX

Bx

BY

BY

Al x +Rt=
x

Bxy

if

By :

Upon multiplying these relations by R.P,Q, respectively and adding,


P


JL(R-4PRQPR
=
;

and the condition P

2.

If p.(x,y,z) -

- ) +

- Q )
Bz

0 follows.

in Problem I, the differential equation is exact. Show that this implies


PQ
x
y

QR
j

?R
x

P
z

These relations follow from A), B), C) in Probi. 1. For example, if u r 1. A) yields
3. Solve (x -y)dx - x dy

!,

= o

= a

Upon regrouping thus x dx - (x dy + y dx) + z dz = o


x2 -

4. Solve y 2 dx - ldy
Here P y 2

ydi

-=2y.

then P(9 -) + Q(

ax

0.

- - -1 -

Since

+ 2

or

x2 -

-. .

the equation is exact.

and integrating, we have

2zy # z = C.

0.

-=0;
) +

x
-

Y2

0.

(_1

-1 .

R =y,

ax

-1) -x(0 -0) +y(2y - . 0) =0

0.

1;
BY

and the equa-

dx z dy 0
tiori is integrable. The integrating factor 1/y 2 reduces the equation to dz+ '
whose solution
Is x

z/y =

C.

168

TOTAL DIFFERENTIAL EQUATIONS

5. Solve (20y + 1)dx + X4 dy + x 2 tan z di

0.

The condition of integrability is satisfied since


(2x 5 y + 1)(0 -0) + x(2x tan -0)

x tan z(2x 3 - 4)

0.

The integrating factor 11x 2 reduces the equation to


(2xy+)dx + x 2 dy + tan di
whose solution is x 2 y -
6. Solve (2x 5

-z)2

+ In see

(2xyd.r+x2dy)+ -L dx + tan zdz o

or

0
C.

dx + 2c 2 yzdy + x(z+x)dz 0.

The normal procedure here would he to show that the equation is integrable and then to seek an integrating factor. By examining for the preceding problems it will be found that, upon using the integrating factor, one variable appears only in an exact differential, for cxarnplc, the variable z in tan z di in
Problem 5.
When the equation of this problem is divided by x 2 z. the variable y appears only in the term 2y dy
which is an exact differential. Thus, we shall use llx l z as a possible integrating factor. The result is
Zdx + 2ydy + dz +

dz - zdx =

whose solution is

x2+ y 2 +Inz

C.

Of course, the separation of the variable here does not indicate that the equation is integrable; for
example, x dx + i dy dz - o is not integrable although x appears only in an exact differential.
7. Show that if P dx + Qdy + A di 0 is homogeneous (i.e., if P Q . R are homogeneous and of the same
degree) then the substitution x ui. y vi will separate the variable z from the variables u and v.
Let the coefficients P. Q . R be of degree n in the variables.
Substituting x uz, y

vi.

the given equation becomes

P(uz vi, z) Cu di + i du] + Q(uz, vi, z) [v di + z dv] + R(uz, vi. z)dz 0.


Dividing out the common factor z' and rearranging, we have
z P(u,

or

v,

1)du + Q(u. v.

1)dv)

+ [(u,v,

z(Pdu + Qj dv) + (uP 1 + vQ1

This may be written as A)

R 1 )dz

1)

+ vQ(u, v, 1) R(u v, 1) )dz

0. where P5 P(u,v,1), etc.

P,du +
Q,dv +

1
+R
UP, +vQ5+Rj

uP 5 .vQ5

di

0 in which

the variable i occurs only in the last term.


Now the condition of integrability for A)

UPS,+ vQ 5

+ A 5

A UP, vQ + )
v

0.

is satisfied provided the original equation is integrable and, when this occurs, the sum of the first two terms
of .4) is an exact differential Moreover, since the third term is an exact differential, A) is an exact differential equation provided only that
Pdx + Qdy + Rdz = 0 is integrable.
8. Solve the homogeneous equation 2(y + z)dx - (x 4 z)dy + (2y -x + z)dz

The equation is integrable since 2(y 2) (-1 -2) - (x + z)(-1 -2)


The transformation x uz,
2z

y - vi

0.

(2y -x + z)(2 + 1) 0.

reduces the given equation to

(v + 1)(u di + zdu) - z(u +1) (v dx + zdv) + z(2v - U + 1)dx

0.


TOTAL DIFFERENTIAL EQUATIONS

169

Dividing by z and rearranging, we have 2z (v + i)du - z(u + i)dv


dividing by z(uu + u + v + 1)
Then

z(u + i)(v + 1).

u+i

2 n(u + 1) - in(v + 1) + in x
(x4. z) 2

(uv+ u + v + 1)dz o or

dv
2 d
- - -- + dx
-
K.

in

x(u + i) = K(v + 1),

or

K(y+z)

0.

v+1

y+z

C(x+z)2.

9. Solve the homogeneous equation yx dx - z2 dy - xy dx 0.


The equation is integrable since yz(-2.z + x) - 2 ( y - y) - xy(z -0)
The transformation x ux, y vi reduces it to
vz 2 (udx+ idu) - 1 2 (vdz+ xciv) - uvx 2 di
Dividing by z 2 and rearranging VI du - xdv - v dx = 0
Then u -

In

v -

10. Solve (2y -z)di

in

in z

K,

vi Ce

2(x -z)dy - (x + 2y)dz

or

o.

0.

du -

o.

y Cc

or

0.

The equation is homogeneous and, by inspection, is seen to be exact since it may be written as
2(ydxtxdy) - (xdx+xdz) - 2(zdy+ydz) = 0,
The solution is Zg y -

xx -

2yz

C.

11. Show that rP + yQ + iR C is the solution of Pdx + Qdy + Rdi = 0 when the equation is exact
and homogeneous of degree n / - i.
First, we check the theorem using the equation of Problem 10. Here
xP + yQ+ iR

x(2y-z) + 2y(x-z) - z(x+2y)

2(2xy-xz-2yz)

and we obtain the solution above.


From xP + yQ +
A)

(P

4,

C. e obtain by differentiation

x - + y - + z )
Bx
Br
B

By

Since the given equation is exact,

By

BR
Y.
Br

Bx

By

BR

Bx

Bi

BQ,
'

Making these replacements, A) becomes


B.'

B) (P+x+y-_+z
B

Bi

dx + (Q + x + y
Bx

LQ
By

Since the given equation is homogeneous,

+y.+x
+ z 2)dy + (R +x
) di 0.
B
By
B
Br

BR
BR
BR
x - + y - + z -
Br
By
Br

to Euler's Formula on homogeneous functions.


Making these replacements, B) becomes
(n + i)P dx + (n + i)Q dy + (n + 1)!? di
or, since n / - 1,

Pdx + Qdy + Rdz

0.

nP, etc., according


170

TOTAL DIFFERENTIAL EQUATIONS

12. Solve ( y 2 +1 2 +2xy+2z)dx + ( x 2 +x 2

( x 2 +y 2 +2xz+2yz)dz

+2xy+2y:)dy

0.

The equation is hcmogeneous of degree 2 and is also exact since


2(y +x)

=.

?.

?/l

2(x + y)

aR

ap

2(x + z)

The solution is x(y2+x2+2xy+2x2)+y(x2+22+2ry+2y1)+z(x2+y2+2x2+2yz).X

or

z(y2+z2) + y(z 2 +x 2 ) +

x(x 2

+y2 )

C.

13. Solve the differential equation Pdz + Qd7 + Rdz - 0 given only that the condition of integrability is
satisfied.
Consider one of the variables, say z, as a constant for the moment and let the solution of the resulting
equation

1)

Pdx+Qdy.o

be

2)

u(x,y,z)

Differentiating 2) with respect to all the variables,

3)

73x

Now

dx + dy + dx .
ZY

;AP and

in3),wehave

pQ, where A

Pd.x +14Qdy +

. d.

Z1

ix,y,x) is an integrating factor of 1). Substituting

dx = d.

But from the given equation A Pdx + jAQdy + uRdz 0 so that


dj = Zu dz - Rdx

- piR)dx.

This relation is free of dx and dy and, using 2) if necessary, can be written as a differential equation
in z and 0.Solving the integral for 0 and substituting in 2), we have the required solution.
14. Solve

2( y + z)dx - (x + :)dy + ( 2y -x + x)dz 0. (See Problem 8.)

We treat z as a constant and solve 2(y + z)dx - (x + x)dy 0 or dy


.
-

L.

x'
+z

integrating factor

"

A)

1
(+

.. ,

x+z

to obtain

(x+z) 2 (x+z)5

dx = -

+
(z+z)2

Differentiating A) with respect to all variables,

dy

(x+z) 2 (x+z)5.

or

- -

d x
(x+z)2

2: (

dx+dz) + dq

(x+z)'

2(y+x)rjx - ( x+z)dy + ( 2y-.x+z)dz + (x+z) 3 d

0.

Comparing this with the given equation, it is seen that (x x )d$. = 0 and
Since, from A).y + z qS(x + 02, the solution is y +x C(x +

'

C.

using the


TOTAL DIFFERENTIAL EQUATIONS
(e2y.e2)dx + (cx +eX)dy * (e),_exy-e)z)dx

15. Solve

The equation is integrable since


( e Xy+e Z ) ( eY_eY+e X +e) Z)
Considering

+ ( eYz+

171

0.

ex) (_eXy_cZ) + (eY_eXy_e)fz)(eX_eX)

0.

as a constant and solving the resulting equation


(eXyd.z + eXdy ) + e Yzdy+e Z dx - 0.

eXy + ez + e2x
we have
Differentiation with respect to all variables yields
(e'y + e 2 )dx + ( e Y: + e')dy + ( f ' + fx)dz
From the given equation.
Thus,

d4

(gXy

(c2y

+ ez + e 2x)dz

cdx and

Ce 2 .
2
Ce.

y
ey*ex+ex

yx dx + (xx -

y2 5

(ey + e y z + e2x)dz.

+ e 2 )d, + ( ez + e X ) dy + (' + e 2x)dz

16. Solve

d.

)dy - 2xy dx

The required solution is

0.

The equation is integrable since yz(x - 3yz + 2x) + (xx - yz 3 )(-2y - y) - 2xy(x Considering y as a constant and solving the resulting equation

x)

0.

yzcLt-2rydz0 or zd.x-2xdz0.

or x =
we obtain
mx - 2lnx = In 0 (y)
Differentiating this result and making the replacement
0.

dx-zdz- 2

x/z2. we have

dx -2dz-z2=Q, or yzdx-2xydx-yxd0.

Comparing this with the given differential equation, we have


(xz-yz)dy + yz 5 d4

Then

0y - y 2 =

K or q6 = f y

22

o or jdy + yd4' - ydy

( 1 z5 - yz5 ) dy + yx 5 &$

(y + X.,y)

0.

X/y, so that the solution is

or 2xy

y2x2 + Cz2.

17. Discuss gcemetrically the solution of the integrable total differential equation
Pdx + Qdy + Rdz = 0.
P0 =

Let ( x0 , y0 . 1 0 ) be a general point in space forwhich not all of


Q(x0 , y0 , z0 ), !t = R(x0 , y0 . 2 0 ) are zero.

P(x0y0,x0).

Q=

Assuming that I', Q, R are single-valued, the set ( P0 .Q,.R0 ) may be considered as direction numbers
of a unique line through the point. Hence, the given differential equation may be thought of as defining
at each point ( z0 . yo. to)
x-
z-x0
Y-Yo
a line
and a plane

P0 (z -x0 )

Q.(y -Yo) + R0 (x -20)

normal to the line.

The solution f(x.y,z)=C of the given differential equation represents a family of surfaces such that
through a general point (xo.Yo. zo) of space there passes a single surface S 0 of the family. The equation
of the tangent plane rtto this surface at the point is

(x-x0)-..-- + ( - Yo)
23

Yo

-of

172

TOTAL DIFFERENTIAL EQUATIONS


and the equations of the normal line L0 are

_i
;;

From Problem 1,

XP,

XQ,

'ay

;;

BZO

,R. Hence, the solution of an integrable total differential

Bz

equation in three variables is a family of surfaces whose tangent plane and normal at each point are
respectively the plane and line associated with the point by the differential equation.

PA IRS OF TOTAL DIFFERENTIAL EQ UA TIONS IN 'THREE VAR/A BLES.

18. Solve the system: (y +z)dx + (z +x)dy + (x 4y)dz 0


(r+z)dx + ydy+ xdx = 0.
Both equations are integrable. The first may be written as
(ydx + xdy) + (xdy + ydi) 4 (xdz + xdx) 0
xy + yz + xx C.

and the solution is

The second may be written as x dx + ydy + (z dx + xdx) o and the solution is


X2 + y 2 +

Thus, zy + yx + xx . C, x 2 +Y

2xz

C,.

2xz C, constitute the general solution.

Through each point in space there passes .a single surface of each of the two families. Since the two
surfaces on a point have a curve in common, the solution of the pair of differential equations is a family
of curves. This family of curves may be given by the cquaions of any two families of surfaces passing
through the family of curves. For example,
xy+yz+zx C, x 2

+y 2

+2(C-xy.yz)

C2

also constitute the general solution.


19. Solve the system:

1) yxdx + xxdy + zydi

2) 2 (dx+dy) + (xx +yz-xy)dz 0.


The first equation is integrable, with solution 3) xyx

C1 . but the second is not.

Multiply 1) by x, multiply 2) by y, and subtract to obtain x(y -x)dy y2(z -x)dz


Multiply this by yx and substitute xyz . C
from 3). The result is
2 2 (y2 z-C 1 )dy

+ y 2 (yx 2 -C1 )dz

= 0

z dy + ydx - C 1 (1 +

or

0.

=0

y2

whose solution is 4) yx + C1 ( 7+2


)
YX

C,.

Equations 3) and 4) constitute a general solution. However. 4) may be replaced by the simpler form 4')
+22 = C,.obtained from 4) by substituting for C1.
20. Solve the system: dx + 2dy - (x +2y)dx - 0
2dz+ dy + (x-y)dx=0.
2 -(x + 2y)
Here

X = X

3,\x.
a

1(x + 2)')
Y A.

x-y

1 2

-3'.(x + y)

x-y

=
21

For the choice X- -1/3. X. -x, Y= z+y. Z. i, and we write the system in the symmetric form
-x

x+y

dz


TOTAL DIFFERENTIAL EQUATIONS
From the integrable equation
From the integrable equation
Thus, z + in x

C. x 2

21. Solve the system

-x

x in x

173
C1.

. we obtain

-x x+y

X2 +

2xy C,.

2zy C, constitute the general solution.

. .-L.
X

we obtain

-2

X+Z

Find the equations of the integral curves through the points

a) (1, 1,1) and b) (2,1.1).


Consider the equations

. .1. and
-Z

_ 8L
.

. The first is integrable and yields

xx

C 1 . The second

is not integrable but is reduced to dy


(1 + C 1 /x 2 )dx by the substitution z C 1Jx. Integrating, WC
have
y x - C jJx + C, or, substituting C - xx, y - x + z
C,.
Thus, xx C 5 , y - x + z C,
constitute the general solution.
The integral curve through the point 1,1,1 is the intersection of the hyperbolic cylinder xz 1 and
the plane y -x + x 1. The integral curve through (2,1,1) is the intersection of the cylinderxx 2and
the plane y - x +1 0.

dx

dy

22. Solve-y-z

dx
-

y-x

2-X

No equation is integrable. By means of the multipliers I

di

At

1. n 0, we obtain

Zdz+ady+ndz
Z(y - z) + a(2 -x) + ri(y-x)
y-x

A)

dx+dy - dz o.

oy-x
r

Then

x+y-x C.

dy we obtaindx
Using Aj to eliminate z in dx - - y - z z-x

]n(x-C) + ln(y-0 5 ) In C 2 ,

or

B)

- . Then

C-x YC5

C,, or, eliminating C by means of A),

(x-05)(y-C1)

(2-y)(2X)

C2.

A) and B) constitute the general solution.

x2dx

dx
'
X7
From the integrable equation

23. Solve -

x2dx

or x5 dx - y 5 dy - O,we obtain

5
i--

Y x 5

X6

A)

C1.

We may then use A) to eliminate x in the non-integrable equationdi 2


to use the multipliers I * i, n o to obtaindx - x 2 d.x +
Th en
2

dx

24. Solve the system


x

Using 1 *

1,

4)'

dy -.
- 2.1)'

0, we obtain

2 . However, it is simpler
x +y = C,z5.

dx

(x+y)z
di
(x+y) 5 z

dx+dy
(x+y)

Cr

(x + y) (dx +

dy). Then


TOTAL DIFFERENTIAL EQUATIONS

174

(x +y) 2 - 2 In x - C1.

Using

- a -1, n 1 =O and 1,'1.


at

_L

1
x+y

25. Solve the system


The equation

+ K

-x

Y
dx

x-y

+ dy

dx - dy

(x+y)2

(x-y)2

n,*0. we obtain dx

Then

2
y C1 (x - y ).

or

-3y

dy
2
2
or xdx + ydy 0 is integrable and we obtain x +y C1.
-x

Using I 3. a -2. is - t, we find 3(y) + 2(-x) + (2x -37) 0. Hence, 3dx + 2dy+ dx - 0 and
3x + 2y +x C1.
dy
4x-22

dx
4y-3z

26. Solve the system

dx
2y-3x

We seek multipliers Z.*.rs such that A) 1(4y -3z) + .(tz - Zi) + n(2y -3x) - 0.
Rearranging A) in the form (4a - i)x + (41 + ) y + (-31 - )x - 0, we see that it will be satisfied when
4. -

or I : ^ :n 2:-3:-4. Then

- 0, 41 + 2n 0, -31- 2a - 0

2x-3y-4zC1.

2dx-3dy-4dz0 and

Using the arrangement 4(Ly + ax) + 3(- Is - ax) + 2(n' - ax) = 0 and setting Zy + ax = 0.
-lx -ax - 0, ny - ax - 0. we obtain 1 : a : ,s z : -y : -x. Then

xdx-ydy-zdx=0 and x2-y2-x'-C2.


21. Solve the system

p dx
(q-r)yz

q dy

r dx

(r-p)XZ

(p - q)xy

Consider
I -.r)yx + ( r -p)x: + n(p -q)xy 0.
From q(lyx - rsxy) + r(axz -lyx) + p(nxy-axz) - 0 we obtain 1: a: a x :y: z. Then
px2+qy2+r:* Cj.

and

pxdz + qydy + rid: - 0

From x(Iqy -spx) + y(rspz - in) + x(w- x -nqy) - o we obtain 1: a: a -px : qy : ni. Then

28. Solve the system

Using
Using

dx
2
2
Z +7 -yx

and
or

dy
2
2
-x -y +x:

C2.

(x - y)x

22
y 2 + xi) - (x -y)x 2 0. Then
1 =. -i, n --1. we obtain (x + 7 2 - yx) + (-x
and
x+y-x=Cj.
dx+dy-dx=0

1 -xx.

a=yz,
xz(x

Then

p2xt +q ty2 +r 2 x2

and

ptxdx + qtydy+ r2 zdx 0

ax-(x t +yt ), we find


2

+y -yx) + yx(-x -

xxdx + yzdy - ( x t +y2 )dx . 0 or

xdx4ydy - dx

ln(x2 + y' - 2 In x - in Co
2

+rz) - (x +y )(x-y)x 0.

x +y =C, x.

+ 7 2

175

TOTAL DIFFERENTIAL EQUATIONS

dx

29. Solve the system

we have xy 2 . C. By inspection,

From

2y() +

2yz(-y

y 2 (4xy t 2z) - 0;

then 2y'dx + 2yx dy - y 2 dx 0 or

- 2YX

Zdx -

30. Discuss geometrically the general solution of

0,

and

it - z

=
Il
Q
For convenience, let us assume that in solving the given system we have obtained a pair of integrable
P

equations

Pd* + Q1 dy + R j dx a 0 and

P,dx + Q2 dy

+ Rdz 0

whose integrals are respectively


(x.y.z) - C 1

and

h(x.y.x) - C,.

Through a general point (z0 . Ye ' Z) of space there pass two surfaces (one of each of the above families)
whose curve of intersection Co is the integral curve of the given system through the point. The tangent
( P5 ,Q5 ,R 1 ) and (P,.Q,.R,)evaluated
planes to the two surfaces at (x. yo. Xo) are normal to the directions
(X. Y. Z) be
at the point, and the line of intersection L 0 of these planes is normal to the rwo directions. Let
a set of direction numbers for L;then

R i l , JR, Pu

Q2

R,

n, P,

Qu

IN

Q,

are proportional to P,Q,R (all evaluated at the point).


since the tangent to a space curve at one of
Now L0 is the tangent to Co at (x0 .y0 .z0 ).
its points lies in the tangent plane at the point of any surface containing the curve, Hence, the integral
curves of the system - consist of a doubly infinite system of curves characterized by the fact
that at any point (xo,yo, e) the tangent to the curve through the point has (Po.Qo,Pso) as direction
numbers.
and the family of integral curves of
31. Show that the family of integral surfaces of 1) Pdx + Qdy + Rdz 0
2)

dx

dy -dx are orthogonal.


This follows from the fact that at any general point (x 0 . yo , so ) the direction (P0 .Q.Ro) is:
a) normal to the integral surface of!) through the point (see Problem 17) and
b) the direction of the integral curve of 2) through the point (see Problem 30).

x+y+2Z)dx
d)xy.a.

32. Solve I) ydx+ xdy -

C) x+y0.

o consistent with a) z a,

b) x + y +

2z

0,

Equation I) is not integrable. From each given surface we may obtain an integrable total differential
equation. Our problem then is to solve this differential equation simultaneously with I) using the particular solution of the former rather than the general solution as inf) of the introduction of this chapter.
a) Here z a, di =0, Substituting in I), we obtain ydx+xdy 0; then zy C.
Equations z a, xy C are said to constitute a solution of I).

176

TOTAL DIFFERENTIAL EQUATIONS


b)

Substituting x+y+2x 0 in 1), we obtain yd.x+xdy


o and xy = C.
The solution is xy = C. x+y+2z 0.

c)

Here y -x, dy -dx. Substitutingin I), we obtain x


dx #z dx 0 and x 2
The solution is x t + z 2 = C. z + y 0.

d)

Herexy a zdy + yd,- o.Equation 1) reduces to


(x +y + 2x)dz 0.
Then, either x + 2y # 2z - 0 or dx. a and x C.
zy a, x +y + 2x a and z C. xy a constitute the solution

33. Discuss geometrically the problem of solving Pdx+Qdy+Rdz

o, we obtain

We solve the system Pdx Q dy + Rdx 0.


(x.y,x) . 0 of the latter. Let

,ax

f(x,y,z) 0 C.

dx + N dy +

dx +

By

dy

C.

0 consistent with the given relation

g(x.y,z) 0.

From the relation g(x,y,z

+ 2

Bz

zz

di 0.

dx = 0 using the particular solution

g(x,y,2) 0

constitute the solution. The integral curves are those cut out on the surface g(x,y, x) 0 by the system
Of surfaces f(x,y,z) C. Thus, Problem 32c may be stated as: Find all curves lying on the surface (plane)
+ y 0 which satisfy the differential equation
ydz xdy - (x+y+2x)dz -0.

At a general point ( x0 , y0 , 20 ) on the surface j(x,y,z) 0,


the line of intersection l..o of the tangent
planes to g(z,y,x) oand the surface of the system f(x.y,x)
c, through the point, is tangent to the curve
of intersection of the two surfaces. Thus, we have found the family of curves on the given surfaceg(x,
7,3) 0 whose tangent at any point lies in the plane, through this point, determined by the differential
equation.
(See Problem 17.)
For example, consider Problem 32c. On the prescribed surface x +y 0,
choose any point (a,-a,b).
At this point, the tangent plane x + y 0 (here, the plane itself) is normal to the direction
(1. 1,0) and the
tangent plane, to the surface (of the family) x2 + It a + 62 is normal to the direction (a.O.6).A set of
direction numbers for the line of intersection L of these planes [the tangent to the curve through (a,-a,
b)J
is (-b,b,a).
Now the plane through (a,-a, b) determined by the given differential equation is normal to the direction
- (-a,a,-2b). Since (-b,b.a) and (-a,a,-26) are normal directions, the line
L
lies in the pane determined by the differential equation.
34. Solve 1)

22

dx. dy+ydz 0 consistent with 2) x +y +x 0.

From 2), y ' -x - x and dy -dx - dz. Substituting for y and dy


in I), we obtain
3)
(2x-1)dx - (x+z+i)d.x 0.
The transformation z - Zt + 1/2, x xj - 3/2 reduces 3) to
4) 2zt dx 1 - (Xt + 2 1 )d.z 1 - o. a homogeneous equation.
The transformation Xj l&Zt reduces 4) to (u - 1)d 1 + 2z1 du - 0 or
+
- 0.
t j
u-i
Then
1n x+ 21n(u-1)aJnK
or
Replacing a by x 2Jx j , xx by x + 3/2 and

by z - 1/2, this becomes (x -z + 2) C(2z -1).

TOTAL DIFFERENTIAL EQUATIONS

177

SUPPLEMENTARY PROBLEMS

Test for integrability and solve when possible.


Ans. xy+2yz +3xz -C

35.

(y+32)dx + (x +2z)dy + (3x +2y)dz

36.

(cos x + e x y)dx + (e

37.

dx + (x+z)dy # di 0

y + ln(x+z)

38.

z3dx+zdy-2ydz0

xx

39.

x 2 d.x z 2 dy

Nol integrable.

40.

(x + z)2 d7 + y"

41.

2x(y+z)dt+(2yz-x

42.

j z ux 2xz dy + xy di 0

i..'.

xd.x + ydy + (x

44.

2(2 2 -yz-2 2 )dx + xz(x+z)dy + x(2 2 - x 2 -xy)dz 0

e X y + ei + sin z C

e y z)dy + ey di 0

xy di 0
(d.

di)
2

.y -

Cz

y(x.x) C(x+y+z)

+y

2- x 2

2
)dy.(2yz-x -y 2 i-i 2)dz0

+ 2 - C(y+z)

+y 2 +1

y Cxi
2
22
(x +7 +z)e

+1)zdi 0

(x+y)/z + ( 7+2)12 - C

Solve the following pairs of equations.

45.

dx+dy+(x+y)dx -0
z(dx+dy) + (x +y)dz 0

46.

2yzd.x+x(zdy+ydz) -0
ydx - x?zdy +ydz -o

Ans.

2
2
xyz -Cj. xz+x+x -C,

47.

49

xi

yz

dx
3y-2z

dx
( 2
2
2

dy
2xy

-y )

di
2xz

-z

y -z -C,

2 2
33
z xy + x 7 + x y + C,

C1 ,

1,

y-0 5 z.

di

dy
z -3x

x(2y -x )
53.

Ci , x

(x+y)(I+2xy+3x2y2)

dx

52.

- y

ax 2

xy
di

dx
2_2-2

51.

14

ii

72

48.

x + y - C 1 e x + y C,/z

2x-y

y(x

di

-z j

+7

+2

7
XZ

14

Cl.
.

+ 7

Is

C22

+7

z y -x )

xyz C1 ,

+y

+2 2

I
+2 1,j

z(x -y e
di

dy
2

x+2y+3z-C1 . 2

dy
;
y(z -2x )

2
+z -C2

Vous aimerez peut-être aussi